You are on page 1of 130

Titu Andreescu Vasile Cirtoaje

Gabriel Dospinescu Mircea Lascu

and

New
Inequalities
VOLUME 1
GIL
TITU ANDREESCU GABRIEL DOSPINESCU
I VASILE CIRTOAJE MIRCEA LASCU

Old and New


Inequalities

propriety of
GIL Publishing House
published by

/■ EDITORA

CAtSMtA,
©GIL Publishing House

OLD AND NEW INEQUALITIES


Authors: Titu Andreescu, Vasile Cirtoaje, Gabriel Dospinescu Mircea Lascu

ISBN 973-9417-35-3
Copyright © 2004 by GIL. All rights reserved.

National Library of Romania CIP Description


ANDREESCU, TITU
Old and New Inequalities/ Titu Andreescu, Vasile
Cirtoaje -Zalau: GIL, 2004
p.; cm.
Bibliogr.
ISBN 973-9417-35-3

I.Cirtoaje.Vasile
II Dospionescu Gabriel
III Lascu Mircea

51(075,35)

GIL Publishing House


P.O. Box 44, Post Office 3, 450200, Zalau, Romania,
tel. (+40) 260/616314
fax.: (+40) 260/616414
e-mail: comenzi@gil.ro
www.gil.ro
Preface

This work blends together classic inequality results with brand new problems,
some of which devised only a few days ago. What could be special about it when so
many inequality problem books have ahead}' been written? We strongly believe that
even if the topic we plunge into is so general and popular our book is very different.
Of course, it is quite easy to say this, so we will give some supporting arguments. This
book contains a large variety of problems involving inequalities, most of them difficult,
quest ions that became famous in competitions because of their beauty and difficulty.
And, even more importantly, throughout the text we employ our own solutions and
propose a large number of new original problems. There arc memorable problems
in this book and memorable solutions as well. This is why this work will clearly
appeal to students who are used to use Cauchy-Schwarz as a verb and want to further
improve their algebraic skills and techniques. They will find here tough problems,
new results, and even problems that could lead to research. The student who is not
as keen in this field will also be exposed to a wide variety of moderate and easy
problems, ideas, techniques, and all the ingredients leading to a good preparation
for mathematical contests. Some of the problems we chose to present are known,
but we have included them here with new solutions which show the diversity of ideas
pertaining to inequalities. Anyone will find here a challenge to prove his or her skills. If
we have not convinced you, then please take a look at the last problems and hopefully
you will agree with us.
Finally, but not in the end, we would like to extend our deepest appreciation
to the proposers of the problems featured in this book and to apologize for not
giving all complete sources, even though we have given our best. Also, we would like
to thank Marian Tetiva, Dung Tran Nam, Constantin Tanasescu, Calin Popa and
Valentin Vornicu for the beautiful problems they have given us and for the precious
comments, to Cristian Baba, George Lascu and Calin Popa, for typesetting and for
the many pertinent observations they have provided.

The authors

3
Contents

Preface 3

Chapter 1. Problems 7

Chapter 2. Solutions 25

Glossary 121

Further reading 127

5
CHAPTER 1

Problems

7
8 Problems

1. Prove thal the inequality

y/a2 4- (1 - b)2 4- y/b2 + (1 — c)2 4- x/c2 4- (1 - a)2 > —y-


holds for arbitrary real numbers a, b, c.
Komal

2. | Dinu Sjerbanescu ] If a, b, c € (0,1) prove that


y/abc + \/(l — o)(l - b)(l - c) < 1.
Junior TST 2002, Romania

3. [ Mircea Lascu ] Let a,b,c be positive real numbers such that abc = 1. Prove
that
b 4-. c. r. 4- n
>^+y/b+y/~c + 3. ■
4- —;— 4
a x/b v/C
Gazeta Matematica

4. If lhe equation .r4 4- ax3 + 2x2 + bx 4- 1 = 0 has al least one real rool, then
a2 + b2 > 8.
Tournament of the Towns, 1993

5. Find the maximum value of the expression x3 -I- y3 4 z3 — 3xyz where x2 4- y2 4-


z2 = 1 and x, y, z are real numbers.

6. Let a,b,c,x,y,z be positive real numbers such that x 4- y 4- z = 1. Prove that ‘ •


ax 4- by 4- cz 4- 2 y/(xy + yz + zx)(ab 4 be 4 ca) < a 4- b 4- c.
Ukraine, 2001

7. [ Darij Grinberg ] If a, b, c are three positive real numbers, then


a b c 9
(b+c)2
H---------------- 2 + (a 4 b)2 > 4 (a + b + c)’
(c 4- a)

8. [ Hojoo Lee ] Let. a, b, c > 0. Prove that


x/a’ 4 a2b2 4 b4 + y/b4 + b2c2 + c4 4 x/c4 4 c2a2 4 a4 >

> ay/2a2 I be. I by/2b2 4- ca I- c\/2c2 4- ab.


Gazeta Matematica

9. If a, b, c are positive real numbers such that abc = 2, then


4- b^ 4“ > ci\/b 4" c + 6\/c *}■ 4* c^/o. 4~ b.
Old and New Inequalities 9

When does equality hold?


JBMO 2002 Shortlist

10. [ Ioan Tomescu ] Let x^y,z > 0. Prove that


___________ xyz___________ ^^4-
(1 4- 3i)(x + 8y)(y 4- 9z)(z + 6)
When do wc have equality?
Gazeta MatematicA

11. [ Mihai Piticari, Dan Popescu | Prove that


5(a2 I b2 I c2) < 6(a3 4- b3 4 c3) I 1,
for all a, b, c > 0 with a 4- b 4- c = 1.

■ 12. | Mircea Lascu ] Let xj,X2, ...,xn 6 /?, n > 2 and a > 0 such that xi i
2a
X2 + ■ • ■ + xn = a and x? 4- x? 4- ... 4- x2 < ----- Prove that x, € 0. — , for all
n- 1 n
i € {1,2, ...,n}.

13. [ Adrian Zahariuc ] Prove that for any a,b,c € (1,2) the following inequality
holds
b\/a cVb a\/c
-I- I- > I-
4by/c — cy/a 4cy/a - aVb 4a\/b - b>/c.

14. For positive real numbers a, b,c such that abc < 1, prove that
abc
7 4- - 4— > a 4- b 4- c.
b c a

15. | Vasile Cirtoaje, Mircea Lascu | Let a,b, c, x,y,z be positive real numbers
such that a + x > b+y > c+z and a4-b4-c = x + y + z. Prove that ay + bx > ac+xz.

16. [ Vasile Cirtoaje, Mircea Lascu ] Let a, b, c be positive real numbers so that
abc = 1. Prove that
3 6
1 4- > ab 4- ac + be'
a 4- b + c
Junior TST 2003, Romania

17. Let a,b, c be positive real numbers. Prove that


a3 b3 c3 a2 b2 c2
4-
lr + c*-2 + a-
tt
b + c— 4a— ’
“^7*
JBMO 2002 Shortlist
10 Problems

18. Prove that if n > 3 and Xi,X2,...,xn > 0 have product 1, then
1 1 1
------------------------- f. -------------------------4-
j- •.
•••• +
4- ---------------------- > 1.
1 I Xj I T1X2 1 I r-2 I X2X3-------------- 1 !• Xn I XnXj
Russia, 2004

19. | Marian Tetiva | Let x, y, z be positive real numbers satisfying the condition
x2 + y2 4- z2 + 2xyz = 1.

Prove that
1
xy: <
3
b) x4- y + z <
, “3 222
c) xy 4- xz + yz < - < x + y + z ;

d) xy + xz + yz < + 2xyz.

20. | Marius Oltcanu ] Let X1.X2.X3, 14,15 € R so that ii -fio 1 X;j 4 X4 4-15 = 0.
Prove that
|cosii| +-ICOSX2I + ICOSX3I + ICOSX4I + I COS X51 > 1.
Gazeta MatematicS

21. [ Florina Carlan, Marian Tetiva ] Prove that if x, y, z > 0 satisfy the condition

x 4- y 4- z = xyz
then xy + xz 4- yz > 3 4- y/x2 + 1 4- ^/y2 4- 1 + y/z2 4- 1.

22. [ Laurenpu Panaitopol ] Prove that


1 + x2 1 4- y2
4-
1 4 y I- z2 ' 1 4 z I x2
+ 1 + x h y2
>2,

for any real numbers x,y,z > -1.


JBMO, 2003

23. Let a, b, c > 0 with a + b + c = 1. Show that


a2 + b b2 + c c2 + a
>2.
—------- 4-------------- 1---------- 7-
b I c c4- a a+ 0

24. Let a,b,c > 0 such that a4 + b4 + c4 < 2(a2b2 + b2c2 4- c2a2). Prove that
a2 4- b2 4- c2 < 2(ab + be + ca).
Kvant, 1988
Old and New Inequalities 11

25. Let n > 2 and X\,.. xn be positive real numbers satisfying


1
xi + 1998
+ x2 + 1 1998 ++, ...
"■ ++ x
rnn +
1 1
1998 ” 1998 ‘
+1998
Prove that
tfx1x2...xn > 1998.
n- 1
Vietnam, 1998

26. [ Marian Tetiva ] Consider positive real numbers x,y, z so that


x2 + y2 + z2 = xyz.
Prove the following inequalities
a) xyz > 27;
b) xy + xz + yz > 27;
c) x + y 4- z > 9;
d) xy +■ xz 4- yz > 2 (1 + y 4- z) + 9.

27. Let x,y,z be positive real numbers with sum 3. Prove that
\/x + y/y + \/z > xy + yz ~ zx.
Russia, 2002

28. [ D. Olteanu ] Let a,b,c be positive real numbers. Prove that


a + b • ■ "'a

b 4- c 2a
-f-
b 4- c■ ■ •
+ b c c + a 2b4-c4-a
2a4-b-l-c
b
2b + c + a
■' “f-
c 4- a
a4-b
c -—
, —■■

2c4-a4-b 4
Gazeta MatematicS

29. For any positive veal numbers a, b, c show that the following inequality holds
a b c c+ a a+b b+c
b c a ~ c+ b a 4- c b+a
India, 2002

30. Let a, b, c be positive real numbers. Prove that


a3 3(ab + be -r ca)
b2 — be + c2
b3 c3
+. c2 - ac + a2
4- >
a2 - ab 4- b2 ~ a+b+c
Proposed for the Balkan Mathematical Olympiad

31. [ Adrian Zahariuc ) Consider the pairwise distinct integers xi.xj, ... ,x„,
n > 0. Prove that
x'j 4- x'2 + • • • 4- i2 > iii2 + Z2I3 + ■ ■ • + znii 4- 2n - 3.
12 Problems

32. [ Murray Klamkin ] Find the maximum value of the expression xjx2 + X2X3 4-
• • ■ -t- ,r’_ ,.r„ 4- I?!, when X1.X2 xn-t.xn > 0 up to I and n > 2.
Crux Mathematicorum

33. Find the maximum value of the constant c such that for any
Xj,x2,...,x„,• • • > 0 for which Xk+i > Xi 4- X2 4- • • • 4- x«.- for any k, the inequality

x/77 + y/^2 + • • ■ + \/^? < Cv/.Tj 4- x2 4- • • ■ 4- .rn

also holds for any n.


IMO Shortlist, 1986

34. Given are positive real numbers a, b, c and x, y, z, for which a-l-x — b + y =
c 4- z = 1. Prove that
1
ay bz
Russia, 2002

35. [ Viorel Vajaitu, Alexandru Zaharcscu ] Let a, b, c be positive real numbers.


Prove that
ab be ca 1
+ +
o + b -l- 2c b + c 4- 2a c 4- a 4- 2b
< -(a + b + c).
4
Gazeta Matematica

36. Find the maximum value of the expression

a '(b 1 c + d) + b'(c 4 d + a) + c'1 (d + a 4- b) + d‘*(a -I- b c)

where a,b.c,d arc real numbers whose sum of squares is I.

37. [ Walther Janous ] Let x, y,z be positive real numbers. Prove that
x
........... ............. ......... 4- y ----------- + -------- ----z -----------
4--------- z. 4- - < I-
x 4- \/{x 4- y)(x 4- ~) y3/ 4- vZ/(y (l/ + z){y + x) z 4- i/(z 4- x)(c 4- y)
Crux Mathematicorum

38. Suppose that Oi < as < ... < an are real numbers for some integer n > 2.
Prove that
ciicij + asa^ + ... + anaf > a? a' + <13(12 d- d* <ii<in-
Iran, 1999

39. [ Mircea Lascu ] Let a, b, c be positive real numbers. Prove that


b4-c + c4-a + a4-b>^/ a b c \\
a b c “ \b4-c
b+ c c4-a
c+ a aa + b) '
+ bJ
Old and New Inequalities Id

40. Let ai.as,... ,a„ > 1 be positive integers. Prove that at least one of the
numbers “{/aL “^03,.... “t/aj is lass than or equal to >/3.
Adapted after a well-known problem

41. [ Mircea Lascu, Marian Tetiva ] Let x, y, z be positive real numbers which
satisfy the condition
xy 4- xz 4- yz 4- 2xyz - 1.
Prove that the following inequalities hold
a) xyz-<
3
b) x + y + z>~;

c) - 4- - 4- - > 4 (x 4- y 4- z);
x y z
oil 14,, (2z-l)2
d) -4----- 1------ (x 4- y 4- z) > , where z = max {x, y, z}.
x y z (z(2z+l))

42. [ Manlio Marangelli ] Prove that for any positive real numbers x,y,z,
3(x2y + y2z 4- z2x)(zy2 + yz2 + zx2} > xyz(x 4- y + z)3.

43. [ Gabriel Dospinescu ] Prove that if a, b,c are real numbers such that
max{a,6,c] - min{a,b, c} < 1, then
1 + a3 4- b3 I- c3 -I- Gabc > 3a2b 4- 3b2c I- 3c2 a

44. [ Gabriel Dospinescu | Prove that for any positive real numbers a, b, c. we have

> 6(n 4- b 4- r) - f 7 + - ).
ka b c)

1 a2 1
45. Let do — - and a.k~i — o.k 4—Prove that 1---- < an < 1.
2 n n
TST Singapore

46. [ Calin Popa ] Let u,b, c be positive real numbers, with a,b, c € (0,1) such
that ab 4- be 4- ca = 1. Prove that
a b c 3 I-a2 1 -b2 1 -c2
1 - a2
+ 1 - b2 4- 1-c2 >~ -4 --------
a
4-
b
4- -------

47. [ Titu Andreescu, Gabriel Dospinescu ] Let x,y,z < 1 and x I y I c = 1.


Prove that
1 1 1
1 4- x2 + 14-y2 4- “ 10
14 Problems

48. | Gabriel Dospinescu ] Prove that if y/x + y/y + yfz = 1, then


(1 - x)2(1 - t/)2(1 - z)2 > 215xyz(x + y)(y + z)(z + x)

49. Let x, y, z be positive real numbers such that xyz = x + y + z + 2. Prove that
(1) xy + yz + zx > 2(x + y + z);
3
(2) y/x + y/y + y/~Z < - yfxyz.

50. Prove that if x, y, z are real numbers such that x2 + y2 + z2 = 2, then

x + y + z < xyz + 2.
IMO Shortlist, 1987

51. [ Titu Andreescu, Gabriel Dospinescu ] Prove that for any xj,X2, ...,xn 6
(0,1) and for any permutation u of the set {1,2,... ,n}, we have the inequality

/ A \
n / n
1
S t— >
- Xi
1 + ——
71 — Xi • Xo(i)

\ /
n
1
52. Let X].X2,...,xn be positive real numbers such that = 1. Prove
i=l
that

Vojtech Jarnik

53. [ Tilu Andrccscu ] Let n > 3 and ai,o.2, ...,an be real numbers such that
ai -f az 1.. .-Hn > n and a2 + a% I-... I d„ > n2. Prove that max{aila2,..., an} > 2.

USAMO, 1999

54. [ Vasile Cirtoaje ] If a,b, c, d are positive real numbers, then


a-b b-c c-d d-ctn
b+c c+ d d+a a+b

55. If x and y are positive real numbers, show that xv + yz > 1.


France, 1996
Old and New Inequalities 15

56. Prove that if a, b,c > 0 have product 1, then


(a + b)(b + c)(c 4- a) > 4(a 4- b 4- c — 1).
MOSP, 2001

57. Prove that for any a, b, e > 0,


(a2 4- b2 + c2)(a + b - c)(6 + c - a)(c 4- a - b) < abc(ab + bc+ ca).

58. [ D.P.Mavlo ] Let a,b,c> 0. Prove that


1 1 1 « b c „ (a4- l)(b4- l)(c4-1)
3 + a + b + c+ —I- 7 4— + 7 + - + — >3
a b c b c a 1 4- abc
Kvant, 1988

59. [ Gabriel Dospinescu ] Prove that for any positive real numbers ii,X2,... ,xn
with product 1 we have the inequality
n

n” ■ (x? + 1) > 4- 52 .
i=l \i = l i=l /

60. Let a, b, c, d > 0 such that a + b 4- c = 1. Prove that


11 d
a3 4- b3 4- c3 4- abed > min 4’ 9 + 27

Kvant, 1993

61. Prove that for any real numbers a,b,c we have the inequality
57(1 + a2)2(l 4-b2)2(a - c)2(b- c)2 >(14- a2)(l + b2)(l + c2)(a- b)2(b- c)2(c- a)2.
AMM

62. [.Tit.u A.ndrcescu, Mircea Lascu ] Let a, x, y, z be positive real numbers such
that xyz = 1 and a > 1. Prove that.
xa Va
y zQ 3
-------- 1- —— 4--------->
y+z z+x x+y 2

63. Prove that for any real numbers xi,...,xn,yi,...,yn such that zj4----- i-z2 =
Vi+••• + !/’ = 1>
/ n \
(Xlt/2 - X21/1)2 < 2 I 1 - 52 XkVk j •
\ fc=l /
Korea, 2001
16 Problems

64. [ Laurencin Panaitopol ) Let ai, a2,... ,au be pairwise distinct positive inte­
gers. Prove that
2n 4- 1
o2 4- a2 4- • • • 4- o2 > (fii 4- aj + • • • + on).
3
TST Romania j
65. [ Calin Popa ] Let a,b, c be positive real numbers such that a 4- b + c = 1.
Prove that
by/c cy/a ay/b > 3/3
a(y/3c 4- y/ab') b(y/3a + y/bc) c(\/3b4- y/ca) - 4 ‘

66. [ Titu Andreescu, Gabriel Dospinescu ] Let a, b, c, d be real numbers such that
(1 4-a2)(l 4-b2)(l 4-c2)(l 4-d2) = 16. Prove that-
I
-3 < ab 4- be + cd + da + ac 4- bd — abed <5. 1

67. Prove that


. (a2 + 2)(b2 .4-2)(c2 + 2) > 9(fl5 + 6c + ca)

for any positive real numbers a, 6, c.


APMO, 2004

68. [ Vasile Cirtoaje ] Prove that if0<x<y<z and x 4- y 4- z = xyz 4- 2, then


a) (1<- iy)(l - j/z)(l - xz) > 0;
b) x2y < l,xV <

69. [ Ti in Andreescu ] Lcla.b.cbc positive real numbers such Lhata4-b+c> abc.


Prove that at least two of the inequalities
2 3 6 2 3 6 2 3 6 „
abc b c a cab
are true. _■
TST 2001, USA

70. ( Gabriel Dospinescu, Marian Tetiva ] Let x, y, z > 0 such that

x 4- y 4- z — xyz.

Prove that
(x~l)(p-!)(?- !)< 6^-10.
Old and New Inequalities 17

71. [ Marian Tetiva ] Prove that for any positive real numbers a,b, c,
n3 - b3 b3 - c3 c3 - rr3 (a - b)2 l (b - c)2 I (c - a)2
4----------- <
a 4- b + b 4- c c+ a
Moldova TST, 2004

72. [ Titu Andreescu ] Let a, btc be positive real numbers. Prove that

(a5 - a2 + 3)(b5 - b2 4- 3)(c5 - c2 •+ 3) > (a -I- b 4- c)3.

USAMO, 2004

73. [ Gabriel Dospinescu ] Let n > 2 and xi,X2>• • •>®n >. 0 such that
/ n n 1 \
Y"- =n2 + 1-
Xk /
Prove that
n 1 2
la
k-1
> rr 4- 4 4-
n(n - 1)

74. [ Gabriel Dospinescu, Mircea Lascu, Marian Tetiva ] Prove that for any po­
sitive real numbers a,b,c,

a2 4- b2 + c2 + 2abc + 3 > (1 + a)( 1 4- b)(l + c).

. 75- [•Titu Andreescu, Zuming Feng ] Let a,b, c be positive real numbers. Prove
that.
(2a 4- b 4- c)2 (2b 4- a 4- c)2 (2c 4- a 4- b)2
4- 4- <8.
2a2 4- (b 4- c)2 2b2 4- (a 4- c)2 2c2 4- (a 4- b)2
USAMO, 2003

76. Prove that for any positive real numbers x,y and any positive integers m, n,

(n-l)(rn-l)(xm+n +ym+n )+(m+n-l)(xmyn+xnym) > mn(xm+n-ly+y",n+n-1z).

Austrian-Polish Competition, 1995

77. Let a, b,c, d, e be positive real numbers such that abede■= 1. Prove that
•' a + abc b+bed c + cde . d-rdea . e + eab 10

1.4- ab 4- abed + 1 4- be 4- bede + 14- -J- ■ ■ ■ -f-' 1 * 1 > ------
T.
cd4-cdea 1.4- de 4- deab . 1 4- ea 4- eabc 3
Crux Mathematicorurri

I
18 Problems

78. [ Titu Andreescu ] Prove that for any o, b,c, e (o, the following inequality
holds
sin a ■ sin(a - b) • sin(a - c) sin b • sin(b - c) ■ sin(b - a) sin c • sin(c - a) • sin(c - b)
sin(bd-c)
+sin(c.d-a) +sin(a d-b) >0.

TST 2003, USA

79. Prove that if a, b, c are positive real numbers then,


i
KMO Summer Program Test, 2001

80. [ Gabriel Dospinescu, Mircea Lascu ] For a given n > 2 find the smallest
constant fcn with the property: if ai,..., an >0 have product 1, then
________ 0102__________ O2O3 , ._____________________ QnQi
(a? + a2)(c4 + Qi) (a|+ a3)(a| + a2) (a2 + ai)(a? + a„)

81. [ Vasile Cirtoaje ] For any real numbers a,b,c,x}y,z prove that the inequality
'.olds
2
ax d- by + cz + y/(a2 + b2 I- c2)^2 I y2 I- z2) > ~(a + b + c)(x y + z).
u
Kvant, 1989

82. [ Vasile Cirtoaje ] Prove that the sides a, b, c of a triangle satisfy the inequality
abc, „ „ (b c a
3 -+-+ --1 >2 -+t+-
b c a \a b c

83. [ Walther Janous ] Let n > 2 and let xi,x2,...,xn > 0 add up to 1. Prove
that

Crux Mathematicorum

84. [ Vasile Cirtoaje, Gheorghe Eckstein ] Consider positive real numbers


ij,i2>•••>in such that iiX2—xn = 1. Prove that • ■
1 ; 1 1
d----------- “------------- 1- <1.
n - 1 + xj n - 1 + x,,
TST 1999, Romania

85. [Titu Andreescu ] Prove that for any nonnegative real numbers a,b,c such
that a2 + b2 + c2 + abc = 4 we have 0 < ab d- be d- ca - abc < 2.
USAMO, 2001
Old and New Inequalities 10

86. [ Titu Andreescu ] Prove that for any positive real numbers a, b, c the following
inequality holds
a+b+c
— x/abc < mar{(y/o - y/b)2,(y/b- y/c)2,(y/e- y/a)2}.
3
TST 2000, USA

87. | Kiran Kcdlaya | Let a,b,c be positive real numbers. Prove that
a + x/ab + \/abc ,' a+b
3 “ V 3

88. Find the greatest constant k such that for any positive integer n which is not
a square, |(1 + y/n) sin(7r>/n)| > k.
Vietnamese IMO TYaining Camp, 1995

89. [ Dung Tran Nam ] Let x,y, z > 0 such that (z + y + z)3 = 32zyz. Find the
2/^ 1/^ |* 2^
minimum and maximum of
(z + y + z)4
Vietnam, 2004

90. [ George Tsintifas ] Prove that for any a,b, c, d > 0,

• • (n +-b)3(b + c)3(c + d)3(d + a)3 > 16a2b2c2d2(a + b + c + d)4.

Crux Mathematicorum

91. [ Titu Andreescu, Gabriel Dospinescu ] Find the maximum value of the ex­
pression
(ab)n | (fee)" [ (ca)»
1 — ab
+ 1 - be + 1 — ca
where a, b, c are nonnegative real numbers which add up to 1 and n is some positive
integer.

92. Let a,b,c be positive real numbers. Prove that


1
a(l + 6) +
+ b(l + c)
1.1 + —-—>___ 2___
1 + a) x/abc( 1 + v^abc)

93. [ Dung Tran Nam ] Prove that for any real numbers a, b, c such that a2 + b2 4-
c2 = 9,
2(a + b + c) - abc < 10.
Vietnam, 2002
20 Problems

94. [ Vasile Cirtoaje ] Let a,b,c be positive teal numbers. Prove that

«+!-. 6+1-1 b+ 1-i e+l-l «+l-i a+|-lj >3.


c

95. [ Gabriel Dospincscu ] Let n be an integer greater than 2. Find the greatest
real number mn and the least real number-Mn such that for any positive real numbers
Xi, X2,..., x„ (with Xji — xq, — X]),
n .
m < y-'__________ Xj________ ■■ < Mn.
nin “ “ z.-i + 2(n - l)xj -I- x<tl

96. [ Vasile Cirtoaje ] If x,y,z are positive real numbers, then


1 1 + 1 > 9
x2 + xy + y2
+ >
y2 + yz + z2 z2 + zx + z2 ~ (x + y + z)2'
Gazeta Matematica

97. [ Vasile Cirtoaje ] For any a,b,c,d > 0 prove that


2(o;' + 1)(6' + !)(? + IJIrf1 + !)>(! + M)(l + a’)(l + t>2)( 1 + c!)(l + d!).
Gazcta Matcmatica

98. Prove that for any real numbers a,b, c,

(a + b)4 + (b + c)4 + (c + a)4 > y(a4 + b4 + c4).

Vietnam TST, 1996

99. Prove that if a,b,c are positive real numbers such that abc = 1, then
• 1 ' 1
1 +a + b + + 1 + c1 + a ~ ' 2 +1 a + 2 +1 b + 2 +1 c
+ 1+b+c +
■ ' Bulgaria, 1997

2 3
100;. [ Dung Tran Nam ] Find the minimum value of the expression - + T + -
a b c
where a,b, c are positive real numbers such that 21ab + 2bc + 8ca < 12.
Vietnam, 2001

101. [ Titu Andrccscu, Gabriel Dospincscu ] Prove that for any x,y, z,a, b,c > 0
such that xy + yz + zx = 3,

T~—(y + -) + + *) + + y) 3-
b+ c c+a a+b
Old and New Inequalities 21

102. Let a,b, c be positive real numbers. Prove that


(b + c - a)2 (c 4- a - b)2
(b 4- c)2 4- a2 + (c 4- a)2 4- b2
(a 4- b — c)2
(a+ b)2 4- c2 >4
Japan,1997

103. [ Vasile Cirtoaje, Gabriel Dospinescu ] Prove that if aj,a2)... ,an > 0 then
n
fli + «2 I......... I ftn-i
a" + a" + •• • + an ” naia2 .. .a„ > (n - 1) ---------------
n - 1;-------------
where an is the least, among the numbers ai, a2,..., an.

104. [ Turkevici ) Prove that for all positive real numbers x,y,z.t,
x4 4- y4 4- z4 4- t4 4- 2xyzt > x2y2 4- y2z2 4- z2t2 4- t2x2 4- x2z2 + y2t2.
Kvant

. 10.5. Prove that for any real numbers , a2,..., an the following inequality holds

n 2

E"< s E ataj.

106. Prove that if ai, a2,... ,an, bi,... ,bn are real numbers between 1001 and
2002, inclusively, such that a2 4- a| 4- • • • 4- a2 = b2 4- b% 4- ■ • • + b2, then we have the
inequality . ••
a?.<
2U ■•4- — < — (a2 4- (A 4- ••• + a2^
bi b2 + bn - ur 1 + + n‘'
: - TST Singapore

107. [ Titu Andreescu, Gabriel Dospinescu ] Prove that if a,b,c are positive real
numbers which add up to 1, then
(a2 4- b2)(b2 4- c2)(c2 4- a2) > 8(a2b2 4- b2c2 4- c2a2)2.

108. [ Vasile Cirtoaje ] If a,b,c,d arc positive real numbers such that abed = 1,
then
1 , 1 1 1
(1-Fa)2 ' (14-b)2 1 (1 + c)2 I (1 4- d)2 > I-
Gazeta MatematicS

109. [ Vasile Cirtoaje ] Let a, b, c be positive real numbers. Prove that


a2 b2 cc?2 a b c
b2 4- c2 c2 4- a2 4- a + bti22 "b + c"*'c + a^a4-b'
a22 4-
Gazeta MatematicS
22 Problems

110. [ Gabriel Dospinescu ] Let ai,a2,... ,a„ be real numbers and let S be a
non-empty subset of {1,2,..., n). Prove that
2

- 52 (Qi + ••• + aj)2-


TST 2004, Romania

111. [ Dung Tran Nam ] Let zi ,z2..., z2ooi be real numbers in the interval [-1,1]
such that Xj -1- x% +... + zi^n = 0. Find the maximal value of the Zi +z24----- Fz2oo4.

112. | Gabriel Dospinescu, Calin Pupa ] Prove that if n > 2 and ai,a2,...,an
are real numbers with product 1, then
2/i z—
+ di + • • • + a2 - n >----- - • x/n - l(ai + a2 + • • • + an - n).
14 n-1

113. | Vasilc Cirtoajc ] If a, b, c arc positive'real numbers, then


r 2a
:------7 4" — < 3.
a I- b c+a
Gazeta Matematica

114.. Prove the following inequality for positive real numbers x,y,z
1 . 1 1 \ > 9
(xy + yz + zx) (z 4- y)2 + (y + z)2 + (z + z)2 J ~ 4
(1/ + *)2
Iran, 1996

115. Prove that for any x, y in the interval [0,1),


\/l + x2 + \/l 4- y2 + x/(l - z)2 + (1 - y)2 > (1 + \/5)(l - zy).

116. | Suranyi | Prove that for any positive real numbers ai,a2,...,an the fol­
lowing inequality holds '
(n-l)(a" I u" I ••• I «") i naiu2...an > (m | a2-|----- 1 an)(a"-1-l a-"-1 F-• -I u"-1).
Miklos Schweitzer Competition

117. Prove that for any Z],z2,...,zn > 0 with product 1,

l<i<J<n i-1
A generalization of Turkevici’s inequality
Old and New Inequalities 23

118. [ Gabriel Dospinescu ] Find the minimum value of the expression


n -----------------------
y' / Q ifl2 ••• On
" y 1 - (n - 1 )at

where 01,02,..., On < ——r add up to 1 and n > 2 is an integer.


n—1
119. [ Vasile Cirtoaje ] Let 01,02,... ,on < 1 be nonnegative real numbers such
that _____________
of + 02 + . 4- a2
a—
n
Prove that
_±J_ +
1-a2
02
1 - Oo r-+ On > na—.
l-*2n - 1 - Q2

120. [ Vasile Cirtoaje, Mircea Lascu J Let a.b,c,x,y, z be positive real numbers
such that
(a + b 4- c)(x 4- y + z) = (a2 + b2 4- c2)(x2 4- y2 4- z2) = -1.
Prove that
, 1
abexyz < —.
oO

121. [ Gabriel Dospinescu ] For a given n > 2, find the minimal value of the
constant kn, such that if Xi ,X2, • • •, xn > 0 have product 1, then
1 1 1 ■ ' . ■
■ 4-
4- ■ - 4+ ••■ +F
--------- ~ < n - 1.
: x/l 4- fcnXl \/l + kn%2 %/l 4-Arnxn . ..
. Mathlinks Contest

122. [ Vasile Cirtoaje, Gabriel Dospinescu ] For a given n > 2, find the maximal
value of the constant fcn such that for any Xi, x2,..., xn > 0 for which x2 4- x2 4-• • • 4-
x2 = 1 we have the inequality
(1 — X1)(1 -I2)...(l -x„) > knXxX2..:Xn. '

I
CHAPTER 2

Solutions

25
26 SullltiOIIH

1. Prove that the inequality


7a2 + (1 - b)2 + 7b2 + (1 - c)2 + 7c2 + (1 - a)2 > ^y

holds for arbitrary real numbers a, b, c.


Kbmal

First solution:
Applying Minkowsky’s Inequality to the left-hand side we have
\/a2 + (1 - b)2+ 7b2 + (1 - c)2+ x/c2 + (1 - a)2 > 7(a + b + c)2 + (3 - a - b - c)2.

Denoting a + b + c = x we get
3 2 9 9
(a + b + c)2 + (3 - a - b - c)2 = 2 X"2 + 2 “ 2’

and the conclusion follows.

Second solution'.
We have the inequalities
7a2 + (1 - b)2 + 7b2 + (1 - c)2 + 7c2 + (l-a)2 >
|a| + |I-b| |b| + |l-c| |c| + |l-a|
> ---------- —-------- -j. ---------- —------- + --------
72 72 72
372
and because |x| + |1 - x| > 1 for all real numbers x the last quantity is at least -y.

2. [ Dinu §crbanescu ] If a, b, c € (0,1) prove that


Tabc + 7(1 - a)(l - b)(l - c) <1. ■

• Junior TST 2002, Romania

First solution:
Observe that < x? for x e (0,1). Thus
■ y/abc < Vabc,

and
7(1 - «)(1 - - c) < </(] - a)(l - b)(l - c).
By the AM-GM Inequality,
a+b+c
x/abc < Tobe <
3
and
(1 - a) + (1 - b) + (1 - c)
7(1 - a)(l - b)(l - c) < 7(1 - a)(l - b)(l - c) <
3
Ol«l and New Inequalities 27

Summing up, we obtain


a + b + c+1—a+1—b+1—c
y/abc 4- \/( 1 — a)( 1 — b)(l - c) < = 1,
3
as desired.

Second solution:
We have
y/abc + ~ a)(l ~ b)(l — c) <,y/b- y/c +
by the Cauchy-Schwarz Inequality.

Third solution:
Let a = sin2 x, b — sin2 j/, c = sin2z, where x,y,z 6 The inequality
becomes ■ ■ ■ ■ . • • ■ ; ■. i

sin x ■ sin y • sin z + cos x • cos y • cos z < 1


and it follows from the inequalities
sin x • sin y • sin z + cos x ■ cos y • cos z < sin x • sin y + cos x ■ cos y — cos(x - y) < 1

3. [ Mircea Lascu ] Let a,b, c be positive real numbers such that abc = 1. Prove
that
b+c c+a a + b^ ,-r-
—1=—I---- 7=—F--- 7=- > y/a + v b + y/C + 3.
y/a y/b y/c
Gazeta MatematicS

Solution:
From the AM-GM Inequality, we have
c+a a+b
'b ' y/c

> 2(\/a + '/b + >/c) > x/a + Vb + y/c +'3x/abc = y/a + y/b + y/c + 3.

4. If the equation x4 + ax':3 + 2z2 + bx + 1 — 0 has at least one real root, then
a2 + b2 > 8.
Tournament of the Towns, 1993
28 Solution!)

Solution:
Let x be the real root of the equation. Using the Cauchy-Schwarz Inequality
we infer that

x2 4- xb
because the last inequality is equivalent to (x2 - I)4 > 0.

5. Find the maximum value of the expression x3 + y3 4- z3 - 3xyz where x2 4- y2 +


z2 - 1 and x, y, z are real numbers.

Solution:
Let t = xy 4- yz 4- zx. Let us observe that
(x3 4- y3 4- z3 — 3xyz)2 = (x 4- y 4- z)2(l - xy - yz - zx)2 = (14- 2i)(l - t)2.

We also have < t < 1. Thus, wc must find the maximum value of the expression

(1 4-2t)(l -1)2, where < I < 1. In this case we clearly have (1 4- 2t)(t - I)2 < 1 <=>
Z2(3-2t) > 0 and thus |x34-y3+z3-3xyz| < 1. We have equality for x = 1, y = z = 0
and thus the maximum value is'l. ‘

6. Let a, b, c,x, y, z be positive real numbers such that x + y + z = 1. Prove that


ax 4- by + cz + 2\/(xy + yz-hzx)(ai + bc-i-ca) < a 4- b 4- c.

Ukraine, 2001

First solution: ■ ,
We will use the Cauchy-Schwarz Inequality twice. First, we can write ax4-f>y 4-
cz < %/a2 4- 62 4- c2 • ^x2 4- y2 4- z2 and then we apply again the Cauchy-Schwarz
Inequality to obtain: . . .
ax + by + cz 4- 2\/(xy 4- yz 4- zx)(ab 4- be + ca) <

VSa2 • VS2:2 + V2Sa6 ‘ \/2E^


< VSi2 + 2S^ • VSa2 -(-^S06 = S a-
Second solution:
The inequality being homogeneous in a, b, c we can assume that a 4- b + c = 1. We
apply this time the AM-GM Inequality and we find that
ax+by+cz+2\/(xy 4- yz 4- zx)(ab 4- be 4- ca) < ax4-by4-cz4-xy4-yz4-zi4-ab4-bc4-ea.
Consequently,
1 -x2 - y2 - z2 l-a2-b2-^.,
xy + yz 4- zx + ab + be + ca = ---------- < i - ax - by - cz,
Old and New Inequalities 29

the last one being equivalent to (x — a)2 + (y - b)2 4- (z — c)2 > 0.

7. [ Darij Grinberg ] If a, b, c are three positive real numbers, then


a b c 9
(b-Fc)2 + (c + a)2 + (a 4- b)22 >
" 4 (a I b I c) ’

First solution-.
We rewrite the inequality as

(a + b -F c)
a
2
(b 4- c)2
+
b
(c + a)22
c
(a 4- b)2 4
Applying the Cauchy-Schwarz Inequality we get
2
a b a b c
(a 4- b + c) 4- > --------- 4- ■ 4-
(b 4- c)2 (c I- a)2 + (« F b)2 b+c c+a a 4- b
It remains to prove that
a b c 3
-------- F-------- F------ > -
b + c c + a a 4- b ~~ 2
which is well-known.

Second solution:
Without loss of generality we may assume that a 4- b 4- c = 1. Now, consider
X
the function f : (0,1) —» (0, oo), /(r) = —------ rx. A short computation of derivatives
(1 X)
shows that / is convex. Thus, we may apply Jensen’s Inequality and the conclusion
follows.

8. [ Hojoo Lee ] Let a, b,c > 0. Prove that


y/a4 + d2b'2 + b4 4- \/b4 4- b2c2 4- c4 + x/c4 4- c2a2 4- a4 >
> a \/2d2 + be. + by/2b'2 4- ca + c\/2r2 -F ab.
Gazeta Matematicd

Solution:
We start from (a2—b2)2 > 0. We rewrite it as 'la* 4--la2b24--lb4 > 3a4 ' tudlr F 3b4.
It follows that x/a'1 4- a2b2 1 b2 > ~/a2 I b2).
Using this observation, we find that.
2 2
(52 x/a4 4- d2b2 4 b*) > 3 •

But using the Cauchy-Schwarz Inequality we obtain


(^2a\/2a24-bc) < ($2 a2) (52 (2a2 4-be)) < 3 a2)

and the inequality is proved.


30 Solutions

9. If a,b,c are positive real numbers such that abc = 2, then


a3 + b3 + c3 > a\/b + c 4- i>v/c + a + cVa + b.
When does equality hold?
JBMO 2002 Shortlist

First solution:
Applying the Cauchy-Schwarz Inequality gives
3(a2 4- b2 4- c2) > 3(a + b 4- c)2 (1)
and
(n2 4- b2 4- c2)2 < (a 4- b 4- c)(a3 4- b3 4- c3). (2)
These two inequalities combined yield
(a2 4- b2 4- c2)(a 4- b 4- c)
o3rb:, + ? >
3
(n2 I b2 I e2)[(b I n) I (n I e) I (a I b)]
6“

(n\/b 1 c 4- b^a I c 4- c 'a I- b)2


> (3)
6
Using the AM-GM Inequality we obtain

aVb 4- c 4- b\/a 4- c 4- c a + b > 3\j abc (\/(a 4- b)(b 4- c)(c 4- n))

> abcVSabc = 3 = 6.
Thus
(a\/b 4- c 4- b\/a 4- c 4- c\/a + b)2 > 6(a\/b 4- c 4- b\/a 4- c 4- c a + b). (4)
The desired inequality follows from (3) and (4).

Second solution:
We have
aVb + c 4- b\/c 4- a 4- c 'a+ b < \/2(a2 4- b2 4- c2)(a 4- b 4- c).
Using Chebyshev Inequality, we infer that.
\/2(n2 4- b2 4- c2)(a 4- b 4- c) < \/6(a3 + b3 4- o’) •
and so it is enough to prove that <r* I b3 I c3 > 3nbr, which is true by the AM-GM
Inequality. We have equality if a = b = c = s/2.

10. [ Ioan Tomescu ] Let x,y,z > 0. Prove that


___________ ryz___________
(1 -I- 3x)(x + 8y)(y 4- 9a)(z -I- 6)
Old and New Inequalities 31

When do we have equality?


Gazcta MatematicS

Solution:
First, we write the inequality in the following form

(1 + 3i)(1 + t) (1 + v) (1 + !)
But this follows immediately from Huygens Inequality. We have equality for
3
x = 2,y = -,z= 1.

11. [ Mihai Piticari, Dan Popescu ] Prove that

5(a2 4- b2 4- c2) < 6(a3 4- b3 4- c3) + 1,

for all a, b, c. > 0 with a 4- b 4- a = 1.

Solution:
Because a -t- b + c = 1, we have a3 + b3 -t c3 = 3abc + a2 -f- b2 + c2 — ab — be — ca.
The inequality becomes

5(a2 4- b2 + c2) < 18abc 4- 6(a2 4- b2 4- c2) - 6(ab 4- be 4- ca) + 1 <=>


18abc 4- 1 - 2(ab 4- be 4- co) + 1 > 6(ab 4- be + ca) <=>
<=> 8(ab 4- be + ca) <2 4- 18abc <=> 4(ab + be + ca) < 1 + 9abc <=>
<=> (1 - 2a)(l - 26)(1 — 2c) < abc <=>
<=> (b 4- c - a)(c + a - b)(a + b - c) < abc,

which is equivalent to Schur’s Inequality.

12. [ Mircea Lascu ] Let ij, X2,... ,x„ G R, n > 2 and a > 0 such that xj 4-
a2 2d
X2 + ... + xn = a and x? 4- x2 4- ... 4- x2 < ----- -. Prove that x, G 0, — , for all
n— 1 n
t G {l,2,...,n}.

Solution:
Using the Cauchy-Schwarz Inequality, we get

Thus,
2a
a2 - 2aX} 4- x2 < a2 - (n - l)xf <=> Xi xi------- < 0
n
and the conclusion follows.
32 Solutions

13. [ Adrian Zahariuc ] Prove that for any a,b, c € (1,2) the following inequality
holds
b\/a cvb a~Jc
+ ----------------------1----------- 1-------- > I-
4b\/c - C\/a 4cy/a - ay/b 4a\/b-b\/c

Solution:
The fact that a,b, r.e (1,2) makes all denominators positive. Then
b^/a
> b(a + b + c) > \/a(4bx/c - c>/a) <=>
46\/c - Cy/ii
<=> (a + b)(b + c) > 4l>\/ac,

the last one coming from ti + b > 2y/ab and b + c > 2\/bc. Writing the other two
inequalities and adding them up give the desired result.

14. For positive real numbers a, b,csuch that abc < 1, prove that
abc
— + —I— > a + b + c.
b c a

First solution:
If ab + be + ca < a+ b + c, then the Cauchy-Schwarz Inequality solves the
problem:
(a + b +c)2
t
—F 7 + —
i r
_abc a2c + b2a +—c2-b
q-----1--- _ ---------- > a b e > a 4- b + c.
b c a abc abc
Otherwise, the same inequality gives
(ab + be + ca)2
abc a2c + b2a + c2b a i b -|- c
r + - + — — --------- ;--------- > > a -f- b + c
b c a abc abc
(here we have used the fact, that abc. < 1).

Second solution:
Replacing a. b, c by La, lb, Ic with I = - preserves the value of the quantity in
V abc
the left-hand side of the inequality and increases the value of the right-hand side and
makes at ■ bl • ct = abctA = 1. Hence we may assume without loss of generality that
y z x~
abc = 1. Then there exist, positive real numbers x, y,z such that a = -,b = - ,c = -.
z
The Rearrangement Inequality gives

x3 + y3 + z3 > x2y + y2z + z2x.


Old and New Inequalities 33

Thus
abc x:* 4- y3 4- z-’ x2y 4- y2z + z2x
7 -f- —I--- —
b c a xyz
> xyz
as desired.

Third solution:
Using the AM-GM Inequality, we deduce that
2a 6
-r + - > 3
6 c
2& c 2c g
Similarly, — + - > 36 and — 4-- > 3c. Adding these three inequalities, the conclusion
is immediate.

Forth solution:
, Jab* J ca* 9/be* ~ , 9,9 9
Let x = y ~^T'V = y ~^~,z ~ \ ^orLsecluen*-*y> a = XV — zx ,c = yz*,
and also xyz < 1. Thus, using the Rearrangement Inequality, we find that

Ej-E^-E^-E^E^'2 = £>•
15. [ Vasile Cirtoaje, Mircea Lascu ] Let. a,b,c,x,y,z be positive real numbers
such that a + x > b 4- y > c I z and a 4- b -I- c = x 4- y 4- z. Prove that ay 4- bx > ac 4 xz.

Solution:
We have
ay l bx-ac-xz a(y-c)-\-x(b-z) - a(a I b-x-z)4 x(b-z) - a(a-x) I (a \ x)(b-z)

= 5(0-i)2 4- - (a2 - x2) 4- (a 4- x)(b - z) — i(a - x)2 4- |(a 4- x)(a - x 4- 26 - 2z) =


= |(u ~ *)2 + |(« + x)(b- c + y - z) > 0.

The equality occurs when a = x, b = z, c = y and 2x > y + z.

16. [ Vasile Cirtoaje, Mircea Lascu ] Let a, b, c be positive real numbers so that
abc = 1. Prove that
3 6
14- >
a 4- b 4- c ab 4- ac 4- be'
Junior TST 2003, Romania

Solution:
1 i z = - and observe that xyz = 1. The inequality is
We set x =
a'V = 6 c
equivalent, to
1+—’— > 6
xy 4- yz + zx x 4- y 4- z
34 Solutions

From (x + y + z)2 > 3(xy + yz + zx) we get


1+^— 9
xy + yz + zx
> 1 + (x + y + z)2’
so it. suffices to prove ( hat.
9 6
1+ > x + y t z'
(s t y i z)2
2
1--------3— > 0 and this ends the.proof. • .
The last inequality is equivalent, to
x+y+z

17. Let o,b, c be positive real numbers. Prove that


a3 b3 c3 a2 b2 c2
T7 + ~5 + -2>-7- + — d---- .
b2 cz a2 b c a
JBMO 2002 Shortlist

First solution:
We have
a3 a2
-j->
— Ta-b<^a3 + b63> ob(o
> — Ta-b<=>a ab(a + b) <=>
« (a - b)2(a + b) > 0,
b2 b
which is clearly true. Writing the analogous inequalities and adding them up gives
a3 b3 c3 a2 , b2 L c2 a2 b2 c2
b2 c2 a2 b c a b c a

Second solution:
By the Cauchy-Schwarz Inequality we have
, . (a ' a2 b2 c2\2
(a+t+c)^ —+_+c—3
> ~r +---- 1---- )
O' b c aJ
so we only have to prove that
a2 bb22 c2
— + — + — >a + b+c.
b c a
But this follows immediately from the Cauchy-Schwarz Inequality.

18. Prove that if n > 3 and Xj,.r2,...,.rn > 0 have product 1, then
1 1 1
........... 'I ---------------------- -j-I........
• • • -j-I ---------------------- > 1.
1 4- X| + .T|.X’2 1 + X2 X2X3
X2 + 2:2^3 1 + Xn + X„X\
Russia, 2004

Solution:
We use a similar- form of the classical substitution , x2 = xn = —
Qi 0.2 o.n
In this case the inequality becomes
flj O2 Qn.
----------------------- ------ ------------------------- F ■ • ’ -f > 1
fli + a2 + 03 o2 + 03 + a.|----------- an + Oi + a2
Old and New Inequalities 35

and it is clear, because n > 3 and a, + Oj+i + cq+2 < ai + aj + • ■ • + u„ for all i.

19. [ Marian Tetiva ] Let x,y, z be positive real numbers satisfying the condition

x2 + y2 + z2 + 2xyz = 1.

Prove that
, 1
a) xyz <
3
b) x + y + z <
x 3 2
c) xy + xz + yz < - < x + i 2 + z\
d) xy + xz + yz < | + 2xyz.

Solution-.
a) This is very simple. From the AM-GM Inequality, we have

1 = x2 + y2 + z2 + 2xyz > 4y/2x3y3z3 => x3y3z3 < —~-4- =$>. xyz < -.

b) Clearly, we must have x, y, z 6 (0,1). If we put s — x+y+z, we get immediately


from the given relation

s2 - 2s + 1 = 2 (1 - x) (1 - y) (1 - z).

Then, again by the AM-GM Inequality (1 - x, 1 - y, 1 - z being positive), we


obtain
3
1 — x+1—y+l-z 3-s
s2 - 2s -I 1 < 2 =2
3
Alter some easy calculations this yields
2s3 + 9? - 27 < 0 (2s - 3) (s I 3)2 < 0

and the conclusion is plain.


c) These inequalities are simple consequences of a) and b):
(x + y 4- z) ’ < 1 9 3
xy + xz + yz < “ 3 ’ 4
3 4
and
x2 + y2 + z2 = 1 - 2xyz > 1 - 2 • |
o 4
d) This is more delicate; we first notice that there are always two of the three
numbers, both greater (or both less) than -. Because of symmetry, we may assume
that x, y < | , or x, y > | and then

(2x - 1) (2y - 1) > 0 o x + y - 2xy < i.


3(» Solutions

On the other hand,


1 = x2 + y2 + z2 + 2xyz > 2xy + z2 + 2xyz =>
-> 2xy (I I z) < I - z2 2xy < 1 - z.
Now, we only have to multiply side by side the inequalities from above
x + y - 2xy < |

and
z < 1 — 2xy
to get the desired result:

xz + yz- 2xyz < - xy xy + xz + yz < + 2xyz.

It is not important in the proof, but also notice that


a /I 1 „
x + y - 2xy = xy I —I----- 2 >0,
\x y J
because - and - are both greater than 1.
x y

Remark.
1) One can obtain some other inequalities, using

z + 2xy < 1
and the two likes
y + 2xz < 1, x + 2yz < 1.
For example, multiplying these inequalities by z, y,x respectively and adding the new
inequalities, we get
•r2 I ?/2 I- z2 l Gr.yz < x H y I z,
or
1 I \xyz < x ]■ y -I- z.
2) If ABC is any triangle, the numbers
.A . B . C
I = sm—, y = sin—, z = sin -

satisfy the condition of this problem; conversely, if x,y,z > 0 verify


x2 + y2 + z2 + 2xyz - 1

then there is a triangle ABC so that


. A . B . C
z = sm-, y = sin —, z = sin-.

According to this, new proofs can be given for such inequalities.


Old and New Inequalities 37

20. [ Marius Olteanu ] Let xi,X2,X3,X4,xr, € R so that xi 4- x3 4- x3 + x4 + xr, = 0.


Prove that
| COSZ1I + | COSX2| + I COSX;t| + | cos x41 4- | cosx5| > 1.
Gazcta Matcniaticft

Solution'.
It is immediate to prove that
|sin(i4- y)| < min{|cosx| + |cosy[,|sinx| 4 |sin i/|}
and
|cos(x 4- y)| < min (| sinx| + | cosy|, | sin y| 4- jcosx|}.
Thus, we infer that
/ 5
( •r'
1 = 1 cos I x, | < |cosxii+|sin | < !<?osri| + |cosr-2|+'cos(x:)+r4 + r-,)|.
\i-2

But in the same manner we can prove that


|cos(x3 + X4 + x5)| < |cosx3| + |cosx4| 4- jcosisl
and the conclusion follows.

21. [ Florina Carlan, Marian Tetiva ] Prove that if x, y, z > 0 satisfy the condition
x + y -I- z = xyz
then xy + xz + yz > 3 -r x/x2 4- 1 + \/y2 + 1 + v^z2 4- 1.

First solution:
We have
xyz = x + y 4- z > ‘2^/xy 4- z => z{^/xy)2 - "2y/xy - z > 0.

Because the positive root of the trinomial zt2 — 2t — z is


1 -I- x/1 -I- z2

we get from here


x/xy > —^-+~- » Zy/xy > 1 4- \/l 4- z2.

Of course, we have two other similar inequalities. Then,


xy l xz l yz > Xy/yz i yx/xz I z^/xy >
> 3 4- \/x2 4- 1 + \/y2 4- 1 4- \/z2 4- 1,

and we. have both a proof of I he given inequality, and a little improvement of it.
3K Solutions

Second solution:
Another improvement is as follows. Start from
l + l + l>-l-4-- + - = l=> x2y2 + x2z2 4- y2z2 > x2y2z2,
x2 y2 z2 xy xz yz
which is equivalent to

(xy -I- xz 4- yz)2 > 2xyz (x -I- y 4- z) I- x2y2z2 = 3 (x 4- y I- z)2.

Further on,

(xy 4- xz 4- yz - 3)2 = (xy + xz + yz)2 - 6 (xy 4- xz 4- yz) + 9 >


> 3 (x + y + z)2 - 6 (xy 4- xz + yz) 4- 9 = 3 (x2 4- y2 4- z2) 4- 9,

so that
xy + xz + yz > 3 + \/3(x2 4- y2 4- z2) 4- 9.
But

is a consequence of the Cauchy-Schwarz Inequality and we have a second improve­


ment and proof for the desired inequality:

xy 4- xz 4- yz > 3 4- \/3(x2 4- y2 4- 22) 4- 9 >


> 3 4- \/x2 4- 1 4- \/y2 4- 1 4- \/z2 4- 1.

22. [ Laurencin Panaitopol ] Prove that


1 4- x2 1 4- y2
1 4- y + z2
4-
1 4- z 4- x2
+ 1 4- x 4- y2
>2,

for any real numbers x, y, z > -1.


JBMO, 2003

Solution:
. 1 +y,2:
Let us observe that y < - and 1 4- y 4- z2 > 0, so
2
1 -I- x2 1 -1- x^___
1 4- y 4- z2 1 4- y,2:
2
and the similar relations. Setting a = l4-x2ii=14-y2, c = 1 4- z2, it is sufficient to
prove that
a b c
+ +
2c 4- b 2a + c 2b 4- a > 1
(1)
Old and New Inequalities 39

C + 4B- 2A
for any a,b,c> 0. Let A = 2c 4- b, B = 2a 4- c, C = 2b 4- a. Then a =
9
A + 4C-2B B + 4A-2C
b- 9 ’ C~ and the inequality (1) is rewritten as
9
C A B B C A
A ‘ B + 7= I 4 A 1 B ' C > 15.
C
Because A, B,C > 0, wo have from the AM-CM Inequality that
CAB, B
A + B + C " 3 V7-
B C
jB C A
and---- F — 4- — > 3 and the conclusion follows.
ABC
An alternative solution for (1) is by using the Cauchy-Schwarz Inequality:
a b c a2 b2 2
c.............. (a + b + c)2
4- -------------- +
-j- 4- 4-
— ————— “I"------------------- ---
> 3(ab I be I ca) > I-
^-> ~~ ■ ' - '
2c I b 2u I c 2b I au 2ac I ab 2ub I cb 2bc I ac

23. Let a, 6, c > 0 with a 4- b 4- c = 1. Show that


2 4- b f.--------
-a--------- b2 + c c2 4- a
> 2.
o 4- c c 4- a a+b

Solution:
Using the Cauchy-Schwarz Inequality, we find that

(EM
E ab2 4+cb > 52 a2(b+c) + 52 “2 + 52ab
And so it is enough to prove that

52q2(^ + c) 4- 52 a2 4- ab >2 14-(52q2) - 252q2(6+c)+252ab-


The last inequality can be transformed as follows
2

1 + (52q2) - 2 52 a2(6+c) 4" 2 52ab ~ 1 + (52a2) >


> 25La2-‘252a:' + 25Lftb ~ (Efl2)2 + 252a:‘^S°2’
and it is true, because

E-^ E£
3
(Chebyshev’s Inequality)

and

(E°’)2 E°* 3
40 Solutions

24. Let «,b,c > 0 such that a4 + b4 4- c4 < 2(a2b2 + b2c2 + c2a2). Prove that

a2 + b2 + c2 < 2(ab + be + ca).

Kvant, 1988

Solution:
The condition
£a4<2£a2b2
is equivalent to

(a + b + c)(a + b - c)(b + c - a)(c + a - b) > 0.

In any of the cases a = b + c, b - c + a, c = a 4- b, the inequality

Ea2-2Za6
is clear. So, suppose a + b / c, b + c / a, c + a / b. Because at most one of the
numbers b + c-a, c + a-b, a + b~ c is negative and their product is nonnegative,
all of them are positive. Thus, we may assume that

a2 < ab + ac, b2 < be + ba, c2 < ca + cb

And the conclusion follows.

25. Let n > 2 and x>,.. xn be positive real numbers satisfying


__ 1 1
Xi +. 1998
+. "■ ++. xn +1 1998 '
i2 + 1998 +
1
1998’
Prove that
tfx}x2...xn
> 1998.
n-1
Vietnam, 1998

Solution:
1998
Let = ai- The problem reduces to proving that for any positive real
1998 + X{
numbers ai,fl2,... ,an such that ai + a2 + • • • + a„ = 1 we have the inequality

This inequality can be obtained by multiplying the inequalities


1 _| _ fli 4------ h Qi-1 + a,4-i + •' • + fln
a< at >
01 • ■ ■ 0,-lQt+l ■•■On
> (n-1)
a”"1
Old <uid New Inequalities 41

26. [ Marian Tetiva ] Consider positive real numbers x,y, z so that

x2 + y2 + z2 = xyz.

Prove the following inequalities


a) xyz > 27;
b) xy + xz + yz > 27;
c) x + y + z > 9;
d) xy + xz + yz > 2 (z + y + z) + 9.

Solution:
a), b), c) Using well-known inequalities, we have
xyz = x2 4- y2 4- z2 > 3\/x2y2z2 => (xyz)' > 27 (xyz)2 ,

which yields xyz > 27. Then

xy I xz -I yz > 3\](xyz)2 > 3s/272 27

and
x + y + z > 3 itfxyz > 3 \X27 = 9.
d) We notice that z2 < xyz => x <yz and the likes. Consequently,

xy < yz • xz => 1 < z2 => z > 1.

Hence all the three numbers must be greater than 1. Set

a = i- l,b = j/-l, c-z-1.

We then have a>0,b>0, c>0 and

x = a + 1, y = b+1, z — c + 1.

Replacing these in the given condition we get

• • • (a + I)2 4-(6 + I)2 4-(c 4-I)2 = (a + 1) (b 4-1) (c-t-1)

which is the same as

a2 + b2 + c2 + a + b + <: + 2 = ubc + ab + «c + lx-.

If we put q = ab + ac + be, we have

q < a2 I b2 |- c2, \/3q < a I b I c

and
42 Solutions

Thus
q 4- >/3q + 2 < a2 + b2 + c2 + a + b + c+ 2 =

= abc 4- ab + ac4- be <


(y^)3
27 +Q
and setting x - y/3q, we have
x3 9
X 4- 2 < — (x - 6) (x 4- 3) > 0.
Finally,
y/3q = x > 6 => q = ab 4- ac 4- be > 12.
Now, recall that a — x - 1, b = y - 1, c = z - 1, so wc get
(x-l)(y-l) + (x- l)(z- 1) 4- (y — 1) (z - 1) > 12 =>
=> xy + xz + yz > 2 (x 4- y 4- z) + 9;
and we arc done.
One can also prove the stronger inequality
xy + xz + yz > 4 (x + y 4- z) - 9.
Try it!

27. Let x,y,z be positive real numbers with sum 3. Prove that
\/x + y/y + \/z > xy + yz + zx.
Russia, 2002

Solution-.
Rewrite the inequality in the form

<=> x2 + 2y/x 4- y2 4- 2^/y 4- z2 4- 2yfz > 9.


Now, from the AM-GM Inequality, we have

y2 4- 2y/y > 3y, z2 4- 2y/z > 3z,


hence

28. [ D. Olteanu ] Let a, b,c be positive real numbers. Prove that


a+b a
b 4- c 2a 4- b 4- c
b+ c b
c + a 2b 4- c 4- a
c4-a. a
a + b 2c4-a4-b 4
Gazeta Matematica
Old and New Inequalities 43

Solution: /
We set x = a + b, y = b + c and z = a 4- c and after a few computations we obtain
the equivalent form
xyz y x
z 1----------- y z
y
- 4-I-----
- 4- —
z x x+
- 4-
4- y
1----------
--------- -b —
y + z
1---------
z+x >-l
But using the Cauchy-Schwarz Inequality,
x.y.z. x .1. y z (x4-
(x + yy4-
+ z) z)22 [ (x + y + z)2
4— + ------
y z x~ x + y y + z z+x xy
xy 4-4- yz zx xy + yz 4- zx 4- x2 4- y2 4- z2 ■:
yz 4-4- zx
2(x 4- y 4- z)4 - > 8(x 4- y 4- z)4
2(xy 4- yz 4- zx)(xy 4- yz 4- zx 4- x2 4- y2 4- z2) “ (xy + yz + zx + (x + y + zy)2
and the conclusion follows.

29. For any positive real numbers a,b,c show that the following inequality holds
abc-c4-aa4-bb + c
v + - + ->----- r +------ + t------ •
b c a. c + ba + cb + a •;
India, 2002

Solution:
be,.,
Let us take - — x, - = y, - = z. Observe that
b c a
a 4- c 1 + xy 1 -x
b 4- c 1+y 1 4- y'
Using similar relations, the problem reduces to proving that if xyz = 1, then
x-1 y-1 z- 1 „
------ + -- ----- + - ------ > 0 «■
y+1 z+l x+1 “
O (I2-l)(2+iy+(y2-l)(l + l) + (z2_l)(y+I)>0 <=>
<=> y x2z+y x2 > y x+3.
But this inequality is very easy. .Indeed, using the AM-GM Inequality wc have
^x2: > 3 and so it remains to prove that y x2 > which follows from the
inequalities
2

£x2> 'U» 3 -

30. Let a,b, c be positive real numbers. Prove that


a3 . b* ■ '• .c3 ' .
3(ab 4- be 4- ca)
b2 — be + c2
4-
c2 — ac + a2
+"r a2 — ab +4- bb22
>
~ a + b-j-c
Proposed for the Balkan Mathematical Olympiad
44 Solutions

First solution'.
3(ab F be + ca)
Since a -F b + c > , it suffices to prove that:
a -F b -I- c
a3 ' b3 c3
b2 — be + c2 + c2 — ca + a2 a2 — ab + b2 > a + b + c.
From the Cauchy-Schwartz Inequality, we get
a . a3

b2 - be+ c2
(r-2)
> 53 a(b2 - be + c2)
2

Thus we have to show that


(a2 + b2 + c2)2 > (a + 6 + c) • ^2 a(^2 “ + c2).

This inequality is equivalent to


a4 + b* -F c* + abc(a + b+c) > ab(a2 + b2) + bc(b2 + c2) + ca(c2 + a2),

which is just. Schur’s Inequality.


Remark.
The inequality
a3 b3 c3
+ c2 - ca + a2
b2 - be + c2 + + a2 - ab -F b2 > a + b + c
was proposed by Vasile Cartoaje in Gazeta Matematica as a special case (n = 3)
of the more general inequality
2an -bn-cn 2bn - cn - an 2cn - an - bn
>0.
b2 - be+ C2 c2 - ca + a2 + a2 - ab + b2

Second solution:
Rewrite our inequality as
(b + c)a3 3(ab + be + ca)
: ■ b3 + c3 > a+b+c ■ ‘
But this follows from a more general result:
If a,b,c,x,y,z > 0 then

-3 V •
b-Fc

But this inequality is an immediate (and weaker) consequence of the result from
problem 101.

Third solution:
Let
a'1
b2 - be + c2
Old and New Inequalities 45

and
b* -I- c3
fa2 - be + c2 -E (fa -F bic)(fa 2 - be F c2)
2 - be + c2
2^a.
So we get ■
(ZL a'3) (12 fa2 - bc + c2 ) =
A+B

1 2
= 5 (y (6+c)^2 ~bc+c’)) (y
b2 — be + c2 )=4(E b+c

from the Cauchy-Schwarz Inequality. Hence


■4>i(£V6Tt)2-2£ a = \/a + fa • Vb + c - 5^ a.

We denote
Ysab
Aa = c + a • 'fa + a — a =
a
and the similar relations with Ab and Ac. So A > Aa+Ab 4- Ac. But because a)

3 (52 ab) we get


/
Aa —
121,11 3y«fa
\ + a2 — a
• • (E“)2 3
+ a2 + a } /
3
and also the similar inequalities arc true. So we only need to prove that

\ £>r
/ ------ \
1 a
E 3
-F a2 - a > a F b + c <=> ^2 3f
\ /

We consider the convex function /(t) = 1/^ + t2. Using Jensen’s Inequality
V o
we finally deduce that ■
2
• a
y v-+
V 3a + fa + c
>2. .

We have equality if and only if a = fa = c. -

31. [ Adrian Zahariuc ] Consider the pairwise distinct integers zi,X2,...,in,


n > 0. Prove that . .
x? + x| + • • • + x2 > Xii2 + X2X3 + • • • + xnii + 2n - 3.
46 Solutions

Solution'. ■
The inequality can be rewritten as
n
-Xi+1)2 > 2(2n —3).

Let zm = min{zi,z21...,zn} and ta/ = max{zi,z2 zn}. Suppose without loss


of generality that m < M. Let
Sj — (zm — Zni+l)" + • • • + (za/ — 1 — ZA/)

and
S2 = -ZAf + l)2 + ••• + (z„ - Z1)2 + (Z1 -Z2)2 + •■• + (z,n-l ~ ®m)2-
k i / k 2

The inequality > y ( 5~^a« (which follows from the Cauchy-Schwarz In-
11 "\l I
equality) implies that
(zAf - zm)2
Si >
M-m
and
(x.w - zm)2
S2 >
n - (iW - m)'
So
1 1
52(t< -Ti+i)2 = Si + S2 > (Xm - Z,n)2 --------- —— -J- ■
M-m n- (M - m)
>

> (n—I)2-= 4n -8 + - > 4n - 8.


nn
But
n n
- Zi+1)2 = 52 z, - Zi+1 = 0 mod 2
t=i i=i
so
n
^2(ii -zi+1)2 > 4n-6

and the problem is solved.

32. [ Murray Klamkin ] Find the maximum value of the expression z2z2 + x^x-j +
• • • + Zn-ixn + t2zi w*’cn xi>x2, • • • ,xn-i,xn > 0 add up to 1 and n > 2.
Crux Mathcmaticorum

Solution:
4
First of all, it is clear that the maximum is at least —, because this value is
27
1 2
attained for xj = -,z2 = y,x3 = ••• = zn = 0. Now, we will prove by induction that
O kJ

Z,Z2 + Z2Z:j + • • • + Tn-iZn + X2 Zj <


Old and New Inequalities 47

for all xi,x2,... ,x„_i,xn > 0 which add up to 1. Let us prove first the inductive
step. Suppose the inequality is true for n and we will prove it for n + 1. We can of
course assume that x2 = min{ii,x2,... , xn+i}. But this implies that
X?X2 F XjX3 -»-•■• + X2 Xi < (Xi + X2)2X3 + X3X4 + ' • ■ F X2 .jX,, + x2 (xj + x2). ..

But from the inductive hypothesis we have


• .4
(Xi +x2)2x3 + X3I4 + ••• + X2_1In + X2(li +x2) < —

4
and the inductive step is proved. Thus, it remains to prove that a2b + b2c + <?a < —
27
if a + b + c = 1. We may of course, assume that a is the greatest among a, b, c. In this
case the inequality a2b + b2c + <?a < a+ ■ (b + follows immediately from
GC^
abc > b2c, -y > . Because

c c 2
a+ 2 1+ 2 cC 3 -51
1 =
V+ 2
4 •
we have proved that a2b+ b2c + c2a < — and this shows that the maximum is indeed
4 ....
27'

33. Find the maximum value of the constant c such that for any
Xi, x2,..., xn, ■ ■ • > 0 for which Xfc+i > ii -F x2 + • • • + x* for any k, the inequality

\/x7 + \/X2 + • • • + y/xTi < Cy/Xi + X2 + • • • + Xn

also holds for any n.


IMO Shortlist, 1986

Solution:
First, let us see what happens if x*+i and Xi +x24------ Fx^ are close for any k. For
example, we can take it = 2* .because in this case we have xi-Fx2+-••-t-xj! = Xfc+i-2.
Thus, we find that

c>

for any n. Taking the limit, we find that c > 1 + \/2. Now, let us prove that 1 -f- \/2
works. We will prove the inequality

\/Xl + y/xi + • • • + y/x^. < (1 + x/2)x/li + X2 + • • • + Xn


48 Solutions
I

by induction. For n = 1 or n = 2 it is clear. Suppose it is true for n and we will


prove that x/rT + y/l? + • • • + + V^n+T <(14- \/2) vAXj + 12 + • • • + Zn + Xn+L
Of course, it is enough to prove that
A+i - U 1 (\/xi ! x2 4 • I- Zu+1 - v/xi I *2 I....... I Xn)
which is equivalent to
'■ Vxi + X2 + ■ • ■ + xn + \/Xi + 12 + • •• + Xn-1 < (1 + \/2)yin-rl.
J

But this one follows because


i! I x2 I ••• I xn < xn+i.

34. Given are positive real numbers a, b, c and x, y, z, for which a+x=b+y=
c + z = 1. Prove that
1 i
(abc 4- + r—b -) >3.
bz ex)
Russia, 2002

Solution:
Let us observe that abc 4- xyz = (1 - b)(l - c) 4- ac 4- ab - a. Thus,
1-c c abc 4- xyz
a + 1-b a(l - b)
Using these identities we deduce immediately that
1 1
__ 4-____ |-
1 \I —---------- bl ; c
a 1-------- 1-c 1-a 1-b
3 4- (xyz 4- abc)
ay bz ex) “ T^c
1 - c +' 11 -
- aa +' T^b
1 + ------
abc
4----- ----- 1--------.
b
Now, all we have to do is apply the AM-GM Inequality
a b c 1-c 1-a 1 - b „
------ 4- ------- 4- ------r 4---------1---- r---- 1------- > 6.
1-c 1-a 1-b a b c

35. [ Viorel Vajaitu, Alexandru Zaharcscu ] Let a, b,c be positive real numbers.
Prove that
ab be ca 1
+ 4- < -(a + b + c).
a 4- b 4- 2c b4- c4- 2a c 4- a 4- 2b 4
Gazeta Matematica

First solution".
We have the following chain of inequalities
ab y—« ab ub f 1 1 a 4- b 4- c
£ a 4- b 4- 2c = E a + c + b + c ~ ---------F i-----
4 \ a 4- c b 4- c 4
Old and New Inequalities 40

Second solution:
Because the inequality is homogeneous we can consider without loss of generality
that a + b 4- c = 1 and so the inequality is equivalent to
v \ 1 1
2—* a(a 4- 1) ~ 4abc'

We have
1 2 — •"+ p so the inequality is equivalent to
t(t4-1) t

a ” a 4- 1 -4abc‘
We will prove now the following intercalation:

a ~ 4 4a.be T a 4- 1 4abc'
The inequality in the right follows from the Cauchy-Schwarz inequality:

and the identity 52(a + 1) = 4- The inequality in the left can be written as 52 —
1 + 9abc
---- ----- , which is exactly Schur’s Inequality.

36. Find the maximum value of the expression


a3(b 4- c + d) + b3(c 4- d + a) 4- c3(d 4- a + b) 4- d3(a 4- b + c)
where a,b, c, d are real numbers whose sum of squares is 1.

Solution'.
The idea is to observe that fl'’’(i>rC+d)+63(c+rf+a) + c3(rf+a+6) + d,t((i+6+c) is
equal to ab(a2 4- b2). Now, because the expression ab(a2 4- b2) appears when writing
(a — b)4, let us see how (.he initial expression can be written:

£ + O’) = + + _

3 52 «■* + ° 52 “42&2
———
- 52(a ~6)4 3- I4 < 3
“ 4‘
4
The maximum is attained for a = b = c = d =

37. [ Walther Janous ] Let x,y,z be positive real numbers..Prove that


.. —y ,— y i— _ <i.
+ x/(y 4-
X + y/(x 4- y)(z 4- z). yy + + z)(y x) + z2 4? \/(z 4- x)(z 4- y)
+ x)
-)(y 4-
Crux Mathematicorum
50 Solutions

First solution-.
We have (x + y)(x + z) = xy + (x2 + yz) + xz > xy + 2xx/iiz + iz = (x/xy + y/xz)2.
Hence
X
X + y/(x + J/)(l -I- z) - X + y/xy + y/xz
But
= 1
x +/xy + y/xz y/x +/y +/z
and this solves the problem.

Second solution:
FYom Huygens Inequality we have ^(x + y)(x + z) > x + y/yz and using this
inequality for the similar ones we get
x_______ _
+ _____ y-----------+ — •
x+ y(x + y)(x + z) y + \/(l/+ z)(l/+ *) z + V(z + x)(2 + !/)
x + y , 2 -
< 2x + y/yz 2j/ + y/zx 2z + y/xy

Now, we denote with a = [, — c= allj the inequality becomes


x y z
1 1 1 <
2+a+2+b+2+c“
FYom the above, notations we can see that abc = 1, so the last inequality becomes
after clearing the denominators ab + be + ca > 3, which follows from the AM-GM
Inequality.

38. Suppose that qj < a2 < ... < are real numbers for some integer n > 2.
Prove that
0102 + a2aj + ... + ana4 > a2a4 + a^a? + ... + oia3
Iran, 1999

Solution:
A quick look shows that as soon as we prove the inequality for n — 3, it will be
proved by induction for larger n. Thus, we must prove that for any a < b < c we have
ab(b3 - a3) + bc(c'‘ - b') > ca(c1 - a3) <=> (c3 - b3)(ac - be) < (b3 - a3)(ab - ac).
Because a < 6 < c, the last inequality reduces to a(b2 + ab + a2) < c(c2 + be + b2).
And this last inequality is equivalent to (c- a)(a2 + b2 + c2 + abT bc + ca) > 0, which
is clear.

39. [ Mircea Lascu ] Let a, b, c be positive real numbers. Prove that


b+ c c+a a+ b a b a \
------- + -T- +------- -4
a b c bL------ a + bL JI •
c + a + -----
+ c h ------
Old and New Inequalities 51

Solution:
1 1
Using the inequality
x+y
<p 4z +
we infer chat

4n a + a 4b . b . b , 4c ' c. c.
------ < - + - and ----- - < - + 7-
b+ c ~ b c' a + c ” a ' c “““ a + b a b
Adding up these three inequalities, the conclusion follows.

40. Let ai,a2,... ,an > 1 be positive integers. Prove that at least one of the
numbers a^/a2, •••> is less than or equal to s/3.
Adapted after a well-known problem

Solution:
Suppose we have > 3s for all i. First, we will prove that n* < 3s for all
natural number n. For n = 1,2,3,4 it is clear. Suppose the inequality is true for n > 3
and let us prove it for n +1. This follows from the fact that
1 + - < 1 + 7 < s/3 => 3^ = ^3 • 3* >
n 4 n
x- i
Thus, using this observation, we find that > 3® > => ai+i > a, for all i,
which means that aj < a2 < • an-i < an < ai> contradiction.

41. [ Mircea Lascu, Marian Tetiva ] Let x,y, z be positive real numbers which
satisfy the condition
xy + xz + yz + 2xyz = 1.
Prove that the following inequalities hold
a) xyz <
3
b) z + y + z >

c) - + - 4- - > 4 (z + y + z);
xyz
,2
(23-1)
d) - + - + -- 4(z + y + z)> , where z = max {z, y, z}.
x y 2 (3(23+1))

Solution.'.
a) We put t3 = xyz', according to the AM-GM Inequality we have
1 = xy + xz + yz + 2xyz > 3t2 + 243 <=> (24 - 1) (4 + I)2 < 0,

therefore 24 - 1 < 0 <=> 4 < this meaning that xyz < -. '
b) Denote also s = x + y + z; the following inequalities are well-known
(x + y + z)2 > 3 (xy + xz + yz)
and
(x + y 4- z)3 > 27zyz;
52 Solutions

then we have 2s3 > 54xyz = 27 - 27 (xy 4- xz 4- yz) > 27 - 9s2, i. e.

2s3 4- 9s2 - 27 > 0 <=> (2s - 3) (s -I- 3)2 > 0,

3
where from 2s-3>0«$>-. '
Or, because p < |, we have
• 8
„ t 2 9
s2 > 3g = 3(1 - 2p) > 3 (1 - -
= 4'

if we put q = xy + xz + yz, p = xyz.


Now, one can see the following is also true
3
q = xy + xz + yz > -.
4

c) The three numbers x,y,z cannot be all less than because, in this case we
get the contradiction
a 3 n 1 .
xy -I- xz 4- yz I 2xyz < - 4- 2 • - = 1:
4 8 • . .

because of symmetry we may assume then that 2 >


We have 1 = (2z 4- 1) xy 4- z (x 4- y) > (2z 4- I)xi/ 4- 22^/xy, which can also
be written in the form ((22 4- 1) y/xy - 1) (^/xy + 1) $ 0; ai‘d this one yields the
inequality
1__
xy <
(2z 4-I)2'

(x 4- y).2
Wc also have 1 = (2z 4- l)xy 4- z (x I- y) < (2z 4- 1) y—I- 2 (x 4-1/), conse­
4
quently ((22 1 1) (1 I y) - 2) (x I y I 2) > 0, which shows us that
2
X+y>
22 4- r
The inequality to be proved
111,. 1
- + - + ->4(x4-J/4-z)
xyzz ,
can be also rearranged as

(* + y) (“ ~4 P 4z2- 1 (2z- 1) (2z 4-1)


\xy ) 2 z
From the above calculations wc infer that
(z + v) (-7 > 2 ((2z + I)2 - 4) = 2 (2z — 1) (2z 4-3)
\xy / 22 + 1 22 4-1
Old and New Inequalities 53

(the assumption z > allows the multiplication of the inequalities side by side), and
this means that the problem would be solved if we proved
2(2z4-3) 2z4- 1
> <=> 4z2 4- 6z > 4z2 4- 4z 4- 1;
2z 4 1 ~ z

but this follows for z > and wc arc done.

d) Of course, if z is the greatest from the numbers x, y, z, then z > -; we saw


that
2
7 + 7 “ 4 (* + y)
y
= (* + ’/) ±-4
xy
>
2z 4- 1
(2z + 1)

2 (2z - 1) (2z 4-3) (2z - I)2


4z- - 4-
z 2 (2z 4- 1) ’
where from we get our last inequality
111,, x (2z- I)2
- 4- - 4- - - 4 z 4- y.4- z) >
x y z z (2z 4-1)
Of course, in the right-hand side z could be replaced by any of the three numbers
which is > - (two such numbers could be, surely there is one).

Remark.
It is easy to see that the given condition implies the existence of positive numbers
a, b, c such that x = , ° ,y = ------ ,z = —^-r. And now a),i») and c) reduce
b+c J c + a’ a+b
immediately to well-known inequalities! Try to prove using this substitution d).

42. [ Manlio Marangelli ] Prove that for any positive real numbers x, y, z,

3(r2y 4- y2z + z2x)(xy2 4- yz2 4- zx2) > xyz(x 4- y 4- z)3.

Solution:
Using the AM-GM Inequality, we find that
1 .i. y2* , xy2 _______________ 3y^/7yz_______________
>
3 y2z 4- z2x + x2y yz2 + zx2 4- xy2 ^3 ■ (y2z + z2x 4- x2y) ■ (yz2 + zx2 + xy2)
and two other similar relations
1 z2x yz2 3z y.ryz
-4------------------------------ 1------------ “-------------- >
3 y2z I z2x 4- x2y yz2 4 zx2 4- xy2 s/3 • (y2z I z2x I x2y) ■ (yz2 I- zx2 I- xy2)

I [ x2y t zx2 > _______________ Sx^xyz_______________


3 y2z-\-z2x-\-x2y yz2 + zx2 4- xy2 >
~ ^3 . (y2z + Z2X h . (y,2 h ZTz b xy2j
Then, adding up the three relations, we find exactly the desired inequality.
54 Solution*

43. [ Gabriel Dospinescu ] Prove that if a, b, c are real numbers such that
max{a,b,c} - min{a,b,c} < 1, then
1 4- a3 + b3 4- c3 4- Gabc > 3a2b 4- 3b2c 4- 3c2 a
Solution’.
Clearly, we may assume that a = min{a, b, c) and let us write b = a4-x,c = a4-y,
where x, y e [0,1]. It is easy to see that a3 4- b3 4- c3 - 3abc = 3a(x2 - xy4-y2) 4- x3 + y3
and a2b 4- b2c + c2a - 3abc ■= a(x2 - xy + y2) 4- x2y. So, the inequality becomes
1 4- x3 4- y3 > 3x2y. But this follows from the fact that 1 4- x3 + y3 > 3xy > 3x2y,
because 0 < x,y < 1.

44. [ Gabriel Dospinescu ) Prove that for any positive real numbers a, b, c we have

27 4- 2 4- — W? 4- (? 4- > 6(a 4- b 4- c) 4- | 4- -Y
X be J \ ca) \ ab J \a b c)

Solution:
By expanding the two sides, the inequality is equivalent to 2abc(a3 4- b3 4- c3 4-
3abc - a2b - a2c. - b2a - b2c - <?a - c2b) 4- (a3b3 4- b3c3 4- r'a3 4- 3a2b2c2 - a3b2c -
a3bc2 - nb’c2 - nb2r* - a2b3c - a2bc') > 0. But this is true from Schur’s Inequality
applied for a, b, c and ab, be, ca. ■

45. Let a0 — - and — ak 4- —. Prove that 1-- < an < 1.


2 n n
TST Singapore *

Solution:
1 ,
We have----- = ------- and so
ak 4- n
_1_ 1 ■)=’f 1
<1 2 1 —< 1.
4=0 '
«4 Ofc+l 7 4=0
n 4- ak an

I • ■ Now, because the sequence is increasing we also have 2 —— = —-— > —■


an *r=0’n + ak n 4-1
and from this inequality and the previous one we conclude immediately that
1
1---- < an < 1.
n
46. [ Calin Popa ] Let a,b,c be positive real numbers, with a,b,c e (0,1) such
that ab 4- be 4- ca - 1. Prove that
a b c 1 - a2 + 1-b2 1 - C2
+ +
1 - a2 T^b2 + T^c2
-------
a b
4--------
c
Old and New Inequalities 55

Solution:
It is known that in every triangle ABC the following identity holds
tan — tan — = 1 and because tan is bijective on [o, — ) we can set a = tan' —
A ,
, b=
B C
tany, c = tan—. The condition a,b,c 6 (0,1) tells us that the triangle ABC is
acute-angled. With this substitution, the inequality becomes
A 2 A
2 tan — 1 — tan — 1
2 -> oXp 2. 4=> tan A > 3 <=>
tan /I
1 — tan2 — 2 (an —
2 2
«• tan /I tan B tan C(tan A + tan B-(-tan C) > 3(tan A tan Z?+tan B tan G+tan C tan A) <=>
<=> (tan A + tan B + tan G)2 > 3(tan A tan B + tan B tan C + tan C tan A),
clearly true because tan A, tan B, tan C > 0.

47. ( Titu Andreescu, Gabriel Dospinescu ] Let x,y, z < 1 and x + y + z = 1.


Prove that
1
+ “--------- X
1
- 10
<3Z.
1 -I- y2

Solution’.
Using the fact that (4 - 3t)(l - 3i)2 > 0 for any I < 1, we find that
1 27
— cn(2 —x)r

Writing two similar expressions for y and z and adding them up, we find the desired
inequality.

Remark.
Tough it may seem too easy, this problem helps us to prove the following difficult
inequality
5—^ (6 + c — a)* 3
a2 + (6 -(- c)2 “ 5‘
In fact, this problem is equivalent to that difficult one. Try to prove this!

48. [ Gabriel Dospinescu ] Prove that if y/x + y/y + y/z = 1, then


(1 - x)2(l - y)2(l - z)2 > 215xyz(x + y)(y + z)(z + x)

Solution:
We put a — y/x, b= y/y and c = y/z. Then 1 - x = 1 - a2 = (a -I- b + c)2 - a2 =
(b + c)(2a + b + c). Now we have to prove that ((a + b)(b + c)(c + a)(2a + b + c)(a +
2t» + c)(a + d + 2c))2 > 2u,a2b2c2(a2 4- b2)(b2 + c2)(c2 + a2). But this inequality is true

II
56 Solutions

(a + b)4
as it follows from the following true inequalities ab(a2 + b2) < ■ (this being
8
equivalent to (a-b)4 > 0) and (2a 4-b 4-c) (a 4-2b 4-c) (a 4-b 4-2c) > 8(b4-c)(c4-a)(a4-b).

49. Let x, y, z be positive real numbers such that xyz = x + y + z + 2. Prove that
(1) xy 4- yz + zx > 2(x 4- y 4- z);
3
(2) Vx + \/y + \/z < - ,/xyz.

Solution:
The initial condition xyz = x 4- y + z + 2 can be rewritten as follows
111,
— ------ 4- ------- = 1.
1+x 1 4- y 1+z
Now, let'
1 1 l 1
■------ = a, :------- — b, —— = c.
14-x 14-y 14-z
Then
a b+c c+a a+ b
x = -— y
a a b ’2 = —

(1) We have

I xy + yzj4- zx >. 2(l + s + 2)„LL£,S+? + S+5.i^ + l+‘.‘+^ >


■ - . ■ a b b c c a
b+c c+ a a+ b
> 2 ------- 1----- +----------o a3 4- b3 + c3 4- 3abc >
a b c
> ab(a + b) 4- bc(b 4- c) + ca(c 4- a) <=> a(a - b)(a - c) > 0,
which is exactly Schur’s Inequality.
(2) Here we have
3
v/x+ x/y+ < -^y/xyz^
a b b c c a 3
«• ------- --------- t- • ------ T 4'
b+ c c+a c. I- a a + b a 4- b b4- c “ 22'
This can be proved by adding the inequality
l~~a " b 1 a b X
V b 4- c c + a ~ 2 --------- h 7----- 1 i
a 4- c b 4- c J
with the analogous ones.

50. Prove that if x, y. z are real numbers such that x1 4- y2 + z2 = 2, then •


x + y + z < xyz + 2.
IMO Shortlist, 1987
Old and New Inequalities 57

First solution'.
If one of x, y, z is negative, let us say x then

2 + xyz - x - y - z = (2 - y - z) - x(l - yz) > 0,


___________ ,2 j. „2
because y + z < \/2(y2 + z2) < 2 and zy < < 1. So we may assume that
2
0<x<y<z.Ifz<l then

2 + xyz -x-y-z = (l- z)(l - xy) + (1 - x)(l - y) > 0.

Now, if z > 1 we have

z + (x + y) < \/2(z2 + (x + y)2) = 2x/l + xy < 2 + xy < 2 4- xyz.

This ends the proof.

Second solution'.
Using the Cauchy-Schwarz Inequality, we find that

x + y + z - xyz — x(l - yz) + y + z < \/(x2 + (y + z)2) ■ (1 + (1 - yz)2).

So, it is enough to prove that this last quantity is at most 2, which is equivalent to
the inequality (2 + 2yz)(2 - 2yz + (yz)2) < 4 <=> 2(yz)3 < 2(yz)2, which is clearly
true, because 2 > y2 + z2 > 2yz.

51. [ Titu Andreescu, Gabriel Dospinescu ] Prove that for any xi,x2,...,xn 6
(0,1) and for any permutation o of the set {1,2,we have the inequality
n
/
n E Xi

E
1
I — Xi
> 1+—
n (e i
1 — Xi • Xg(i)

\ /

Solution:
1 1 ‘ 1
Using the AM-GM Inequality and the fact that <----- 1----- , we can
x 1 y 4x 4y
write the following chain of inequalities
n n n
1 1
E i - z. • y, S E— 1-11-
= 2E 1“ I
<
2
1 1 \ ’* i

2(1-x2) 2(i-yf)
5X Solutions

So, it is enough to prove the inequality

\
n
1
< E Xi
/ n 1 \
E 1 - XI
> 1 4- —
n • E 1 - x2 ) <=>

\ /
n n t \
Xj
■ ( E 1 _x2 J ’
E 1 - Xj
>-•
n
>>i
\i=l1 ’/

which is Chebyshev Inequality for the systems (xi,x2,... ,xn) and


( 1 1__ __ 1
\ 1 - xf 1 - x22' ’'' ’ 1 - x2n) ‘

n
1
52. Let x1,x2,...,xn be positive real numbers such that = 1. Prove
i=i
1 + X,
that

Vojtech Jarnik
t

First solution:

Let ~ = a,. The inequality becomes

n n
1 - at Qt 1
E
i 1
a.
ao-nE^ - a. -E \/QiU ~ Oi) >
n n
Oi x-' I a< 1
1 ~ a> ~"E V 1 ~ ai s(E-<
1=1
E yai(l — ch) J \i=l .

But the last inequality is a consequence of Chebyshev’s Inequality for the n-tuples
(ai,a2,...,an) and

1___________ 1 1
yai(l - ai)’ ya2(l - a2)’ ’ yan(l -an) J

Solution 2:
With the same notations, we have to prove that

(n -1) E J — a' — - x~~ <


■“ V al + a2 + • • • + ai-l + ai+l 4- ... 4- On

n
/ai 4- g2 4- ... 4- at-i 4- <2,-1 4- ■ ■ ■ 4- an
- er V
Old and New Inequalities 59

But using the Cauchy-Schwarz Inequality and the AM-GM Inequality, we


deduce that
n
E >

x/a? + x/d2 + . . . + y/Cli-1 + y/aTr\ + . . . + y/ttn


>
y/n - 1 • y/ai
n
y/Q-i 1 1 i 1 1
£ y/n — 1 —= + -= +-•••+• -—+ -'Qi+1
vai Va2
— + ••• + -p= >
VQn 'aiT?
n
(n - 1) x/ n - 1 • y/a~t
> E x/a? + x/o2 + ■ • • + +■ yaj-i +-■•• +
>

>

and we are done.

53. [ Titu Andreescu ) Let n > 3 and ai, da. • * < > a„ be real numbers such that
Qj + a2 +.. . + an > n and a2 + a| + • • • + °n — n2- Prove that max{ai, a2,... ,an} >2.

USAMO, 1999
Solution:
The most natural idea is to suppose that a* < 2 for all i and to substitute
n n
x, = 2 — a< > 0. Then wc have ^(2 — x<) > n => x» — n a^so

n 2
n
Now, using the fact that x, > 0, we obtain x2 < E* , which combined with

the above inequality yields


n / n 2 n
! n2 < 4n — 4 I» + I x‘ 4n -I- (n — 4) x.

n n
(wc have used the fact that < n). Thus, we have (n - 4) | Xi — Il > 0,

I n
which is clearly impossible since n > 4 and x, < n. So, our assumption was wrong

and consequently max{ai, d2,..., an} > 2.


GO Solutions

54. [ Vasile Cirtoaje ] If a, ft, c, d are positive real numbers, then


a - b b- c c-d d- ci n
ftlc c I d dd II aa alb

Solution.’.
We have
a-b b-c c-d d - a =a------
+c b+d cc ++ aa d+b
------ 4.------- 4 ft + d -|.------- 4----------- 4 =
b + c-- c + d d+a a + ft c + d dd++aa a + ft
ft + c---- c-f-d
, , ( 1 1 1
= (a + c) I —— + 3—— + (b + d) f—— + ——7 -4..
\ ft + c d + a \c 4~ d a 4*
cl + 0ft
Since
1 1 4 1 1 > 4
ft + c d + a “ (ft + c) + (d + a) ’ c. + d a + ft ~ (c + d) + (a. + ft) ’
wc fiCt
a -ft ft - r c-d. d - ri 4(a l c) 4(ft I d)
—- ■ 4- —4" ~
ft+c c + d d + a a + ft
4* — >
(ft + c) + (d + a)
+ (c + d) + (a + ft)
-4 = 0.

Equality holds for a - c and ft - d.


I
Conjecture. (Vasile. Cirtoaje.)
If a,b,c,d,c. arc positive real numbers, then
a - ft ft - c c-d d — e. e - a
------ +------- +------- +------- 4-------- > 0.
ft + c--- c + d d + e e + a--- a + ft- “

55. If x and y are positive real numbers, show that xv + yx > 1.


France, 1996

Solution:
a
We will prove that ab >----- :----- - for any a, b 6 (0,1). Indeed, from Bernoulli
OL ~ ab
a + 0b - GO *
Inequality it follows that a1-6 = (1 + a - I)1-4 < 1 + (a - 1)(1 - ft) = a + 6 - ab
and thus the conclusion. Now, it x or y is at least 1, we are done. Otherwise, let
l
0 < x,y < 1. In this case we apply the above observation and find that xv + yx >
—^—+ —y—
x + y-xy x + y-xy x + y x + y
56. Prove that if a, b,c > 0 have product 1, then
(a + b)(b + c)(c + a) > 4(a + b + c - 1).
MOSP, 2001
Old and New Inequalities til

First solution:
Using the identity (a 4- b)(b 4- c)(c 4- a) = (a 4- b 4- c)(ab 4- be 4- ca) - 1 we reduce
the problem to the following one
3
ab 4- be 4- ca 4- > 4.
a 4- b 4- c
Now, we can apply the AM-GM Inequality in the following form

3 J (ab 4- be 4- ca)3
ab 4- be 4- ca 4- > 4
a 4- b 4- c y 9(a4-i>4-c)
And so it is enough to prove that

(ab + be 4- ca)3 > 9(a + b 4- c).

But this is easy, because we clearly have ab 4- be 4- ca > 3 and (ab 4- be 4- ca)2 >
3abc(a 4- b 4- c) = 3(a 4- b 4- c).

Second, solution:
8
Wc will use the fact Chat (a 4- b)(b 4- c)(c 4- a) > -(a 4- b 4- c)(a‘b + be 4- co).
9
2 _ 1
So, it is enough to prove that - (ab 4- be 4- ca) 4- > 1. Using the AM-GM
a I b I c
Inequality, we can write

2. , 1 J (ab 4- be 4- ca)2
— (ab -h be l ca) >3 > 1,
a 4- b 4- c y 81 (a 4- b 4- c)
because
(ab 4 be 4- ca)2 > 3abc(a + b+ c) = 3(a 4- b 4- c).

57. Prove that for any a, b, c > 0,

(a2 4- b2 4- c2)(a 4- b — c)(6 4- c - a)(c 4- a - b) < abc(ab 4- be 4- ca).

Solution:
Clearly, if one of the factors in the left-hand side is negative, we are done. So,
we may assume that a,b,c are the side lenghts of a triangle ABC. With the usual
notations in a triangle, the inequality becomes
16 K2
(a24b24c2)- < abc(at>+bc l-ca) (a4-b4c)(ab4-bc4-ca)/?2 > abc(a2 Ft24-c2).
a+ b+c
But this follows from the fact chat (a 4- b 4- c)(ab 4- be 4- ca) > 9abc and

0 < OH2 = 9/?2 - a2 - b2 - c2.


62 Solutions

58. [ D.P.Mavlo ) Let a,t,c > 0. Prove that


„ , 1 1 1 a b . c (a+l)(b+l)(c4-l)
c4----- F t7 14----- F 7 4- ■- 4— >3
3 + a + 6 + cd
a b0 c b c a 1 + abc
Kvant, 1988

Solution:
The inequality is equivalent to the following one
_ „ 1 _ n \\ib +? a
atc+1
or
abc£a4-£i + £a2c+52j >2(£a + £ab).

But this follows from the inequalities

n2bc I - > 2ab, b2cn I - > 2hr.,c2nb \ T > 2rn


cab
and
>
a2c I - > 2a, b2 a I - > 2b,c2b | 7 > 2c.
c " a ~ b

59. [ Gabriel Dospinescu ] Prove that for any positive real numbers Xi,X2, •. . ,x„
with product 1 we have the inequality
n n n
nn
i=l \>=1 t=l */

Solution:
Using the AM-GM Inequality, we deduce that
x? x? x^_i , 1 n
4- >
1 4- x" 14-x? l + x^
rf[(l+«?)
i=l

and
1 ___ 1 n • xn
1+xS >
I 11------ n *•••• 1 F
-r Xj 1 4- x„.
: n<i + x.“)
\ »=l

Thus, we have

? n(l-Fx")>^4-^-.
\ t=i x"
Old and New Inequalities 63

Of course, this is true for any other variable, so we can add all these inequalities to
obtain that
n n n
n" nd + O +
which is the desired inequality.

60. Let a, b, c, d > 0 such that a 4- b 4- a = 1. Prove that


11 _d
a3 4- b3 4- c3 4- abed > min 4 ’ 9 + 27
27
Kvant, 1993

Solution:
Suppose the inequality is false. Then we have, taking into account that abc <

the inequality d — abc | > a3 4- b3 4- c3 - . We may assume that abc < -z. Now,
27 J 9 2<
we will reach a contradiction proving that a3 4- b3 4- c3 4- abed > It is sufficient to
•1
prove that
a3 + + c3 _ 1
------ i-------------— abc 4- a3 4- b3 4- c3 > |.
A-abc 4
— ~ abc
27
But this inequality is equivalent to 4 q3 4- 15abc > 1. We use now the identity
1 4- Qabc.
a3 = 3abc-f-1 — 3 anc^ reduce the problem to proving that ab <
4
which is Schur’s Inequality.

61. Prove that for any real numbers a, b, c we have the inequality
J2(l + a2)2(l 4- b2)2(a - c)2(b - c)2 >(14- a2)(l + b2)(l + c2)(a - b)2(b- c)2(c - a)2.

AMM

Solution:
The inequality can be also written as Y' 7——~rr? — 1 (°f course, we may
' (14 cq(a — b)z
assume that a, b,c are distinct). Now, adding the inequalities
(1 4-a2)(l 4-b2) 4-b2)(l 4-
(1 4- 4-c2) 1 + b2
4- > 2
(1 4 c2)(n - b)2 4- a2)(b - c)2
(1 4- |a-b||c-b|
(which can be found using the AM-GM Inequality) we deduce that
v—' (1 4- a2)(l 4- b2) 1 4- b~
^(14 c2)(a - b)2 ~ *E [(b-a)(b-c)|
64 Solutions

and so it is enough to prove that the last quantity is at least 1. But it follows from
1 + b2 1 + b2
E |(b-a)(fr-c)| (b - a)(b - c)
=1

and the problem is solved.

62. [ Titu Andreasen, Mircea Lascu ] Let q,x, y, z be positive real numbers such
that xyz = 1 and n > 1. Prove that
xa ya za 3
y+ z z + x xx+y 2
+ y

First solution:
Wc may of course assume that x > y > z. Then we have
xys
------ > —— > ------
y I z z I x x |- y
and x* >y' > z0-1. Using Chebyshev’s Inequality we infer that

Now, all wc have to do is to observe that this follows from the inequalities xQ-1 > 3
z 3
(from the AM-GM Inequality) and - > -

Second solution:
According to the Cauchy-Schwarz Inequality, we have:
xa ya za 2
|(x(y+z) + y(z+x) + z(x+y)]] -------- f.------- -|-------- X^ +1/^+2^
y+z z+x x+y
Thus it remains to show that
I + zz 1^ > 3(xy + yz + zx).
(x1^2 + y1^2 +

Since (x + y + z)2 > 3(xy + yz + zx), it is enough to prove that

+ t/
x1^1 + .’/ 1’2 + z2^2 > x + y + z.
From Bernoulli’s Inequality, we get
, 1 + a, 1 -a T+ a
— [] + (x - 1)] > 1 + —(x-1)- —+
~T~X
and. similarly,
1-n l + o no 1 -a 1 +a
</ - 2 + 2 1/12 ~ 2 + ‘2 Z‘
Tints
i + q,
z-r- + y-r- + 3-r-_ (j + y + 2) > 3(1-a) + — v / x
2~(x + y + z) - (x + y + z) =
2
Old and New Inequalities 65

- ^2~(i + !/ + z-3) > a — 1 (3 tyxyz - 3) = 0.


2
Equality holds for x — y = z — 1.

Remark:
1 i z — - (abc - 1) the
Using the substitution 0 = a + 1 (0 > 2) and x = y =
a b c
inequality becomes as follows
1 1 . 1 3
a°(b 4- c) + bd(c + a) + ^(a I b) ~ 2
For 0 = 3, we obtain one of the problems of IMO 1995 (proposed by Russia).

63. Prove that for any real numbers Xi,..., xn, yi,...,yn such that xf d----- Fx2 =
l/i + ••• + !/?, = 1,
/ n
(xij/2 - X21/1)2 <211 - ^xkyk
\ fc=i
Korea, 2001

Solution:
We clearly have the inequality
n / n \2
(111/2-X21/1)2< 52
l<i<J<n
(^i/j- 52(J2^ -152 ]
H \ ( u

i - 52 ) (1 ■* 52^
i-i
h ;
Because we also have 52 x«y* — h we immediately that
i=l i
/ n n / n
11 - 52 ^y* 1 + 52z‘yi <2(1-52 X'V'
and the problem is solved.

64. | Lauren^iu Panaitopol ] Let ai,a2,...,a,t be pairwise distinct positive inte­


gers. Prove that
2n + 1
al + a2 + ■ ■ ' + an — (ai + 02 + ■ • • + Qn)>
3
TST Romania
66 Solutions

Solution:
Without loss of generality, we may assume that dj < d2 < • • ■ < and hence.
a, > i for all t. Thus, we may take b< = at - i > 0 and the inequality becomes
-A .2 a V"' 1 n(n + l)(2n + 1) 2n 4-I- 1 v~'. , n(n + l)(2n + 1)
Eb>+ 2 E lb< +------ g------ ->~T~'
3 ■Y,
E bi++------- e------ •
Now, using the fact that «j+i > a, we infer that &i < b2 < ••■ < b,t and from
Chebyshev’s Inequality we deduce that
n n n
2n+ 1
■2^^, > (n I 1)£> >
3 E*
and (he conclusion is immediate. Also, from the above relations we can see imme­
diately that we have equality if and only if ai,a2,...,an is a permutation of the
numbers 1,2.... ,n.

65. [ CSJin Popa ] Let a,b,c be positive real numbers such that a + b + c = 1.
Prove that
by/c
byfc Cy/a a\/b >
4
a(y/3c I- Vab) b(\/3a + x/bc) c(\/3b -I- y/ca) - ■
I

Solution:
Rewrite the inequality in the form

3\/3
l3c.a + a
^>~r
4
^~

[be [ca [ab , ,


With the substitution .r. = —,z = y — the condition a + b + c = I
I becomes ,ry | yz I zx 1 and the inequality turns into
I
•T
E v/3y + yz > “ 4 ’

But, by applying the Cauchy-Schwarz Inequality we obtain

■r2

E y/?>xy t- xyz > (E*r


>/3 I 3zi/c
3E*y 3v/3
4 ’
"73 + E
\/3
where we used the inequalities

(Ex) 11/2-^5-

I
Old and New Inequalities 67

66. [ Tilu Andreescu, Gabriel Dospinescu ] Let a, b, c, d be real numbers such that
(1 4- a2)(l 4- b2)(l + c2)(l + d2) = 16. Prove that
—3 < ab 4- be + cd -t- da 4- ac 4- bd - abed < 5.

Solution:
Let us write the condition in the form 16 = JJ(i + a) ■ JJ(a — ’)• Using symmetric
sums, we can write this as follows
16 = (1 - i a— ab 4- i aba + abed) (1 + i a— ~ ’ E/ ahc+ n^) •

So, we have the identity 16 = (1 — ^2 ab + abed)2 4- (52 a ~ 52 ohc)2. This means that
|1 - £ab+ abcd| < 4 and from here the conclusion follows.

67. Prove that


) (a2 4- 2)(b2 + 2)(c2 + 2) > 9(ab + bc~ ca)
for any positive real numbers a, b, c.
APMO, 2004

First solution:
We will prove even more: (a2 I- 2)(b2 4 2)(c2 I 2) > 3(a I b I c)2. Because
(a 4- b + c)2 < (|a| 4- |b| 4- |c|)2, we may assume that a,b, c are noimegative. X\e will
use the fact that if x and y have the same sign then (1 4- x)(l 4- y) > 1 4- x 4- y- So,
we write the inequality in the form

and we have three cases


n(^ +1 >
(a 4- b 4- c)2
9

a2 - 1
i) If a, b, c are at least 1, then PJ
3
4- 1 - 1 + E 3
a2 — 1
- g
o»2
ii) If two of the three numbers are at least 1, let them be a and b, then we have

n(^ +1 >
a2 - 1 b2 - 1\ (c2 +2\
1 + 3 ++ 33 J Q 3 J
(a2 + b2 + 1) (I2 + I2 + c2) > (a 4- b 4- c)2
>
9 9 ~ 9
by the Cauchy-Schwarz Inequality.
iii) If all three numbers are at most 1, then by Bernoulli Inequality we have

n(^ >* + E
a2 - 1
3
>
(z»29
and the proof is complete.
68 Solutions

Second solution:
Expanding everything, we reduce the problem to proving that
(aba)2 4- 2 y a2b2 4- 4 y a2 4- 8 > 9 y ab.

Because 3 y a2 > 3 y; ab and 2 y a2t>2 + 6 > 4 y ab, we are left with the
inequality (nbc)2 4- y a2 + 2 > 2y ab. Of course, we can assume that a,b,c are
non-negative and we can write a = x2,b = y2,c = z2. In this case
2y^ob - yy2 = (r + y + z)(r + y- z)(y4- z - j)(z + x-y').
It. is clear that if x,y,z are not side lengths of a triangle, then the inequality • •
is trivial. Otherwise, we can take x - u + v,y = v 4- w,z = w 4- u and reduce the
inequality to
((u + v)(v 4- w)(w + it))4 + 2 > 16(u + v + w)uvw,
We have ((u + v)(v + w)(w 4- u))4 4- 1 + 1 > 3 (u 4- v)*1 (v 4- w)4 (u 4- w)4 and
it remains to prove that the last quantity is at least 16(u 4- v 4- w\uvw. This comes
down t.o
163
(u 4- v)4(v + w)4(w + u)4 > —^-(uvw)3(u 4- v + w)3.
v
But this follows from the known inequalities
g
(u 4- v)(v + w)(w 4- u) > ~(u + V + w)(uv + VW 4- wu),
g
(uv -I- VW + wit)4 > 34(wvw)3, 11 + v + w > 3\/uvw.

Third solution:
In the same manner as in the Second solution, we reduce the problem to proving
that
(abc)2 4- 2 > 2 y ab - y a2.
Now, using Schur’s Inequality, we infer that
—' ,
2 vy ab - vy
-' o tycibc.
a2 < -— ----
a 4- b 4- c
and as an immediate consequence of the AM-GM Inequality we have
-9-,bC <3^bc^.
a I b 4- c v '
This shows that as soon as we prove that
(abc)2 + 2 > 3 (/(abc)2,
the problem is solved. But the last assertion follows from the AM-GM Inequality.
Old and New Inequalities G9

68. [ Vasile Cirtoaje ] Prove that ifO<x<y<z and x + y + z = xyz + 2, then


a) (1 ~ ^3/)(l - l/z)(l - > 0;
b) x2y < l,x3y2 <

Solution:
a) We have

(1 -xy)(l-yz) = 1 - xy - yz + xy2z - 1 - xy - yz + y(x 4 y + z - 2) - (y - I)2 > I)

and similarly

(1 - yz)(l - zx) = (1 - z)2 > 0, (1 - zx)(l - xy) - (1 - x)2 > 0.

So the expressions 1 - xy, 1 - yz and 1 - zx have the same sign.


b) We rewrite the relation x + y + x = xyz+ 2 as(l—x)(l —y) + (l—x)(l-xy) = 0.
If x > 1 then z > y > x > 1 and so (1 - x)(l - y) + (1 - z)(l - xy) > 0, impossible.
So we have x < 1. Next we distinguish two cases 1) xy < 1; 2) xy > 1.
32
1) xy < 1. We have x2y < x < 1 and x3y2 < x < 1 <
2) xy > 1. From y > ^/xy we get y > 1. Next we rewrite the relation x + y + z =
xyz + 2asx + y- 2 = (xy - l)z. Because z > y gives x + y - 2 > (xy - l)y,
(y - 1)(2 - x — xy) > 0 so 2 > x(l + y). Using the AM-GM Inequality, we have
1 + y > 2yy and l+y=l+|+|>3 + y2 ^22 Tl,us wc l,ave > 2xyy and
r~^
2 > 3xy —, which means that x2y < 1 and x3y2 < —
32 &
- 27'
32
The equality x2y - 1 takes place when x y - 1 and the equality x’y2
27
2
lakes place when x = y = z = 2.

69. [ Titu Andrccscu ] Let a, b, a be positive real numbers such that a -i- 6-t c > abc.
Prove that at least two of the inequalities
236,236„236„ 3
- -r r + - > 6, -r + - + ->6, - + - + 7 > 6.
abc b c. a cab a
are true.
TST 2001. USA

Solution:
The most natural idea is to male the substitution - = x, 7 = y, - = z. Thus, we
a b c
have x, y,z > 0 and xy + yz + zx > 1 and we have to prove that at least two of the
inequalities 2x+3y + 6c > 6, 2y+3x + 6x > 6, 2c + 3x+6y > 6 are true. Suppose this is
not the case. Then we may assume that 2x+3y + 6z < 6 and 2z + 3x+6y < 6. Adding,
we find that 5x + 9y + 8s < 12. But we have x > Thus, 12 > -—+ 9y + 8z
70 Solutions

which is the same as 12(y + c) > 5 + 9y2 + 8z2 + 12yz (2z -1)2 + (3y + 2z- 2)2 < 0,
which is clearly impossible. Tints, the conclusion follows.

70. | Gabriel Dospincscu, Marian Tetiva | Let x,y, z > 0 such that
x + y + z = xyz.
Prove that
(i - 1) (y - 1) (z - 1) < 6\/3 - 10.

First solution’.
Because of x < xyz => yz > 1 (and the similar relations xz > 1, xy > 1) at most .
one of the three numbers can be less than 1. In any of these cases (x < 1, y > 1,
z > 1 or the similar ones) the inequality to prove is clear. The only case we still have
to analyse is that when x > 1, y > 1 and z > 1.
In this situation denote
x-l = a, y-l = b, z - 1 = c.
I
I
Then a,b,c are nonnegative real numbers and, because
x = a+l,y = 6+l, z = c + 1,
they satisfy
a + 1 + b + 1 + c + 1 — (<i + 1) (6 + 1) (c + 1),
which means
abc + ab + ac + be = 2.
Now let x = \/abc; we have
ab + ac + be > SVabacbc = 3x2,

I that’s why wc get


x3 + 3x2 < 2 <=> (x + 1) (x2 + 2x - 2) < 0 <=>
«• (x + 1) (x + 1 + x/5) (x + 1 - \/3) < 0.
For x > 0, this yields
\/abc - x < v/3 - 1,
or, equivalently
abc< (x/3 — l)3,

which is exactly
(x - 1) (y - 1) (z - 1) < 6t/3 - 10..
The proof is complete.
Old and New Inequalities 71

Second solution-.
Like in the first solution (and due to the symmetry) we may suppose that x > 1,
y > 1; we can even assume that x > 1, y > 1 (for x = 1 the inequality is plain). Then
we get xy > 1 and from the given1 condition we have
x —
x + y-i •
. xy - 1
The relation to prove is
(x - 1) (y - 1) (z - 1) < 675 - 10 <=>
<=> 2xyz — (xy + xz + yz) .< 675 — 9,
or, with this expression of z, ' * ’• . .
2xy- + y - xy - (1 + y) - + V- < 6 75 - 9 ■»
xy - 1 ■ xy — 1
<=> (xy - x - y)2 + (675 - 10) xy < 675 - 9,
after some calculations. • ■ ■
Now, we put x = a+ l,y = 6+ l and transform this into •
a2b2 + (&T5 -r io) (a + b + ab) - 2ab > 0.

But
a + b > 2 Tab
and 675 - 10 > 0, so it suffices to show that
a2b2 + (675 — io) (2>/ab + ab) - 2ab > 0.,

The substitution t = Vab > 0 reduces this inequality to


t4 + ^6x/3 — 12) I2 + 2 (gT5 - io) 1 > 0,

or
t3 + (675 - 12) t + 2 (gT5 - 10) >0.

The derivative of the function


f (t) = t3 + (gT5 - 12) t + 2 (gT5 - io) , t > 0

is
/'(t) = 3 (t2 - (73
-1

and has only one positive zero. It is 75 — 1 and it’s easy to see that this is a minimum
point for f in the interval [0,co). Consequently
/(*)>/(75-1) =0,

and we are done.


A final observation: in fact we have
/ (t) = (t - 73 +1)2 (t + 2J3 - 2),
72 Solutions

which shows that / (t) > 0 for t > 0.

71. [ Marian Tetiva ] Prove that for any positive real numbers a,b,c,
a3 - l> ' b3 - c3 r’-a3 (a-b)2 + (b-c)2-F(c-a)2
——----F
b+c c+ a
<
4“-------------
4
a -F b
Moldova TST, 2004

First solution’.
First of all, we observe that right hand side can be transformed into
<? - b3 = (a..‘-b..3)(/ —1 k 1 VX (b3-c3)(-^~- 1
a -F b ka + b
' \a aa+cj
+ cj \b + c a+c
(a - b)(c - b)(a - c) (£ab)
(a + b)(a + c)(b + c)
and so we have to prove the inequality
|(a - b)(b - c)(c - a)|(ab + bc-F ca)
(a + b)(b + c)(c + a) s I (£«*-£■»)•
8 + b + c)(ab + be + ca) and
It is also easy to prove that (a + b)(b + c)(c -Fa) > |(a
9
' we are left with .

Ing the AM-GM Inequality, we reduce this inequality to the following one

£a»3*|n<«-4
This one is easy. Just observe chat we can assume that a > b > c and in this case it
becomes
8
(a - b)(a - c)(b - c) < ^(« + b I- c)3
i and it follows from the AM-GM Inequality.

Second solution (by Marian Tetiva)’.


It is easy to see that the inequality is not only cyclic, but symmetric. That is why
we may assume that a > b > c > 0. The idea is to use the inequality
y^ + ^ + y2 >v + £
x+ 2 “
> x+y ~y 772'
>1/+
which is true if x > y > 0. The proof of this inequality is easy and we won’t insist.
Now, because a > b > c > 0, we have the three inequalities
b a2 + ab + b2 a c b2 + be + c2 • b
a + 7 > ---------------- > b+ -,b+ - >---- —------- >
2 a 4* b 2 2 ofc 2
and of course
a2 + ac + c2 a
a+ £ > a+c 2‘
OI<1 and New Inequalities 73

That is why we can write


y, a3 - b3 a2 I tib + b2 b2 4 be. f- c2 , . a2 I ac I c2
(a - b) + (6 - c) —c -(°-e) a + c >
a+b a+b

> (a - b) (b F I- (b- e) I e + 0 - (a - e) ('

E(q ~6)2
4 ' '
In the same manner we can prove that
E(a-fe)2
E^<
a+b ~ 4
and the conclusion follows.

72. [ Titu Andreescu ] Let a, b,c be positive real numbers. Prove that
(a5 - a2 + 3)(b5 - b2. + 3)(c° - c2 + 3) > (a + b + c)3.
USAMO, 2004

Solution'. ■ ■
We start with the inequality a5 — a2 + 3 > a3 + 2 <=> (a2 - l)(a3 — 1) > 0. Thus,
it remains to show that
n (°3+2) (e a) •
Using the AM-GM Inequality, one has
a3 1 1 3a
+ >
a3+ 2 + b3 + 2 ^4-2

Wc write two similar inequalities and then add up all these relations. We will find
that
II(a3 + 2)> (Ea)3'
which is what, we wanted.

73. [ Gabriel Dospinescu ] Let n > 2 and xi, Z2,..., zn > 0 such that
/ n \ / n - \

(E*»
\fc=l
(E^ / ="2 + ‘-
/ \fc=l
Prove that
n i 2
E^
,k=l
e?
fc=l Xk n(n-l)'
74 Solutions

Solution:
In this problem, a combination between identities and the Cauchy-Schwarz
inequality is the way to proceed. So, let us start with the expression
2

E f- + £-2 Xi

We can immediately see that


s 2
. Xj .Xi Xj ]
E (r +
l<«<7<n
fl -2 |
Xi
E
1<«<2<«
fl + -4 — 4 ----------- 4 . -i + b
Xi Xj Xi I
=

n
1
■ = IE*)(e^ i=l 1
-4
.1=1 /
(Ej-l+3n2-
\i=l •/

Thus we’could find from the inequality


2
'fi + fl _ 2
E
l<i<j<n
< Xj Xi
>0

that
n
1
\i-i
) (E?) ^n2+4-
I‘i</
/ \>=i X

'Jnfortunalely, this is not enough. So, let us try to minimize


/ \ 2

£ f- + 5i-2
~ Xi

This could be done using the Cauchy-Schwarz Inequality:


2

^-2
2 Xi

E X1
1 ^-2
Xi
>

Because E (-+ —
Xi
- 2 ] = 1, we deduce that
)

n
2
~2 ] > «2 + 4 1-
' \i=l X / n(n - 1) ’

which is what we wanted to prove. Of course, we should prove that we cannot have
equality. But equality would imply that = x2 = ••• s= xn, which contradicts the
assumption
n
=n2 + l.
,k=l .*:=! Xk*/
Old and New Inequalities 75

74. [ Gabriel Dospinescu, Mircea Lascu, Marian Tetiva ] Prove that for any po­
sitive real numbers a, b, c,
a2 + b2 + c2 + 2abc + 3 > (1 + a)(l + b)( 1 + c).

First solution:
Let /(a, b, c) = a2 + b2 + c2 + abc + 2 - a - b - c — ab - be - ca. We have to prove
that all values of f arc nonnegative. If a, b, c > 3, then we clearly have - + - + - < 1,
Cl 0 C
which means that /(a, b, c) > a2 + b2 + c2 + 2 — a — b - c > 0. So, we may assume
d *1“ c
that a < 3 and let m. — —-—. Easy computations show that /(a, b, c) - f(a,m, m) =

———— > 0 and so it remains to prove that /(a, m, m) > 0, which is the same
as
(a + l)m2 — 2(a + l)m + a2 - a + 2 > 0.
This is clearly true, because the discriminant of the quadratic equation is —4(a +
l)(a-l)2<0.

Second, solution:
Recall Turkevici’s Inequality
z4 + y4 + z4 + t4 + 2xyzt > x2y2 + y2z2 + z2t2 + t2x2 + x2z2 + y2t2
for all positive real numbers x,y,z,t. Taking t — l,a = z2,b = yr,x = z2 and using
the fact that 2\/abc < abc + 1, we find the desired inequality.

75. [ TitU Andreescu, Zuming Feng ] Let a,b, c be positive real numbers. Prove
that
(2a + b + c)2 (2b + a + c)2 (2c + a + b)2
2a2 + (b + c)2 2b2 + (a + c)2
+ 2c2 + (a + b)2
< 8.
USAMO, 2003

First solution:
Because the inequality is homogeneous, we can assume that a -l b I c. - 3. Then
(2a + b 4- c)2 a2 + 6a + 9 1 / 4a + 3
1 + 2-
2a2 + (b + c)2 “ 3a2 - 6a + 9 “ 3< 2+ (a - I)2
4a + 3\ 4a + 4
~2 ) ~ 3 '
Thus
(2a + b + c)2 < | + 4) = 8.
' 2a2 -I- (b -|- a)2
76 Solutions

Second solution:
b+c c+a
c4- a + b XU x. , >
Denote x =------, y = —;, z =------ . We have to prove that
a ■ b
b c
(^ T 2)2
2^ X2 _| 2 -
o _ v-' 2x T * < 5
x2 .h 2 - 2
But, from rhe Cauchy-Schwarz Inequality, we have
V' ~ I)2
x2 + 2 >4
v(x-l)2 (x4-y + z-3)2
2-, xz 4. 2 > i2 4- y2 4- z2 4- 6 ’
It remains to prove that
2(x2 + y2 + z2 4- 2ii/ 4- 2yz f 2zx - 6x - 6y - 6z 4- 9) > x2 + y2 4- z2 4- 6 <=>

<=> x2 4- y2 + z2 + 4(xy 4- yz 4- zx) — 12(x 4-y 4- z) + 12 > 0.


Now xy +:yz 4- zx > 3 \Jx2y2z2 > 12 (because xyz > 8), so we still have to prove that
(x 4- y 4- zj2 + 24 - 12(x 4- y 4- z) 4-12 > 0, which is equivalent to (x 4- y 4- z - 6)2 > 0,
clearly true. • .

76. Prove that for any positive real numbers x, y and any positive integers m, n,
(n -1) (m -1) (xm+,‘ 4- ym+n) 4- (m 4- n -1) (xmyn 4- xnym) > mn(xm+n"1 y 4- y’n+n~1 x).
Austrian-Polish Competition, 1995

Solution:
We transform the inequality as follows:
rnn(x -y)(xm*n-' -ym*H~’’) > (ni 4- n - l)(xm - y,n)(xn - y") <=>
x.m+n-l
mTn-i _ ytn-t-n-i xm_ym Xn - yn
<=> >
(mln- l)(x - y) ~ m(x - y) n(x - y)
(we have assumed that x > y). The last relation can also be written

,n+ ~2dl > F tm~'dt ■ [X tn~ldt


(x - y) f tlm+n
Jy Jy Jy

and this follows from Chebyshev’s Inequality for integrals.

77. Let a,b,c,d,e be positive real numbers such that abede = 1. Prove that
a 4- abc b 4- bed c 4- cde d 4- dea . t e 4- eab
--------- > 10
4- 4---------------------- F
1 4- ab 4- abed 1 4- be 4- bede 1 4- cd 4- odea 1.4- de 4- deab 1 4- ea 4- eabc ~1 3
Crux Mathematicorum

Solution:
We consider the standard substitution
z . V z , I u
a— ~/ -,C c = -,a
2 t U X
Old and New Inequalities 77

with x,y,z,t,u > 0. It is clear that


1 1
— + *7
a 4- abc y__ L
1 + ab 4- abed ■ 1 + 11 1
x z u
Writing the other relations as well, and denoting -1 = ai, — a2, |1 = an, y = a,t, -1 =
X z u
a5, we have to prove that if a, > 0, then
y-A 02 + 04 >
ai 4- a3 4- as ~ 3
Using the Cauchy-Schwarz Inequality, we minor the left-hand side with
____________________________ 4S2____________________________
2S2 - (a2 4- a4)2 - (ai 4- a^)2 — (a3 4- as)2 - (02 4- as)2 - (a1 4- a3)2 ’
5
where S = By applying the Cauchy-Schwarz Inequality again for the

denominator of the fraction, we obtain the conclusion.

78. [ Titu Andrccscu ] Prove that for any a,b,c, e (fl, the following inequality
holds -
sin a • sin (a - b) ■ sin (a - c) sin b • sin(b - c) ■ sin(b - a) sin c ■ sin(c - a) ■ sin(c - b)
sin(b 4- c) + sin(c4-a) sin(a 4-b) >0.
TST. 2003, USA
Solution: ~ • ’
Let x = sin a, y = sin b, z = sin c. Then we have x, y, z > 0. It is easy to see that
the following relations are true:
sin a ■ sin(a - b) • sin(a — c) • sin(a + b) ■ sin(a + c) = r(r2 - y2)(x2 - z2)

Using similar relations for the other terms, we have to prove that:
^2x(x2 - y2)(x2 - y2) > 0.

With the substitution x = y/u, y = \/v, z = y/w the inequality becomes ~


v)(u - io) > 0. But this follows from Schur’s Inequality.

79. Prove that if a,b, c are positive real numbers then,


\/a4 4- b4 4- c4 4- \/a2b2 4- b2c2 + c2a2 > \/a3b + b3c + c3a 4- y/ab3 4- be’ 4- ca3.
KMO Summer Program Test, 2001

Solution:
It is clear that it suffices to prove the following inequalities

i- y7 °2^2 - n3b y7n^3


78 Solutions

and
(5>‘)
The first, one follows from Schur’s Inequality .

^2 fl4 + abc 57a - 57 + 57 ab3


and the fact that
y~' a2b2 > abcy^a.
The second one is a simple consequence of the Cauchy-Schwarz Inequality:
(a3b 4- b3c + c3a)2 < (a2b2 4- b2c2 + c2a2)(a4 4- b4 4- c4)

(ab3.4- be3 4- ca3)2 < (a2b2 + b2c2 4- c2a2)(o4 4- b4 4- c4).

80. [ Gabriel Dospinescu, Mircea Lascu ] For a given n > 2 find the smallest
constant'A",, with the property: if ai, .>. .a,, > 0 have product 1, then
________ Uia2________ ________ Q2Q3__________ _________________ QnQl
■F < fcn-
(a2 4-a2)(a2 4-aj (a2 4-a3)(a?t 4-a2) (a2 4- ai )(a? 4- an)

Solution.'.
Let ns take first aj = a2 = • • • = a„-i = x,a„ = ’n^er ^at

2x2n~1 n-2 n-2


kn > (xnl 1 + l)(x2n' 1 + 1) •F (1 + l)2 > (1 + x)2
(1 4- x)2
for all x > 0. Clearly, this implies k„ > n-2. Let us prove that n - 2 is a good
constant and the problem will be solved.
First, we will prove that (x2 4- y)(y2 4- x) > xy( 1 4- x)(l + y). Indeed, this is the
same as (x F y)(x - y)2 > 0. So, it suffices to prove that
■: i ’ i
1
—------------------4+
----------------------- + ••• + (l-|-an)(l + ai)
<n-2.
(1 4- ai)(l 4- n2) (1 4-a2)(l 4-aa)
Now, we take ai = an = — and the above inequality becomes
x2 ii
n

’ .
E
k=l
1-
________ Xk+lXk+2._______
(xk 4-xfc+i)(ifc+i 4- x*+2)
>2,

which can be also written in the following from

V' Xk 4.______ _____ >2.


“Xfc-Fifc+i (xfc 4-ifc+i)(xfc+i 4-ifc+2)
Clearly,
n n
Xk >y------ it------ 1.
Xk + XkI 1 “ Xl + + ■ ■ ■ + Zn
K= !
Old and New Inequalities 79

So, we have to prove the inequality


« ~2
y-'_________ Jfc+i______■ ■
> I-
(ijt 4-Xfe+i)(xfc+i 4-Zfe+2)
Using the Cauchy-Schwarz Inequality, we infer that
’ n 2

X2k+1___________
Vn-' __________ -r
> -7T—--------- J------------------------
(Xk 4- Xfc+i)(xfc+i 4- Xk+2)
+Xk+i)(xjt+i+xk+2)
k=l
and so it suffices to prove that
2 n n n
{t-k k=1
h2
k—l
XkXk+l I'
k=l
XkXk+2’

But this one is equivalent to


n n
2 hXj > 2 ^2 XfcZfc-H + x^Xk+2

and it is clear. Thus, fc„ = n — 2.

81, ( Vasile Cirtoaje ] For any real numbers a,b,c,x,y, z prove that the inequality
holds
t________________________ 2
ax 4- by 4- cz 4- \/(a2 4- b2 4- c2)(x2 4- y2 4- c2) > - (a 4- b 4- c) (x 4- y + s).
o
Kvant, 1989

Solution:
Let us denote t = / n- t • Using the substitutions x — tp, y — tq and
a2 4- b2 4- c2
z =' trwhich imply
a2 + b2.4- c2 — p2 + q2 4- r2.

The given inequality becomes


2
ap 4- bq 4- cr + a2 4- b2 4- c2. > - (a 4- b 4- c) (p 4- q 4- r),
o
(g 4- p)2 4- (b 4- q)2 4- (c 4- r)2 > ^(a 4- b 4- c)(p 4- q 4- r).
o
Since
4(a + b 4- c)(p 4- q + r) < [(a + b + c) + (p 4- q + r)]2,

it suffices to prove that


(a + p)2 4- (b 4- q)2 + (c + r)2 > h(a 4- p) 4- (b 4- q) 4- (c 4- r)]2.
o
This inequality is clearly true.
HO Solutions

82. [ Vasile Cirtoaje) Prove that the sides a, b, c of a triangle satisfy the inequality
a b c „ fb c a
3 - + - + - - 1 >2
oca \a b c

First solution:
Q "F c
We may assume that c is the smallest among a,b,c. Then let x = b------ —. After
some computations, the inequality becomes
(3a-2c)x2+ (x + c - (a-c)2 >0 « (3a-2c)(2b-a-c)2+(4b+2c-3a)(a-c)2 > 0

which follows immediately from 3a > 2c, 4b 4- 2c - 3a = 3(b 4- c - a) 4- b — c > 0.

Second solution: '


Make the classical substitution a = y + z,b = z + x,c = x + y and clear denomi­
nators. The problem reduces to proving that '

X3 + y3 + z3 + 2(i2j/ + y2z + z2x) > 3(xy2 + yz2 + zx2).

We can of course assume that x is the smallest among x, y, z. Then we can write
y = x + m, z = x + n with nonnegatives m and n. A short computation shows that
the inequality reduces to 2x(m2 - mn + n2) + m3 + n3 + 2m2n - 3n2ni > 0. All we
need to prove is that in3 I- n3 f- 2rn2n > 3n2m m)3 - (n - in)m2 -I- m3 > 0
and this follows immediately from the inequality t3 + 1 > 3, true for. t > —1.

83. [ Walther Janous ] Let n > 2 and let zi,X2,...,xn > 0 add up to 1. Prove
hat

Crux Mathematicorum

First solution:
The most natural idea is to use the fact that
n- Xj _ , , n- 1
= 1+
1-Xj X1 + I2 + • ■ ‘ + Xi-l + li-j-l + • • •+,Xn

Thus, we have
________ 1__________
" ~^Xil2-.-Xi-lXi+l ... Xn

and we have to prove the inequality

/ fiH) W+ ______ 1.________ _


—yX1X2...Xi-iXi+i ...Xn
Old and New Inequalities

But this one is not


very hard, because it follows immediately by multiplying the
inequalities

n(^)>
' X3 J
r+
n-1

obtained from H’uygens Inequality.

Second solution:
We will prove even more, that

...V W \ n ) 111-,,'
It is clear that this inequality is stronger than the initial one. First, let us prove that

tt 1 I x, n-1 1 \n
■I, 1 ~ > n - 1)

This follows from Jensen’s Inequality for the convex function /(x) = ln(l + x) -
ln(l - x). So, it suffices to prove that
n
n+ 1
n
n- 1 n2 - 1
•Jp! -X,)2 >
ri*< n

But a quick look shows that this is exactly the inequality proved in the solution of
the problem 121. .

84. [ Vasile Cirtoaje, Ghcorghe Eckstein ] Consider positive, real numbers


ii,X2,...,in such that Xii2—Xn = !• Prove that
1 i -
+ n - 1 + xn
< 1.
n - 1 + ii n — 1 +12
TST 1999, Romania

First solution:
Suppose the inequality is false for a certain system of n numbers. Then we can
find a number k > 1 and n numbers which add up to 1, let them be a,, such that
------ i------ = kaA. Then we have
n - 1 T Xi

1 -n(£-+>)•
82 Solutions

We have used here the fact that a, < ——-. Now, we write 1 - (n - l)a*,. = bjt and
n— 1
n n
we find that ^b* = 1 and also JJ(1 - b*) < (n - l)n6i.... bn. But this contradicts
Jt=l lc=l
the fact that for each j we have
1 - bj = bi + • + bj-i + bj+i + • • • + b„ > (n — 1) ”"\/bi... bj-ibj+i... bn.

Second solution-.
Let us write the inequality in the form
ii
■ +, 12 , , in
-f- ----------------------- T“ . . . “T -----------------------
n - 1 + ii n -1 + 12 n - 1 + in
>1.

This inequality follows by summing the following inequalities


1“ n
___________ I] ___________ In In~"
n - 1 -I- ii
> i-J- ’
,1-i + X2’“ + •••+ ln~"
X]"" n-l + Xn + l2~" + ... + Xn "
The first from these inequalities is equivalent to

xi~"" + i2
ij 4- In
+ ... + xn-" > (n - l)i! "
and follows from the AM-GM Inequality.

Remark. . '
Replacing the numbers Xi,i2>■ • • »in with —,—, .., — respectively, the in-
Il 12 in
uality becomes as follows
1 1 1
1 + (n - 1)^! + 1 + (n - l)x2 + ...+ l + (n- l)xn
>1.

85. [ Titu Andreescu ] Prove that for any nonnegatiye real numbers a,b,c such
that a2 + b2 + c2 + abc = 4 we have 0 < ab + be + co. - abc < 2.
USAMO, 2001

First solution (by Richard Stony)’.


The lower bound is not difficult. Indeed, we have ab + bc + ca > 3\/a2b2c2 and
thus it is enough to prove that abc < SS/a’^c2, which follows from the fact that
abc < 4.: The upper bound instead is hard. Let us first observe that there are two
numbers among the three ones, which are both greater or equal than 1 or smaller
than or equal to 1. Let them be b and c. Then wc have
4 > 26c + a2 + abc => (2 - a)(2 + a) > bc(2 + a) => be < 2 - a.

Thus, ab + bc + ca -abc < ab+2-a+ac-abc and it is enough to prove the inequality


ab + 2 - a + ac - abc < 2 <=> b + c - be < !<=>(&— l)(c - 1) > 0, which is true due to
our choice.
Old and New IiicqiiAlltiefi 83

Second solution:
We won’t prove again the lower part, since this is an easy problem. Let us con­
centrate on the upper bound. Let a > b > c and let a = x 4- y, b = x — y. The
hypothesis becomes x2(2 4- c) 4- y2(2 - c) = 4 — c2 and we have to prove that
2 | q
(x2 - y2)(l c) < 2(1 - xc). Since y2 = 2 + c - x2, the problem asks to
4x2 — (4_ 1
prove the inequality -------- --------- -(1 - c) < 2(1 - cz). Of course, we have c < 1
2—c
and 0 < y2 = 2 + c - |5^z2 =>z2<2 — c=>z< \/2 - c (we have used
the fact that a > b => y > 0 and 6>0=^z>y>0). Now, consider the
4z2 — (4 ~
function f : [0, y/2 - c] -» /?, f(x) = 2(1 - ex)--------- ----------- -(1 - c). We have
1 ““ c
/'(?) = -2c— 8x • - ---- < 0 and thus f is decreasing and f{x) > /(v2 — c). So, we
2 “* c .
have to prove that
f(y/2 - c} > 0 2(l-cv/2^e) > (2-n)(l-r) <=> 3 > c4-2\/2 - r. « (l-v/2^7)2 > 0

clearly true. Thus, the problem is solved.

86. [ Titu Andreescu ] Prove that for any positive real numbers a, 6, c the following
inequality holds
a 4- b 4- c
- tyabc < max{(y/a - y/b)2, (v^b - \/c)2, (\/c - y/a)2}. ■
3
TST 2000, USA

Solution:
A natural idea would be to assume the contrary, which means that
a + b + C^^c<a + b-2^b ■
3
a 4- b 4- c — yfabc <b+c — 2yfbc.
3
a 4- b + c
— y/abc < c + a — 2y/ca.
3
Adding these inequalities, we find that
a 4- b 4- c - 3v/abc > 2(a + b 4- c - y/ab - y/bc - y/ca).
Now, we will prove that a + b 4- c — 3 y/abc < 2(a + b + c — y/ab — y/bc — y/ca) and the
problem will be solved. Since the above inequality is homogeneous, we may assume
that abc = 1. Then, it becomes 2y/ab 4- 2y/bc 4- 2 y/ca — a — b — c<3. Now, using
Schur’s inequality, we find that for any positive reals x,y,z we have:

2xy 4- 2yz + 2zx - x2 - y2 - z2 < % z - 3 y/x2y2z2

All we have to do is take x = y/a, y = Vb, z = y/c in the above inequality.


84 Solutions

87. [ Kiran Kedlaya ] Let a,b,c be positive real numbers. Prove that
a + Vab + \/abc ' a+b a+b+c i
3 “ < a’ —

Solution (by Anh Cuong):


We have that

a + '/ab + x/abc < a + \ ab^a—+—6 + s/a6c.


2
Now wc will prove that
' a+b a +b+c
a + \ ab- ~ + ■Yabc < a'~ 3
V 2 ” V
By the AM-GM Inequality, we have
, 2a 3a
3a 1+ ----- 7 +
3. 1--^- < a I- 6 a + 6+ c
a I b a I b I c 3
36
36 2+
1-1- <---- a + 6-i- c
a+b+ c 3
26 3c
%b 3c + a -H 6 a+ 6+ c
V a + 6a + 6 + c“ 3
Now, just add them up and we have the desired inequality. The equality occurs when
a = 6 — c.

88. Find the greatest constant k such that for any positive integer n which is not
a square, |(1 + %/n)sin(7Tv/n)| > k. -
Vietnamese IMO Training Camp, 1995
I
Solution:.
We will prove that - is the best- constant. .We must clearly have k <
(1 + Vi2 + 1) | sin (ttx/i2 + 1) | for all positive integers t. Because | sin (nVi2 + 1) | =
sin------ 7 , we deduce that 7 > sin ——% , from
i + v/PTT i+ " i+VP + l l + x/PTT
7T
where it follows that k < -. Now, let us prove that this constant is good. Clearly, the
inequality can be written
tr
sin (tt • {v/n}) >
2(1 + y/n)
We have two cases
Old And New Inequalities 85

i) The first case is when {x/n} < . Of course,

1
{>/n} > x/n - x/n - 1 = x/n i/n — 1 ’

x3
and because sinx > x - — we find that
6

it
siTi(n'{v/n}) > sin >
x/n- 1 -i- x/n 6 x/n - 1 + s/n

Let us prove that the last quantity is at least . This comes down to
2(1 I- x/n)

or 6(\/n + x/n - I)2 + 3(x/n + n - 1) > ^2(1 + y/n) and it is clear.

ii) The second case is when {x/H} > Let x = 1 - {\/n} < - and let n =

k2 + p, 1 < p < 2A:. Because {x/n} > - => p > k + 1. Then it is easy to see that
i >
---------- \ „ and so it suffices to prove that
A- + 1 + x/k^ + 2A-
. 7T
sm------------ . — >
A- + 1 + x/k2 + 2A- 2(1 + ZA-2 + k)
x.:i
Using again the inequality sinx > x —— we infer that,
6
JT2
2x/k2Tk - x/k2 -h 2k - A + 1
> “ 2 •
1 + v/PTA? 3 (1 + A- + x/k2 t- 2A-)

But from the Cauchy-Schwarz Inequality we have 2x/k2 + k - x/k2 + 2k - k > 0.


____ 2 7T2 /
\/k2 I A-) holds, this case is
Because the inequality (1 I k -I x/k2 I 2A-) >
t(1 1
also solved.

89. ] Dung Tran Nam ] Let x, y, z > 0 such that (i + y + z)3 = 32it/z. Find the
. . z4 I / I z1
minimum and maximum of ------------ rr.
(x + y+ z)4
Vietnam, 2004
First solution (by Tran Nam Dung)'.
We may of course assume that x + y + z = 4 and xyz = 2. Thus, we have to find the
8G Solutions

x4 I y4 I 2-1 M
extremal values of ----- . Now, we have
4'
x4 I y4 | z4 — (x2 I- y2 I- z2)2 - 2 x2y2 -
= (16 - 2^xy)2 - 2(^xy)2 + 4xyz(x 4 y 4- z) =
= 2a2-64a 4 288,
2 8
where a = xy+yz + zx. Because y+z = 4 — x and yz = -, we must have (4 -x)2 >
x x
which implies that 3 - \/5 < x < 2. Due to symmetry, we have x,y,z 6 [3 - y/b, 2].
This means that (x - 2)(j/ - 2)(z - 2) < 0 and also
(x - 3 4 y/b)(y -34 Vb)(z -34 x/5) > 0.

Clearing paranthesis, we deduce that


5^/5- 1
a G 5.
2

i4 4 / 4 z4 (a - 16)2 - 112
But because , we find that the extremal values of the
4* 128
383 - 165x/5 9 / l + v/5 l + v/5\
expression are ’ PR ’ a*'ta’ne^ r°r triples I 3 - y5, 2’2/’
256
respectively (2,1,1).

Second solution:
As in the above solution, we must find the extremal values of x2 4 y2 4 z2 when
x 4 y 4- z = l,xi/z = —, because after that the extremal values of the expression
x4 + y4 + z4 can be immediately found. Let us make the substitution x = -,y =
b c , . , , , , a2 4 b2 4 4c2
-, z = -, where abc = 1, a 4 o 4 2c = 4. Then z + f + r =------- - —— and so we
4 2 16
2
must find the extremal values of a2 4 b2 4- 4c2. Now, a2 4 b2 4 4c2 = (4 - 2c)2----- h 4c2
]C

and the problem reduces to finding the maximum and minimum of 4c2-8c— where
c
there are positive numbers a,b,c such that abc = l,a4 b4 2c = 4. Of course, this
4 3- • 5
comes down to (4 - 2c)2 > -, or to c G 1 . But this reduces to the study
c 2

3 of rhe function /(x) = 4x2 - 8x - - defined for , 1 , which is an easy task.


i - X

90. [ George Tsintifas ] Prove that for any a,b,c, d > 0,

(a + b)3(b 4 c)3(c + d)3(d + a)3 > 16a2b2c2d2(a + b 4 c + d)4.

Crux Mathematicorum
Old and New Inequalities 87

Solution:
Let us apply Mac-Laurin Inequality for

x = abc, y = bed, z = eda, t = dab.


We will find that
3
abc • bed • eda a2b2c2d2 a
I 4 > 4 4

Thus, it is enough to prove the stronger inequality

(a + b)(b + c)(c 4- d)(d 4- a) > (a + b 4- c + d)(abc + bed 4- eda ~ dab).


Now, let us observe that
(a I- b)(b |- c)(c I d)(d I a) •• (ac \ bd \ ad I bc)(ac I bd I ab t cd)
= (ac 4- bd)2 4- 52 u?(bc 4- bd 4- cd) > 4 abed 4- 52 (‘2(bc + bd + cd) =
= (a 4- b 4- c 4- d)(abc 4- bed 4- eda 4- dab).
And so the problem is solved.

91. | Titu Andrccscu, Gabriel Dospincscu | Find the maximum value of the ex­
pression
, (be)’1 . (car
1 — ab 1 - be. 1 - ca
where a, b, c are nonnegative real numbers which add up to 1 and n is some positive
integer.

Solution:
First, we will treat the case n > 1. We will prove that the maximum value is
- 4n-1. It is clear that ab, be, ca < - and so

(ab)n (bc)n (ca)n 4 .. , ,, . Vl.


1 — ab
+ 1 — be
+ 1 — ca 3

Thus, we have to prove that (ab)n 4- (be)" + (cu)n < Let a the maximum among
a,b,c. Then we have

~ > an(1 - a)n = an(b 4- c)n > anbn 4- ancn 4- nanb' *c > anbn + bncn + cnan.

So, we have proved that in this case the maximum is at most -—;—- . Bui for
I 3 • 4n 1
a = b = -,c = 0 this value is attained and this shows that the maximum value is
* • *1 . .
3~ .n_t for n > 1. Now, suppose that n = 1. In this case we have

ab
I-ab = £ 1 -1 ab -3.
SX Solutions

Using the fact that a + 6 + c= l, it is not difficult to prove that

1 1 1 3-2 ab + abc
1 - ab + 1 - 6c + "1----
- ca 1- “6 + a6c - a2b2c2
ab 3
Wc will prove that < -. With the above observations, this reduces to
1 - ab
3 -27(a6c)2 + 19a6c
Z^a - 11
But using Schur’s Inequality we infer that
v—v , 1 + 9a6c
- 4
and so it is enough to show that
9a6c + 13- 27(a6c)2 + 19a6c
<=> 108(a6c)2 + 23a6c < 1,
4 <
~ 11 '

which is true because abc <


lienee for n •- 1, the maximum value is | attained for a b - c.
o
I 92. Lei o,6, c be positive real numbers. Prove that
1
a(l I 6)
+■ 1
6(1 I c) + a) “
c(l II a)
+ c(l >
1 3
4 l/abc)'

Solution:
The following observation is crucial
1 + abc + a + ab
1 • 1 ■ 1

(1 4- abc)
a(l + b)
+ 6(1 + c)
+ l(iT^) + 3 - £ a(l + 6)
* 6(c + 1)
- E a(l1 ++a6) +! 1+6 ’
i We use now twice the AM-GM Inequality to find that
£L+^ + y^£±J)> +3^
a(l + 6) 1+6 y]abc
And so we are left with the inequality
3
—= + 3v/a6c-3
yabc> 3
1 + abc v/a6c(l + x/abc) ’
which is in fact an identity!
93. [ Dung Tran Nam ) Prove that for any real numbers a, 6, c such that a2 + 62 +
c2 = 9,
2(a + b + c) - abc < 10.
Vietnam, 2002
Old and New Inequalities 89

First solution (by Gheorghe Eckstein):


Because max{a,6, c} < 3 and |a6c| < 10, it is enough to consider only the cases
when a, b,c > 0 or exactly of the three numbers is negative. First, we will suppose
that a, 6,c are nonnegative. If abc > 1, then we are done, because

2(a + 6 + c) - abc < 2-/3(a2 + 62 + c2) - 1 < 10.

Otherwise, wc may assume that a < 1. In this case wc have


62 + c2
2(a + b I- c) — aba < 2 I a I 2 = 2a I 2\/18 - 2a2 < 10
2
Now, assume that not all three numbers are nonnegative and let c < 0.
Thus, the problem reduces to proving that for any nonncgalive x.y,z whose
sum of squares is 9 wc have 4(x I y - z) r 2ryz < 20. But we can write this as
(x-2)2 + (y-2)2 + (z-1 )2 > 2xyz — 6z — 2. Because 2xyz-6~—2 < x(y2+c2)-G:-2 =
-23 + 3z - 2 = — (z - l)2(z + 2) < 0, the inequality follows.

Second solution:
Of course, we have |a|, |6|, |c| < 3 and |a + b + c|,|a6c| < 3>/3. Also, we may
assume of course that a,b, c are non-zero and that a < b < c. If c<0 then we have
2(a + 6+c)-abc < -abc < 3\/3 < 10. Also, ifa<6<0<c then we have 2(a+6+c) <
2c < 6 < 10 + abc because abc > 0. If a < 0 < b < c, using the Cauchy-Schwarz
Inequality we find that 26 + 2c- a < 9. Thus, 2(a + 6 + c) = 26 + 2c —a + 3a < 9 +3a
and it remains to prove that 3a — 1 < abc. But a < 0 and 26c < 9 - a2, so that it
remains to show that —° ■ > 3a - 1 <=> (a + 1 )2(a - 2) < 0, which follows. So. wc
just have to threat the case 0 < a < 6 < c. In this case we have 26 + 2c + a < 9 and
2(a + 6 + c) < 9 + a. So, we need to prove that a < 1 + abc. This is clear if a < 1 and
if a > 1 we have 6, c > 1 and the inequality is again. Thus, the problem is solved.

94. [ Vasile Cirtoaje ] Let a,b,c be positive real numbers. Prove that
1
6- 1) (t+ c
•(.“4- 1-! 6 + - -1
c I-1 1
6

First solution:
With the notations x = a + - — l,u = 6 +---- 1, z = c +------ 1, the inequality
, b c a
becomes
xy + yz + zx > 3.
We consider without loss of generality that x = max{z, y, z). Rom

(I+l)(y+l)(z+l) = afec+-7-+a+fe+c+- + T + - > 2+a+6+c+- + | + - — 5+z+y+c,


abc a b c ab c-
90 Solutions

we get
xyz 4- xy 4- yz 4- zx > 4,
1
with equality if and only if abc = 1. Because y+z = -+b+
(c-iyI-2 > 0 we distinguish
a c
two cases a) x> 0, yz < 0; b)x> 0,y> 0,z> 0.
a) x> 0, yz < 0. We have xyz < 0 and from xyz 4- xy + yz 4- zx > 4 we get •
xy + yz 4- zx > 4 > 3.
b) x> 0,y> 0,z> 0. We denote xy + yz + zx — 3d2 with d > 0. From the mean
inequalities we have
xy 4- yz 4- zx > 3 \/x2y2z2,
from which we deduce that xyz < d3. On the basis of this result, from the inequality
xyz 4- xy 4- yz 4- zx > 4, we obtain d3 4- 3d2 > 4, (d - l)(d 4- 2)2 > 0, d > 1 so
xy l- yz I- zx > 3. With this the given inequality is proved. We have equality in the
case a = b = c = 1.

Second solution:
Let u = i41,i/ = y4 1,w = z 4- 1. Then we have
uvw — u 4- v 4- w 4- abc 4- -J- >u4ti + w+2.
abc
Now, consider the function f(t) = 2t3+t2(u+v+w)-uvw. Because lini fit) — oo and
l—oc
/(I) < 0, we can find a real number r > 1 such that /(r) = 0. Consider the numbers
UVW
m = -,n = -,p = —. They verify mnp = m4-n4-p4-2we deduce from problem 49
r r r
that mn + np + pm > 2(m 4- n 4- p) => uv 4- vw 4- wu > 2r(u 4- v 4- w) > 2(u 4- v 4- w).
But because u = if l,v = y 4- l,w = z 4- 1, this last relation is equivalent to
xy 4- yz 4- zx > 3, which is what we wanted.

95. [ Gabriel Dospinescu ] Let n be an integer greater than 2. Find the greatest
real number mn and the least real number Mn such that for any positive real numbers
Xl,X2,...,Xn (with Xn = 2>0>Zn-rl = Zl)>

n
___________ Xi____________
mn < M,
Xi-i 4- 2(n - l)zj 4-Xi+i

Solution:
1
We will prove that mn = > A-fn - First, let us see that the inequality
2(n - 1)
n
__________ Xj_________ ____ 1
~ Xi-i 4- 2(n - l)Ti 4- .Ti-rt
> 2(n-l)
Old and New Inequalities 91

n
is trivial, because x,-! + 2(n - l)ij + < 2(n - 1) • y x* for all i. This shows
*=i

that mn > —------—. Taking Xi — x*, the expression becomes


2(n - 1)
________ 1________ (n - 2)x xn~1
x + x”"1 +2(n-l)
+ 1 + 2(n- l)x + x2 + 1 + 2(n - l)xn-' + xn"2

1
and taking the limit when x approaches 0, we find that mn < and thus
2(n-l)
1
77ln —
2(n-l)-
Now, we will prove that Mn > -. Of course, it suffices to prove that for any
x,, X2,..., xn > 0 we have
H
___________ Xj_______________
x,_i + 2(n - l)x, + Xj-i

But it is clear that


n

£ Xj-i + 2(n 2x,


- l)x, + x,. i "
-- :___
- < y 2^*1 --______
i x,, i + 2(n - l)x,

n
1________
-£ ----
»=l n — 1 +
y/x\ I ■ -Ti I 1

Xi
n
y/Xj-! -1,4.1
Taking a,, we have co prove that if a. 1 then
Xi
" • 1
y —-—
7^1 n — 1 + ai
< 1. But this has already been proved in the problem 84. Thus,

■A/n > and because for Xi = X2 = • . = rn we have equality, we deduce that


Mn — which solves the problem.

96. [ Vasile Cirtoaje ] If x, y, z are positive real numbers, then


. 1
+
1
y2 + yz + z2
+ __ 1__
z2 + zx + z2
> __ —?
> (x 4- y1+ z)2
x2 + xy + y2
Gazcta Matematica

Solution:
Considering the relation

x2 + xy + y2 = (x + y + z)'2 - (xy + yz + zx) - (x + y + z)z,


92 Solutions

we gel
(x + y + z)2 1
x2 + xy + y2 xy + yz + ~zx z
1-
(x + y + z)2 x+y+z
or
(j + y + z)2 _________ 1________
x2 + xy + y2 1 - (ab + be + ca) - c ’
x y %
where a = b = ----------- , c = ----------- . The inequality can be rewritten
x + y + z' x+y+ z x+y+z
as
___ 1___ 1 1 >
1— d — c^l-d-b^l— d —a ~ ’
where a, b, c are positive reals with a + b + c = 1 and d = ab + be + ca. After making
some computations the inequality becomes
9da - (id2 - 3d I 1 I- Oabr. > 0
or
d(3d — 1)2 + (1 -4d + 9a6c) > 0.
which is Schur’s Inequality.

97. [ Vasilc Cirloajc ] For any a, b,c,d> 0 prove that


2(a3 + l)(b3 + l)(c3 + l)(d3 + 1) > (1 + abcd)(l + «2)0 + &2)(l + <=2)(1 + d2).
Gazeta MatematicS

Solution".
Using Huygens Inequality
JJ(1 + a4) > (1 + abed)4,
we notice that it is enough to show that that

24 n^3 + - IB1 + + a2)4>


Of course, it suffices to prove that 2(a3 + l)4 > (a4 + l)(a2 + I)4 for any positive
real a. But (a2 + l)4 < (a+l)2(a3 + l)2 and we are left with the inequality 2(a3+l)2 >
I
(a 4- I)2(a4 + !)<=> 2(a2 - a + I)2 > a4 + 1 (a - I)4 > 0, which follows.

98. Prove that for any real numbers a, b, c,

(a + ft)4 + (6 + c)4 + (c + a)4 > y(a4 + b4 + c4).


Vietnam TST, 1996
Old and New Inequalities •13

Solution:
Let us make the substitution a + b = 2z, b + c = 2x. c + a — 2y. The inequality
becomes 52(1/ + z ~ x)4 < 28 52 x4. Now, we have the following chain of identities
2
2zz)2 = 3(52x2) +4

-yz)2 =3(52x2)2-4(52xy) (52®2) +


=4 12) +ie52*v,2- (521)4 -2852i4
because (52 x2) < 3 52j4, 52l21/2 < 52

99. Prove that if a, b, c are positive real numbers such that abc = 1, then
1 1 1 1 1 1
1+a+b+
+ -11— -------
+ bb +
+ f-
+ cc + 11 + + aa ~ 2 + a
+ cc + 2+b 2 + c’
I
Bulgaria, 1997
| '

Solution:
Let x = a + b + c and y — ab + be + ca. Using brute-force, it is easy to see that
the left hand side is ~—1 J~T 1 1 8 , while the right hand side is 1 *2 Now,
x2 I- 2x + y + xy 9-I 4x I-2y
the inequality becomes
x-2 + 4a: + y + 3 < 12 +
-I- 4x + y 2a~ + 3 - xy < 3-y
<
< 9 + 4x + 2y - 1 «- x2 + 2x + y + xy ~
-1 ~
x2 + 2x + y + xy 9 + 4x + 2y
For the last inequality, we clear denominators. Then using the inequalities x > 3,y >
3, x2 > 3y, we have . .

^x2y > 5x2, > y2,xy2 > 9x, 5xy > 15x, xy > 3y and x2y > 27.

Summing up these inequalities, the desired inequality follows.

1 2 3
' 100. [ Duog Tran Nam ] Find the minimum value of the expression - + - + -
where a, b, c are positive real numbers such that 21ab + 2bc + 8ca < 12.
Vietnam, 2001

First solution (by Dung Tran Nam):


12 3
Let - - x, - - y, - — z. Then it is easy to check that the condition of the problem
a b c
becomes 2xyz > 2x + 4y + Iz. And we need to minimize x + y + z. But

2xy > 7
z(2xy - 7) > 2x + 4y => < 2x + 4y
2 > ------------
“ 2xy - 7
94 Solutions

Now, we transform the expression so that after one application of the AM-GM
2x + Ay
Inequality the numerator 3.ry - 7 should vanish .r + y + z>z + y +
2ry - 7

11 7 2.t I — 11 7
x I’ I- y ~ z- 4'
____ 1 -I- But, it is immediate to prove
2.t y 2r 2ry - 7 2x
3 1 . 3 . 9 15 ... .
that 2 > ---- - ---- iiuu so x
and au y iI 4z c.
j, ■!I y > -n >I x -I< — > —•. We have equality for
~ C.
1 2
Z x
X Z2
5 ......................................................... 1515 . . .r
x “ 3.y - z = 2. Therefore, in the initial problem the answer is —, achieved for
2
1,-1 3
«= -,o = -,c - -.
3 5 2

Second solution-.
We use the same substitution and reduce the problem to finding the minimum
value of x + y + z when 2xyz > 2x + Ay 4- 7z. Applying the weighted the AM-GM
Inequality we find that

x + y -F z >

And also 2x + 4y -1- 7z > 10* • 12i • 5n • zs • y3 • z^. This means that (z+ .
225
'•y + z)2(2z + Ay + 7z) > —xyz. Because 2xyz > 2x + Ay + 7z, we will have

. + y + z),22> 225
(z 15
— =>x + y + z>y
5
with equality for x = 3, y = -,z = 2.

101. | Titu Andreescu, Gabriel Dospinescu ] Prove that for any x, y, z,a,b,c> 0
such (hat xy + yz + zx = 3,

a . , b . . c , , „
(y I- z) 4 —— (z i x) f- ——(x I y) > 3.
o+c c+a o+a

I
Solution:
We will prove the inequality

-^-(y + z) + -~-(z + x) + -^t(x + y) > >/3(xy + yz + zx)


u "f c c ~r u u t u

for any a, b, c, x, y, z. Because the inequality is homogeneous in x, y, z we can assume


that x + y + z = 1. But then we can apply the Cauchy-Schwarz Inequality so that
Old tuid New IiicquiditiiM •r.

to obtain
a b c /
;----- x
b+c
+ —;—y +-----rz + V 3(xy + yz + zx) <
c -F a. a+b
a
<
b+c I>2 + xy <

"7 ,___________ I 2 3
s.tE + 2

2
Thus,'we arc left with the inequality y ^b ) x < , a • But this one
+ 2 ~ b -F c
ab
is equivalent to y* ------- which is trivial.
a)(c + b) ~ 4
(c + o)(c
Remark.
A stronger inequality is the following:

b¥c^y + + + s) + + - 52 x/(x + v)(l + 2) - (x + y + z),


which may be obtained by applying the Cauchy-Schwarz Inequality, as follows
! -±-(j + x) , -^-(x ,

(o+t+c)f£±i + £+5 + £i£ - 2(x F y 4- z) > (x/y+z + Vz + x + x/x + y)2-


\ b -F c c Fa a -F b
2(x + y + z) = 5^ \/(x + y)(x -F z) - (x + y F z).
A good exercise for readers is to show that
' ^2 /(x I !/)(x + z) x *• y 2 1 \/3(xy I yz F zr).

102. Let a,b, c be positive real numbers. Prove that


(a + b - c),2
(b 4- c - a)2 (c + a - b)2
(b + c)2 + a2 + (c
(c + a)22 +
+ n)
+
+ bb22 + (a + b)2 + c2 4
Japan, 1997

First solution'.
r b+c c+a a+b
Let x = -- ,y = —-—, z = —-—. The inequality can be written

(x - I)2 3
x2 + 1 “ 5‘
Using the Cauchy-Schwarz Inequality, we find that
(x-i)2 (r + i/ + z - 3)2
i2 +1 > x2 + y2 + z2 + 3

ffflTI
96 Solutions

and so it is enough to prove that


(t + y 4- z - 3)2
> j «■ (Ex)2-15£x + 3£xi/+18>0.
x2 + y2 + z2 -I- 3
But from Schur’s Inequality, after some computations, we deduce that
7 xy > 2 y x. Thus, we have

fy* x)' - 15 y r + 3 y xy + 18 > (y x)' -9 y x + 18 > 0,

the Inst. one being clearly true since y x > 6.

Second solution:
Of course, wc may take a + b + c = 2. The inequality becomes
V 4(1-ft)2 >3 _ v 1 27
^2 + 2(1-n)2 “ 5 1 + (1 -a)2 ~ 10’
But with the substitution 1— a = x, l-b = j/,l-c = z, the inequality reduces to
that from problem 47.

103. ( Vasile Cirtoaje, Gabriel Dospinescu ] Prove that if dj, a2l..., an > 0 then
fti + a2 + • • t• + <in-i \”
nn )
a" + a2 + • •' + a" - naia2... on > (n - 1)
n-1 )
where an is the least among the numbers ai, o2,..., an.

Solution:
Let a, - an = > 0 for i G {1,2,... , n - 1}. Now, let us look at
\"

Sa”-n-na> “f71-1) n-1


~an
i=l 1=1

\ /
as a polynomial in a - an. It is in fact
n-1 n-1 z n
a" 1 y (a I Ii)i*n - na JJ (a I- x,) - (n- 1) ( X) + I2 4------- 1- Xn-l

We will prove that the coefficient of ak is nonnegative for all k G {0,1,... ,n-1},
because clearly the degree of this polynomial is at most n-1. For k - 0, this follows
from the convexity of the function /(x) = xn

E«?>(n-1)
fz t=l
Xi

n-1

/
Old and New Inequalities 97

Fur A.- > 0, the coefficient of ak is

-n t.
i<n< 1 <n-1

Let us prove that this is nonnegative. Prom the AM-GM Inequality we have
n ^2 Xi,t <

n
ji - k z X«1 + xh +... k
n- 1

which is clearly smaller than . This shows that each coefficient of the

polynomial is nonnegative and so this polynomial takes nonnegative values when


restricted to nonnegative numbers.

104. [ Turkevici ] Prove that for all positive real numbers x, y, z, t,


x4 -F y4 + z4 + t4 + 2xyzt > x2y2 -I- y2z2 + z2t2 + t2x2 -F x2z2 F y2t2.
Kvant

Solution:
Clearly, it is enough to prove the inequality if xyzt = 1 and so the problem
becomes
If a, b, c, d have product 1, then a2 F b2 I c2 -F d2 -I 2 > ab + be F cd I da I ac I bd.
Let d the minimum among a,b, c, d and let m = Vahc. We will prove that
a2 + b2 -F c2 4- d2 + 2 - (ab + be + cd + da + ac F bd) > d2 -F 3m2 + 2 - (3m2 F 3md).
which is in fact
a2 + b2 + c2 — ab — be - ca >d (a+ b + c — 3 tfabc'j .

Because d < \/abc, proving this first inequality comes down to the inequality
a2 -I- b2 -I- c2 - ab - be — ca > \/abc f b I c - 3 v/abc^ .

a b c
Take u = ,v = — ,w = - Using problem 74, we find that
\/abc yabc vabc
u2 + v2 + w2 + 3 > u + v + w + uv + vtu + wu
which is exactly a2 -F b2 + c2 - ab - be - ca > \/abc (a -F b I- c - 3x/abcj. Thus, it
remains to prove that d2 + 2 > 3md <=> d2 + 2 > 3\^bP, which is clear.
9N Solutions

105. Prove that for any real numbers (ii.aj,... ,an the following inequality holds
n 2 >.
ij
*E i+j-1 ■aittj.

Solution:
Observe that
n
= [\t+i-2dt.=
E i + ijj-1 -aidj
ij=i Jn

= f ( E iai ■ | dt =
Jo /
2
ri
/•i / n

/ (E. ,a*-f
•/n
'n

Now, using the Cauchy-Schwarz Inequality for integrals, we get


2 2 / 11
2
,1 / »
dt >
Jo.. .

which ends the proof.

106. Prove that if ax,a2}...,antbi,...,bn are real numbers between 1001 and
2002, inclusively, such that a2 1 n| + • • • -I- a2 = b2 + bj + • • • + b2, then we have the
inequality
n' + a2
bT + b? + + £ - T5(a’+a-2 + '" + a’1)'
TST Singapore

Solution:
a. 1 J
The key ideas are that — e ^,2 for any i and that for all x 6 -,2 we have
bi
the inequality x2 + 1 < -x. Consequently, we have

5 Qi
2 ' bi - 1 + 2°'^ - at +

and also

I Eai6i E (a? + = 2Ea? f1)


Ohl and New Inequalities 99

a?
Now, the observation that a? and the inequality | > 1 4- p- allow us to

5 a3
write -a2 > — + atbt and adding up these inequalities yields
2 bi
n n

t=l i=l ' z i=l i=l

Using (1) and (2) we find that


a? a2 a?i I? , 2 2 2\
Vi+b2+'"+b;~W^ +“2 + -'- + “n).

which is the desired inequality.

107. ( Titu Andreescu, Gabriel Dospinescu ] Prove that if a,b,c are positive real
numbers which add up to 1, then
(a2 4- 62)(62 + c2)(c2 4- a2) > 8(a2b2 + b2c2 4- c2a2)2.

Solution:
1 i. We find the equivalent form if — 4— 4— = 1 then
Let x = -, v = b'Z =
a c r. y z
(x2 4- y2)(y2 4- z2)(z2 4- x2) > 8(x2 4- y2 4- z2)2.

We will prove the following inequality

\ •*' y
for any positive numbers x, y,z..
Write x2 4- y2 = 2c, y2 4- z2 = 2a, z2 4- x2 = 2b. Then the inequality becomes

z abc
b 4- c - a
Recall Schur’s Inequality
52 + abc(a 4- b 4- c) > 52 a‘3(b + c) ** a^c(a + b 4- c) > 52 fl3(b + c — a)-

Now, using Holder’s Inequality, we find that

___
^a3(6 + c-a) = T —
J y , 4-1c - a
2•

+c-a y-'
Combining the two inequalities, we find that
I abc
:Z b+ c— a
a

and so the inequality is proved.

i
100 Solutions

108. ( Vasile Cirloaje ) If a,b, c, d are positive real numbers such that abed = 1,
then
1 1 1 1
(14-a)2
+ (1 + b)2 + + (1 + d)2 > 1.
(1 + c)2
Gazeta Matcmatica

Solution:
If follows by summing the inequalities
1 , 1 1
(1 + a)2 1 (1 + b)2 > 1 + ab'
1 .
1
r’

(1+c)2 + + (1+d)2 > 1 + cd


The first from these inequalities follows from
1 1 1 ab(a2 + b2) - a2b2 - 2ab + 1
(1 4- a)2 + (1 1- b)2 1 4- ab (1 + a)2(l + b)2(l + c)2
‘ ab(a - b)2 + (ab -1)2 „
>0.
' • ~ (l + a)2(l + b)2(l + ab) “
Equality holds if a = b = c - d = 1.

109. ( Vasile Cirtoaje ] Let a, b, c be positive real numbers. Prove that


a2 . b2 b 4. -----
a -f. ------
c2 > —---- c _
b2 I c2 + c2 + a' a2 + b2 b + c c 4- a a I- b
Gazeta Matcmatica

Solution:
We have the following identities
a2 a nb(a - b) + ac(a - c)
f^ + c2 b+c (b + c)(b2 + c2)
b2 b bc(b - c) I ab(b - a)
c2 + a2 c+a (c + a)(c2 + a2)
c2 c ac(c - a) + bc(c - b)
a2 + b2 a+b (b + a)(b2 + a2)
Thus, we have
a \ ab(a — b) ab(a - b)
^b2 + c2 "b+c (b + c)(b2 — c2) (a + c)(a2 + c2).
= (a2 + b2 + c2 + ab + be + ca) • Y* —---- - —&\v~2----- 2\
>0. •
•. 7 (b + c)(c +a)(b2 + c2)^ + a2)
Old find New Iiicqunlitin* 101

110. [ Gabriel Dospinescu ] Let ai, O2>. ■. be real numbers and lei S be a
non-empty subset of {1,2,... ,n}. Prove that

E* < E (a, + ... + a?)2.


TST 2004, Romania

First solution:
Denote St = Oj + • • • + ajt for k = 1, n and also sn+j = 0. Define now
1 if i € S
bi =
0 , otherwise
Using Abel’s summation we find that

E«.
. tes
= Clibi + 02^2 + ' ' • + Clnbn —

= Si(6j — 62) + 32(62 — 63) + •■■ + sn_i(6n_j - 6n) + sn6n + s„+i(-6i)

Now, put 6X - 62 = xi,62 - 63 = X2,-..16n-i - 6n = in-i,6n = in.^n+i = ~bi- So


we have ' ?
n+1

Ea< = E x*s<
itS 1=1

and also X{ e {-1,0,1}. Clearly, x, = 0. On the other hand, using Lagrange

identity we find that


n
E (a< + ■■• + a>)2
- E
l<i<j<n
~ s«)2 =
2

l<i<j<n+l

So wc need to prove that


2 'n + 1 2

z>.
n+1 /n+1
(n+!)Esi ( E5*x<
But it is clear that

2 n+1 2 n+1

Z2s<i‘ E* = Es2(1+i<)
i=l

+ 2 52 Sis/riij + 1)
l<t<j<n+l
102 Solutions

Now, using the fact that 2sjSj < s;’ + s’, 1 + ijXj > 0, we can write

2 } SiSj(liXj + 1) < (s, + Sj)(l + XjXj) =


l<i<jntl l<i<j<>iH
n (s? + ••• + s’ + 1) + s’li(l2 +13 + ••• + Xn) + ••' + S’zn(ll +
+ i2 + • • • + in-i) = -s'ix'i-------- s’x’ + n(s? + • • • + s’).

So,
'n+l 2 'n+l ' ’ ' n+l n+l ,
y + < ys2(i* + l) + ysfc(n-i*) =
k=l k=l .
n+l
= (n + 1) y s’ and we are done.
k+1
I

Second solution (by Andrei NeguL):


First, let us prove a lemma
Lemma
For any a1, <12,..., 02*+1 € R we have the inequality
’ k
2

y “24-rl < y (m + '-' + aj)2.


.4=0 l<l<j<2fcll. . .

Proof of the lemma


Let us take s* = U) + • • • + a*. We have
k
y 024+1 = 51 + S3 - S2 + • • • 4- S2fc+l - S2k
4=0
and so the left hand side in the leiiuna is
2fc + l
y «2+2 y s2i+i52j+i+2 y S24S2J “ 2 y 521+1S2J
i=l n<t<J<k !<4<)<k 0<t<*
^J<k

and the right hand side is just


2k+l
(2fc+l)£s’-2 y siSj.
' i=l l<i<j<2k+l.

Thus, we need to prove that. . .


2*+l
2^ y s’> 4 y S2i+is2;+i.+4 y S2i$2j
4=1 0<i<j<k l<i<j<k
and it comes by adding up the inequalities

*. 2$2j+lS2i+l < S21+1 + 52j + l> 2S2tS2j < s2i + S2j-


Old and New Inequalities 103

Now, let us turn back to the solution of the problem. Let us call a succession
o a> s a sequence and call a sequence that is missing from S a gap. We group the
successive sequences from S and thus S will look like this
{nu > at> 11»• • •, <1^ i , al3 f 1,..., ola f ,..., at<,..., J
where ij 4- kj < ij + 1 — 1. Thus, we write S as a sequence, followed by a gap, followed
by a sequence, then a gap and so on. Now, take Si = a,, 4- • • • + , s2 = a,, +fc) +, 4.
••• F a»3-i,...,s2r-i = air 4 - .. -I- air+kr
Then,
2

•• • (Ea*- — (S1 + S3 + • • • + S2r-1)2 <


\i€S >

E (Si + --- + ^)2 < E (ai + -.. + a>)2


l<i<j<2r-l l<t<j<n

the last inequality being clearly true, because the terms in the left hand side arc
among those from the right hand side.
Third solution:
We will prove the inequality using induction. For n = 2 and n = 3 it’s easy.
Suppose the inequality is true for all k < n and let us prove it for n. If 1 £ S, then
we just apply the inductive step for the numbers a2,...,an, because the right hand
side doesn’t decrease. Now, suppose 1 6 S. If 2 € S, then we apply the inductive step
with the numbers ai + <22, a3,..., an. Thus, we may assume that 2 £ S. It is easy to
see that (a + b + c)2 + c2 > — — > 2ab and thus we have (aj +’ a2 4-------t- a*)2 4-
(a2 4- a3 4- • • • + a*-i)2 > 2aia* (*). Also, the inductive step for 03,... ,an shows that

’ • £ < E (ai+---+aj)2-
\i€S\{l} 3<i<j<n

So, it suffices to show that

aj + 2ai 57 a» _ + • • • + Qi)2 + E (02 + • • • + a,)’


ies\{i} *=’ 1=2
Bui this is clear from the fact that, a? appears in the right hand side and by summing
up the inequalities from (*).

111. [ Dung Tran Nam ] Let xi,x2 • • • ,*2004 be real numbers in the interval [-1,1]
such that x'j -I- xa2 +... + = 0. Find the maximal value of the + x2 + • • • + X2004.

Solution: ... !
Let us take a, = x? and the function f : [-1,1] - R, /(*) = x5. We will prove
first the following properties of f:
1114 Solutions

1. f(x + y + 1) + /(-I) > /(x) + f(y) if -1 < x,y < 0.


2. f is convex on [-1,0] and concave on [0,1].
3. If x > 0 and y < 0 and x + y < 0 then f(x) + /(y) < /(-I) + f(x + y + 1) and if
x + y > 0 then f(x) + f(y) < /(x + y). The proofs of these results arc easy. Indeed,
for the first one we make the substitution x = -a3,y = -b3 and it conies down to
1 > a3+lr3+(l-a-b)3 1 > (a+b)(a2—ab+b2)+l—3(a+b)+3(a+b)2-(a+6)3 <=>
<=> 3(a + b)(l - a)(l - 6) > 0, which follows. The second statement is clear and the
third one can be easily deduced in the same manner as 1.
From these arguments we deduce that if (ti,t2> • • •, £2004) = t is the point where
the maximum value of the function g : A = {x 6 [-1, l]2004 + ■ •• + xn = 0} —»
2004
—» R,y(ii,...,x2oo4) = '2-jJ(xk) (this maximum exists because this function is

defined on a compact) is attained then we have that all positive components of t are
equal to each other and all negative ones are -1. So, suppose we have k components
equal to -1 and 2004 - k components equal to a number a. Because ti4-t24---- ^2001 =
k ' k
0 we find that a = —- and the value of g in this point is (2004-fc) y -k.
2004 - k
I fa
Thus we have to find the maximum value of (2004 — k) (7 —— — k, when k is in
the set {0,1 2004). A short analysis with derivatives shows that the maximum is
attained when k — 223 and so the maximum value is i/223 • x/l 7812 - 223.

112. [ Gabriel Dospinescu, Calin Popa | Prove that if n > 2 and aj,a2,...,an
are real numbers with product 1, then

a2 + a2 + • • ■ + a2 - n >----- - • x/n - 1 (a, + a2 + ■ • • + an - n).


71 — 1

Solution:
We will prove the inequality by induction. For n = 2 it is trivial. Now, suppose the
inequality is true for n -1 numbers and let us prove it for-n. First, it is easy to see that
it is enough to prove it for aj,...,a„ > 0 (otherwise we replace ai,a2,...,an with
|ail> lazL - • - > |QhI* which have product 1. Yet, the right hand side increases). Now,
let an the maximum number among ai,a2,...,an and let G the geometric mean of
ai,a2,... ,an_j. First, we will prove that
2n
d2 + o.2 + • + Q2 — n x/n — l(ai + 0-2 4- * • • + — n) >
n —’ 1

> a2 + (n - 1)G2 - n - • vTi- l(an + (n - 1)G - n)


n— 1
which is equivalent to

a2 + a2 + • • • + a2 - (n - 1) n~^a2lal...a2n >
Old and New Inequalities 105

2ti
> ------ - ■ Vn-1 (ai 4- a2 4- • ■ ■ 4- an-i - (n - 1) "-^0102 ... an_i).
n— 1
Because, •—. an_j < 1 and ai4-024--- -4-an-i-(n-1) "-5/0102 . -. an_i >
0, it is enough to prove the inequality

a? + -*- + an-i -(n-l)G2 > • >/n — 1 • G • (ai + • • ■ + fln-1 - (n — 1)G).

Now, we apply the inductive hypothesis for the numbers 7-,..., % 1 wliich have
G G
product 1 and we infer that
Qi + --+Qn-i 2(n-l) ai + • • • + an-i
\/n — 2 — n 4- 1
G2 n-2 G
and so it suffices to prove that
^^•"-y^2(a14---Fon_1-(n-l)<7) > 2n
• \/n - l(fl|d------- Fan-i —(«—1)^):
n— 1
1 x/n - 1 ,
which is the same as 1 4-
n(n — 2)
> — ----- This becomes
‘~x/n — 2
\ n(n-l)
1 (n — l)n-1
1 -F ) >
n(n — 2) (n-2)"
and it follows for n > 4 from
n(n-l)
. 1
>2
n(n - 2)
and
(n- l)n~r. 1
(n — 2)n “n-2
1 4-
1
n-2
For n = 3 and n = 4 it is easy to check.
H1+ 1
n-2
e j 1
n-2 1 1 4- n-2

Thus, we have proved that


Q__ .
aj 4- 02 4------- F a2 — n — —-—- • >/n — l(ai 4- 02 4- •—Fan - n) >
n — 1 . . ■ . ,

> a2 -F (n - 1)G2 - n - .• ^TTI( Qn +


tyn - l(an. 4- ((n
n_— 1)G - n)
1)G - n)
n— 1 ■
and it is enough to prove that

^(n-l) + n-1 2n n— 1
-n> • 1/n^T x_n-i 4- ----------- n
x2 n— 1 x

i). Let us consider rhe function


for all x > 1 (we took x =
G
+ n-1 2n n — 1
_n-1 + -----------
/(z) = x'.2(n-l) X
x
n
n— 1
We have
xn — 1 (n - l)(in 4- -1)
/'(z) = 2- -nv5Tl >0
x2 x .
1U6 Solutions

because
1 1 1
x + - = in-1 + (n - l)r + ■•■ +
+ (n - l)i > nl'j (n-l)n-i-
x
Thus, f is increasing and so f(x) > /(I) = 0. This proves the inequality.

113. [ Vasile Cirtoaje ] If a,b,c are positive real numbers, then


l~2a~ I 2b I~2c~ .
Va + ^^V^ + c + Vc + u~
Gazeta Matematica

First solution:
With the notations x the problem reduces to proving
that xyz = 1 implies
2 2
1 + x2 + \ 1 +1/2 + <3.

We presume that x < y < z, which implies xy < 1 and z > 1. We have
' 2 2‘ 2 ‘ l-x2y,2
2
1 + x2 + < 2 ------ 7 + =4 1 + <
1 + y2 1 + X2 1 +y2 (1 + x2)(l + y2)
1 - x2y2 8 8z
4 1+
(1 + xy)2 1 + xy z+ 1
so
■ 2 2 2z
V 1 + i2
+ 1 + y2
<2
2+1
and we need to prove chat
' 2z ^-2 <3,
2 - --- 7 +
Because
2 2
1 + z2 - 1 + z
we only need to prove that .
2z 2
2 ----- r H < 3.
z+ 1 1+z ~
2+1
This inequality is equivalent to 7
1 + 3z - 2\/2z(l+ z) > 0,(v^z - x/z+T)2 > 0,
and we are done.
Old and New Inequalities 107

Second solution:
Clearly, the problem asks to prove that if xyz = 1 then

y\P-
V x+ 1
<3.

We have two cases. The first and easy one is when xy 4- yz 4- zx > z 4- y 4- z. In this
case we can apply the Cauchy-Schwarz Inequality to get. ;.

j +. i - 3E zA-
41 •
But

E —<-
^Z41 “ 2
E 2(zy 4- z 4- y 4- 1) < 3(2 4- z 4- y 4- z 4- zy 4- yz 4- zz) <=>
<=> z 4- y 4- z < xy 4- yz 4- zz
and so in this case the inequality is proved.
The second case is when xy + yz + zx < x + y + z. Thus,
(z - l)(y - 1 )(z -1)=z4-y4-z-zy-yz-zz>0
and so exactly two of the numbers z, y, z are smaller chan 1, let them be z and y. So,
we must prove that if z and y are smaller than 1, then
2 r2 ' 2zy
----- - 4- -- 7 + < 3.
z 4- 1 y 4- 1 zy + 1
Using the Cauchy-Schwarz Inequality, we get
2 r~2~ / 2zy '1^ 1 2zy
z+1 + y+1 V xy + 1 < 2 z + 1 4- -----
y 4-1
7+ zy 4- 1
and so it is enough to prove that this last quantity is at most 3. But this comes down
to
1 2zy . ,
1-
2-
z + 1r + 7TT~‘ zy 4- 1 ■
< --- I 2zy
' 1 1
1+ 1 + < I xy 4- 1 '
z 4-1 S+1
1 1 '
Because we have 4------— > 1, the left hand side is at most
z 4-1 1/4-1 ~ ’
1 1 _i =
z 4- 1 + 1/+1 (x+ I)(y 4-1)
and so we are left with the inequality
1 - zy 1 - zy 2zy
< <=> xy + 1 + (zy + I) <
(z4- l)(y 4- 1) 2zy zy + 1
(zy + 1) I 1 +
zy 4-1
< zy4-14-z4-y<=>z4-y> \/2zy(Ty 4- 1)
which follows from i/2zy(zy 4- 1) < 2^/zy < z 4- y. The problem Is solved.
108 Solutions

114. Prove the following inequality for positive real numbers x,y,z
1 1 1
(xy + yz + zx) (x 4- y)2 +
(y + *)2
+
4- (y 4- z)2 + (z 4- r)2
Iran, 1996

First solution (by lurie Boreico)'.


With the substitution x 4- y = c, y 4- z = a, z + x — b, the inequality becomes after i
some easy computations

yf---
2--' \ab c2
(a - b)2 > 0.

Let a > b > c. If 2c2 > ab, each term in the above expression is positive and
we are done. So, let 2c2 < ab. First, we prove that 2b2 > ac, 2a2 > be. Suppose that
2b2 < ac. Then (b 4- c)2 < 2(b2 4- c2) < a(b 4- c) and so b 4- c < a, false. Clearly, we
can write the inequality like that
(— - — 2_ _ _1 1 2
(a - c)2 + (b-c)’2> (a-b)2.
\ ac b2 be a2 c2 ab
We can immediately see that the inequality (a - c)2 > (a - b)2 F (b - c)2 holds
and thus if suffices to prove that
/ 2_ _2 _ _1_ _ T 1 1 2_ _ 2_
\ ac \ be a2 b2 (b-c)2> -L--------- (a - b)2.
C2 ab ac
2
1 1 2 2 1 1
But it is clear that + ~2------k----- < and so the right hand side
c1 ab ac b c
(g-b)2(b-c)|2:
s at most Also, it is easy to see that
b2c2
2 2 _L _1 > 1 +1 >
ac + be a2 b2 b2^ ’
(°~6)2
be ac
(a - b)2(b - c)I2:
which shows that the left hand side is at least - and this ends the
solution.

Second solution:
Since the inequality is homogenous, we may assume that xy + yz + zx - 3. Also,
we make the substitution x 4- y 4- z = 3a. FYom (x 4- y + z)2 > 3(xy + yz + zx), we
get a > 1. Now we write the inequality as follows
1: 1 1 3
(3a - z)2 (3a - y)2 + (3a - z)2 “4’

4[(zy 4- 3az)2 + (yz 4- 3az)2 -I- (zx 4- 3ay)2] > 3(9a - xyz)2,
4(27a4 - 18a2 4- 3 + 4azi/’) > (9a - zys)2,
3(12a2 - l)(3a2 - 4) 4- zyz(34a - xyz) > 0, (1)
Old and New Inequalities 109

12(3a2 - l)2 + 208a2 > (17a -xyz)2. (2)


We have two cases.
i) Case 3a2 — 4 > 0. Since
34a — xyz = | [34(x + y + z)(xy + yz 4- zx) — 9xyz] > 0,

the inequality (1) is true. ' •


ii) Case 3a2 — 4 < 0. FYom Schur’s Inequality

(x + y + z)3 - 4(x + y + z)(xy + yz + zx) + 9xyz > 0,

it follows that 3a3 - 4a + xyz > 0. Thus,


12(3a2 - I)2 + 208a2 - (17a - xyz)2 > 12(3a2 - I)2 + 208a2 - a2(3a2 + 13)2 = <

= 3(4 - Ila2 + 10a4 - 3ac) = 3(1 - a2)2(4 - 3a2)2 > 0.

115. Prove that for any x,y in the interval [0,1],

\/l + x2 + \/l + y2 + \/(le— x)2 + (1 - y)2 > (1 + x/5)( 1 - xy).

Solution (by Faruk F. Abi-Khuzam and Roy Barbara - ”A sharp in­


equality and the iradius conjecture”);
Let the function F : [0,1]2 —♦ R, F(x,y) = \/l + x2 + \/l + y2 +
\/(l - x)2 + (1 - y)2 — (1 + x/5)(l - xy). It is clear that F is symmetric in x and
y and also the convexity of the function x —♦ >/l + x2 shows that F(x, 0) > 0 for all
x. Now, suppose we fix y and consider F as a function in x. It.’s derivatives are
T * 1 — X
. J'(x) = (1 .1 Vb)y I -7=y== - /. .2 .. .2
Vi2 + 1 v(l - x)2 + (1 - y)2
and
i + (i-v)2
r (x) =
x/d + x2)3 ^((1 _ x)2 + (I - y)2)3
Thus, f is convex and its derivative is increasing. Now, let
f 6^3
r= , c = 1 + \/5.
(1 + x/5)2

The first case we will discuss is y > 7. It is easy to see that in this case we have cy >
1 • .
. (the derivative of the function y —» y2c2(y2 - 2y + 2) - 1 is positive)
vy2-2y + 2
and so f'(0) > 0. Because f' is increasing, we have /'(x) > 0 and so j is increasing
with /(0) = F(0. y) = F(y, 0) > 0. Thus, in this case the inequality is proved.
110 Solutions

Due to case 1 and to symmetry, it remains to show that the inequality holds in
[ r r
the cases x e 0, - ,y e [0, r] and x 6 r i]
N’
In the first case we deduce immediately that
x 1 -x
f'(x) < r(l + \/5) +
\/l + x2 + (x - I)2

Thus, we have f < 0, which shows that / is decreasing on 0, . Because from

the first case discussed we have f >0, we will have F(x,y) = /(x) > 0 for all
points (x,y) with x € 0, i
.3/ e [0,r].

Now, let us discuss the most important case, when x e


the points 0(0,0),>1(1,0),B(l, 1),C(0,1),M(l,y),N(x, 1). The triangle OMN has
>y 6
H]-Let
oerimeter
1 - xy
\/l + x2 + \/l + y2 + \/(l - x)2 + (1 - y)2 and area
2 ‘

But it is trivial to show that in any triangle with perimeter P and area S we have the
P2 ,______ ,______ ___ ________________
’ inequality S < —y=. Thus, we find that x/1 + x2+\/l + y2 + >/(l - x)2 + (1 - y)2 >
___ 12v3
x/eTsyr - xy > (1 + x/5)(1 - xy) due to the fact that xy > r2. The proof is complete.

116. [ Suranyi ] Prove that for any positive real numbers ai,<12,...,an the fol­
lowing inequality holds

(n-l)(a"+ajd ...... f-a")+naia2 ...an > (a! + a2 + -|-Qn)(a’1’ ‘-l-a” 1 + ---+o" x).

Miklos Schweitzer Competition

Solution:
Again, we will use induction to prove this inequality, but the proof of the inductive
step will be again highly non-trivial. Indeed, suppose the inequality is true for n
numbers and let us prove it for n + 1 numbers.
Due to the symmetry and homogeneity of the inequality, it is enough to prove it
under the conditions ai > a2 > • • • > an+i and aj + <22 d------ F an = 1. We have to
prove that
H 71 / n
nf>r’ + n.a."|} + nan+i •
i=l i= l
a, + an+i • a. — (1 + an+i) >0.
\i= 1
But from the inductive hypothesis we have

(n - 1 )(aj* + a'2 + • • • + aj() + naia.2.. .an > a" + a. + •■• + <


Old and New Inequalities Ill

and so n n n

nrtn+1 IIfli - an+IEar-1 - (n- o^+iE*2"-


Using this last inequality, it remains to prove that

n E-?+1-E< — On + l +

+an+i ( °* + ~ 1)a"+i _n—1


> o.
~ an+.l

Now, we will break this inequality into


n '
an+l J] a< + (n - l)a„+l - “nil >0

and
(nE^r-E^ — an+i nE^-E<“ >0.

Let us justify these two inequalities. The first one- is pretty obvious
n n
JTJ Oj + (n — 1)qJ{+1 ~ an + l = ~' an+l + an+l) + (fl — l)on.

n
— an + l ’I'1 On„;} • E(a< - °n+i) + <n ”
n■ “ °"+i = °-
i=l
Now, let us prove the second inequality. It can be written as

n E °"+1 _ E - On+’
n n . • j
1
Because nEa? “ Eai*-1 — (us'Hg Chebyshev’s Inequality) and an+i < -, it

is enough to prove that


n
_\ A
n2_ya*

but this one follows, because na"’rl + —a"-1 > a" for all t. Thus, the inductive step
is proved.

117. Prove that for any xi,z2,.;.,x„ > 0 with product 1,

E (Xi-ij)2 >Exi ~n-


l<i<j<n i=l
A generalization of Turkevici’s inequality
112 Solutions

Solution,'.
Of course, the inequality can be written in the following form
n 2
> -(n- 1) (^x'l
n +
\k I ik- 1

.. We will prove this inequality by induction. The case n •- 2 is trivial. Suppose- the •
inequality is true for n - 1 and let us prove it for n. Let
/ n 2

/(xi,X2,...,x„) - —(n — 1) xk +
,k=l

and let G - "'^/x-jx^... x„, where we have already chosen = min{ii,i2> •.. ,x,t}.
It is easy to see that the inequality /(xi,i'2> • ■ • >^>1) S /(xi, G, G,..., G) is equivalent
to -
n 2 (n \

(n~ Jt=2
S1*
~ (,fc=2 -2x1 (- (n~
\k=2 /
l•

n
Clearly, we have Xi < G and 52^ > (n - 1)G, so it suffices to prove that
. - *=2
2
(n- 1)52 x2 - ( ^xk I n
>2G £xfc -(n-j)G .
fc=2 Jc=2 \fc=2
We will prove that this inequality holds for all X2,1... ,xn > 0. Because the inequality
is homogeneous, it is enough to prove it when G = 1. In this case, from the induction
step we already have
n 2
(n~2)^x2k+n-l> [
Xk
k=2 ,k=2

and so it suffices to prove that


n n n

52 I- n - 1 > 52 <=> J2(xfc -1)2 > 0,


fc=2 k=2 k=2

clearly true.
Thus, we have proved that J(xi,X2, ■ ■ ■ ,xn) < /(xi, G, G,... ,G). Now, to com­
plete the inductive step, we will prove that /(xi,G, G, ...,G) < 0. Because clearly
Xj = ^,,-T » the last assertion reduces to proving that
2
1 + „>{(„-l)C+d^T
(n- 1) (n- 1)G2 +
G2<"-1)
which comes down to
n -2 2n - 2
G'^n-2 + n - (Jn-2
and this one is an immediate consequence of the AM-GM Inequality.
Old and New Inequalities 113

118. [ Gabriel Dospinescu ] Find the minimum value of the expression


n .--------------------------
flirt2 ■■ ■an
1 — (n - l)a,

where ui,a2,... ,an < ——- add up to 1 and n > 2 is an integer.

Solution:
1
We will prove that, the minimal value is . Indeed, using Suranyi’s In­
nn“:*
equality, we find that
n n
(n 1) a" + naia2 ... an => naia2---an > E"'’ fl - (n — l)a,).

Now, let us observe that

E^o - <n~ i)«k) = E 2


fc=l fc=l (\/l - (n - l)afc

and so, by an immediate application of Holder Inequality, we have


3

E«r
E< > n
1
2-

E 1 - (n - l)ak

But for n > 3, we can apply Jensen’s Inequality to deduce that

E^
LJ------ > k 1
Qk
1
it 71 n
k /
Thus, combining all these inequalities, we have proved the inequality for n > 3.
For n = 3 it reduces to proving that
abc
^Vb + c-a 2E«
which was proved in the solution of the problem 107.

119. [ Vasile Cirtoaje ] Let < 1 be nonnegative real numbers such


that
n'f + ra j -V ... + a'n
a=
n ~ 3
114 Solutions

Prove that
na
-^- + Q2 <<n
1-a? 1 -aS 1 - a2A " 1-a2'

Solution:
We proceed by induction. Clearly, the inequality is trivial for n= 1. Now we
suppose that the inequality is valid for n = k — l,fc>2 and will prove that
aj , a2 ak ka
1 - ajj + • • • +
1 - a2 + ------ 1-a? > 1 - a2’
for
a2 + a| + , ■. -F a?
a =
I-------- -- "T'
We assume, without loss of generality, that ai > «2 > ... > at, therefore a > ak.
Using the notation

a2 + a2 + • • • + a2 ka2 - a2
x-
k-1
v3
from a > at it follows that x > u > —Thus, by the inductive hypothesis, we have
o

<12 Qfe-1 (k — l)x


rF...+ > 1 — x2 ’
T^F2 4+ 1 — a? 1 - a?
and it remains to show that
Uk (k - l)x ka
l~ak 1 — x2 - r^2-
From
ka2 - a2 Q2 ~ «t
x2 — a2 — - - a2—
k-1 k— 1 ’
we obtain
(a-at)(a + afc)
(k - l)(x - a) =
and
_^ + (k - l)x ka Qk a
+ (*-1)
x a
1-a2 1 - x2 1-a2 1 - a2 1 - a2 1 - x2 1-a2
-(a - Ofc)(l I- aat) , (k - l)(x - a)(l -F ax) a - Qk [-(lb aafc) (a 4- at)(l + ax)' _
(l-a2)(l- a2) + + (l -x2)(l - a2) “ F^a2 1-a2
+ (l-x2)(x + a)
(a - aji)(x - afc)[— 1 + x2 + a2 + xat + a(x -F at) + a2 -F axat(x + at) -F a2xafc]
(1 - a2)(l -a?.)(l -x2)(x + a)

Since x2 - a* =
ka2-«i -a;.= fc(°-°4(°-m)|ilfollowsthtt
k— 1
_ A-(a - at)2(a + at)
(o - afc)(x - afc) >0,
(k- l)(x + at)
Old and New Inequalities 115

and hence we have to show that


x2 4- ak 4- xak 4- a(x 4- ak) + a2 + axak(x 4- ak) 4- a2xak > 1.

In order to show this inequality, we notice that


ka2 + (k - 2)a2 ka2
x2 + a2k =
k— 1 -
and
x I ak > y/x2 I a2k > a^/

Consequently, we have
x2 4- uk + xak 4- a(x +ak) 4- a2 4- axak(x + u*) 4- a2xa* > (x2 4- a2) 4- u(x 4- «<•) 4- a2 >

k —■ l 1 | a2 > 3a'2 1,
> A- -1 '* k- I /
and the proof is complete. Equality holds when ai = a2 = • • • = an.
Remarks.
1. From the final solution, we can easily sec that the inequality is valid for the
larger condition
1
a>
14- n-1 n | u
n-1

This is the largest range for a, because for ay = 02 = ••• = an_i = x and an = 0
‘ I
(therefore a = x -------), from the given inequality we get a >
n n i n
7TZT + 77ZT

v3
2.‘ The special case n = 3 and a = — is a problem from Crux 2003.
U

120. | Vasilc Cirtoajc, Mircea Lascu | Let a,b, c, x,y, z be positive real numbers
such that
(a 4- b + c)(x 4- y + z) = (a2 4- b2 4- c2)(x2 4- y2 4- z2) = 4.

Prove that
, 1
abexyz < —.

Solution:
Using the AM-GM Inequality, we have
4(a64-6c+ca)(xy-t-yz4-zx) = [(a 4- b -r c)2 - (a2 -t- b2 4- c2)j [(x + y + z)2 - (x2 - y2 4- z2)] =

= 20 - (a 4- b 4- c)2(x2 4- y2 4- z2) — (a2 4- b2 4- c2)(x 4- y 4- z)2 <


< 20 - 2\/(q 4- 6 + c)2(x2 4- y2 4- z2)(a2 4- b2 4- c2)(x 4- y 4- zj2 = 4,
116 Solutions

therefore
(ab 4- be 4- ca)(zy 4- yz 4- za) < 1. (I)

By multiplying the well-known inequalities

(ab + be 4- ca)2 > 3abc(a 4- b + c), (ay + yz -I- zz)2 > 3zyz(z + y + z),

it follows that

(ab + be 4- ca)2(zy + yz 4- zz)2 > §abcxyz(a 4- b + c)(z 4- y 4- z),

or
(ab 4- be + ca)(zy 4- yz 4- zz) > 36abczyz. (2)
From (1) and (2), we conclude that

1 < (ab 4- be 4- ca)(zy 4- yz 4- zz) > 36abczyz,

therefore 1 < 3Gabcx.yz.


To have I - 3Gabcxyz, the equalities (ab 4- be 4- ca)2 = 3abc(a 4- b 4- c) and
(zy 4- yz 4- zz)2 = 3zyz(z 4- y 4- z) are necessary. But these conditions imply a = b = c
and x = y = z, which contradict the relations (a 4- b 4- c)(z 4- y 4- z) = (a2 4- b2 4-
c2)(z2 4- y2 4- z2) = 4. Thus, it follows that 1 > SGabexyz.

121. [ Gabriel Dospinescu ] For a given n > 2, find the minimal value of the
constant kn, such that if xi,x2 xn > 0 have product 1, then
1 1 ‘ 1
— 4-
-f" ~ 4-4- • •••
• * 4" — <n-l.
Vi 4- I'nZi x/1 4- knx2
VI VI 4- k„xn
Mathlinks Contest

Solution:
2n- 1
We will prove that kn . Taking zi z2 • zn ~ 1, we find that
(n-l)2
2n - 1
^'n — So, it remains to show that
(»-l)2-
n
1
E 1 4-
2n - 1
= <n-l
Xk
(n - 1)2
if -Tl ■ *2 xu = 1. Suppose this inequality doesn’t hold for a certain system of n
numbers with product 1, zi,z2,... ,x„ > 0. Thus, we can find a number M > n - 1
and some numbers a, > 0 which add up to 1, such that
1____
= = Mak.
2n - 1
1 4-
(^Ip Xk
Old auid New Inequalities

i
Thus, ak < and we have
n-- 1
n n
f(n-l)2 , 1 2n - 1 I
i=n fc=l L
2n-.l 1 M2al1 1
=>
(n-1)2
<
(n- 1)M
- 1

n
Now, denote 1 — (n — l)at = 6* > 0 and observe that ^6* — 1. Also, the above

inequality becomes
2

(n — l)2n
k=l
fe* • JJ (2— &*..)> (2n - l)n
fc=l ,k-l
IPl-M •
Because from the AM-GM Inequality we have
n
f[(2-W< ( 2n n- 1
our assumption leads to
(n-l)I2; "
n(i-bk)2 nn
k=l
So, it is enough to prove that for any positive numbers ai, a2,..., an the inequality
n
(n - l)2n
P[ (di + a-2 4------ F ajt -1 -F a*.- i i 4------F an)2 > aiftt ... an(ai Faj -I------ F nn)n
nn
holds.
This strong inequality will be proved by induction. For n = 3, it follows from the
fact that

(iW’ >
abc abc 27 k2—J /
Suppose the inequality is true for all systems of n numbers. Let ai,a2,... ,an+i
be positive real numbers. Because the inequality is symmetric and homogeneous, we
may assume that ai < a? < ■■ ■ < an+i and also that ai 4 aj F • • • F an = 1. Applying
the inductive hypothesis we get the inequality
(n - l)2n
I[(l-a.)2> —---- aia-2...an.

To prove the inductive step, we must prove that


" „2nt2
H(a„, + l-a.)2>—+- yataa ... a„a„+i(l 4- a,l+i)n+l

Thus, it is enough to prove the stronger inequality


an l I 2 n-:‘,,+2
n
1+ ~—i-^r;-(nVi)"‘^an+,(1 + an-,),,T
1 - a. (T^iy
118 Solutions

Now, using Huygens Inequality and the AM-GM Inequality, we find that
/ V"
2 2n
tlO>i+1
1 I > 1 I > 1 I
1 - «> n-1
\ f[(l “*»*)
\ /
and so we arc left with the inequality
/ ___ \ 2n n3nt2
n«n+i
1+ > ■a>i+l (1 + <ln+l)n 1 1
n- 1 (n - l)2n ■ (n + l)n+1
r , 1 c "C1 + an+l)
if “n-n > maxfaj,n2l...,an) > — • bo, we can put---- —-------- = 1 -(- x, where x is
nonnegative. So, the inequality becomes
2n
z 1 + (n + l)z
1+ >
n(z + 1) (z + I)""1
Using Bernoulli Inequality, we find immediately that
z \2n > 3z + l
1+
n(z 4-1)/ — x + 1
Also, (1 -I z) > 1 I- (n - l)z and so it is enough to prove that
3z + 1 1 4- (n + l)z
>
z + 1 ~ 1 + (n - l)z
which is trivial.
So, we have reached a contradiction assuming the inequality doesn’t hold for a
1 certain system of n numbers with product 1. which shows that in fact the inequality
*2ti “ 1
is true for k„ = -------- — and that this is the value asked by the problem.
(n-1)-
Remark.
For 7i ~ 3. we find an inequality stronger than a problem given in China Math­
ematical Olympiad in 2003. Also, the case n - 3 represents a problem proposed by
Vasile Cartoaje in Gazela Matematica, Seria A.

122. | Vasilc Cirtoajc, Gabriel Dospincscu | For a given n > 2, find the maximal
value of the constant kn such that for any Z;, x2,..., xn > 0 for which z, + z2 4------ 1-
z„ — 1 we have the inequality
(1 -Zi)(l-z2)...(l -rn) > knx\x2...xn.
Old and New Inequalities 119

Solution:
We will prove that this constant is (y/n - l)n. Indeed, let a, = z2. We must find
the minimal value of the expression

n(i-v^)

»=1
when ai +02H----- Fan = 1. Let us observe that proving that this minimum is (y/n— l)n
reduces to proving that

— a<) > (y/n - l)n • U y/ai ■ JJ(1 + y/ai).

But from the result proved in the solution of the problem 121, we find that
(n-1)2
f[(l n
So, it is enough to prove that
n
i + -4
But this is an easy task, because from the AM-GM Inequality we get
/ A _\n
n
f[(l I- x/£) < 1 I -I—!— < ni
71 Vn

\ 7
the last one being a simple consequence of the Cauchy-Schwarz Inequality.
Remark.
The case n = 4 was proposed bj’ Vasile Cartoaje in Gazeta Matematica Annual
Contest, 2001.
Glossary

(1) Abel’s Summation Formula


If ai,G2)... ,an, bi,b2,—,bn are real or complex numbers, and

Si = G] + (12 + ... + Gj,l = 1,2, n,

then
n n-1
52 — b,+i) + Snbn.

(2) AM-GM (Arithmetic Mean-Geometric Mean) Inequality


If Gi,02,.-,an are nonnegative real numbers, then
1 n
- Ea* > (ala2...an)”,

with equality if and only if oi = 02 = ... = a, This inequality is a special


case of the Power Mean Inequality.

(3) Arithmetic Mean-Harmonic Mean (AM-HM) Inequality


If ai,a2,...,an are positive real numbers, then
i n 1
■. ■
n 1
1-1 1
n 52-
with equality if and only if ai = a2 = = an. This inequality is a special
case of the Power Mean Inequality.

• . (4) Bernoulli’s Inequality


For any real numbers x > — 1 and a > 1 we have (1 + x)° > 1 + ax.

121
122 Solutions

(5) Cauchy-Schwarz’s Inequality


For any real numbers O], 02,..., an and bi, 62,.... b„
(a? + Oj + ... + a2)(b] + b^ + ... + b2) >
> (ojbi + O2b2 + ... + anbn)2,
with equality if and only if a, and b, are proportional, i = 1,2, n.

(G) Cauchy-Schwarz’s Inequality for integrals


If a,b are real numbers, a < b, and f,g : [a, b] —» R are integrable functions,
then
2
■b ■b \ yb \
/(x)g(x)dx < f2(x)dx I • I g2(x)dx .
a J

(7) Chebyshev’s Inequality /


If ai < 02 < ... < On and bi, 62,..., are real numbers, then
■ ' ' n 1 n n \
l)If bi < b-j, < ... < b„ then ^Ojbj > -

’* 11 / n / n \
2)If bi > b? > ... > bn then ^Ja.b, < -

(8) Chebyshev’s Inequality for integrals


If a,b are real numbers, u < b, and f,g : [a, b] —» R are integrable functions
and having the same monotonicity, then
r-b fb rb
(6- a) / f(^)g(x)dx > / f(x)dx ■ / g(x)dx
Ja Ja J <x
and if one is increasing, while the other is decreasing the reversed inequality
is true.

(9) Convex function


A real-valued function f defined on an interval / of real numbers is convex
if, for any x,y in / and any nonnegative numbers a,/3 with sum 1,
f(ar + 0y) < af(x) + 0f(y).

(10) Convexity
A function /(x) is concave up (down) on [a,b] C R if /(t) lies under
(over) the line connecting (ai,/(aj)) and (bi,/(bi)) for all
a < oi < x < bi < b.
A function 5(1) is concave up (down) on the Euclidean plane if it is concave
up (down) on each line in the plane, where we identify the line naturally
Old and New Inequalities 123

with R.
Concave up and down functions are also called convex and concave, re-
spectively.
If f is concave up on an interval [n,b) and Ai,A2,...,An arc nonnegative
numbers wit sum equal to 1, then

Ai/(xi) -F A2/(z2) I- — + An/(zn) > f(XtXi 1- A2z2 + ... I Anzn)

for any xi,x2,-.-,xn in the interval [a,b]. If the function is concave down,
the inequality is reversed. This is Jensen’s Inequality.

(11) Cyclic Sum


Let n be a positive integer. Given a function f of n variables, define the
cyclic sum of variables (zi, x2,xn) as

53/(®i.X3. ••.,»«) = /(xi,i2,...,a:n) + f(x2,x3, ...,zn,zi)


eye

+ ■■• + a2> •••> an-i).

(12) Holder’s Inequality


1 1
If r,s are positive real numbers such that —I— = 1, then for any positive
real numbers ai, a2,..., an,
bi,b2,.. . ,bn,
i
n / n \!
Y,aibi
t-1 i i 1 1

n
< n n
/ /
. (13) Huygens Inequality
If Pi,P2,---,Pn,a\,a2,...,On,b\,b2,...,bn are positive real numbers with
Pi + P2 + ... + pn = 1, then

n(ai + Mp- >f[ar + n^«

(14) Mac Laurin’s Inequality


For any positive real numbers zj-, z2,. -.,in>

Si > S2 > . • • > Sn,


124 Solutions

where
E
• 1 < 11 <i? <-«L<n
X/, ‘ Xj3 ' • • • ' Xf*

sfc= ‘

~0
(15) Minkowski’s Inequality
For any real number r > 1 and any positive real numbers
Oi,O2,... ,an, bi, 6'2>••• i^n,
1 1
n

E(a* + f,‘)r + E^_-


(16) Power Mean Inequality
For any positive real numbers ai,02l...lan with sum equal to 1, and
any positive real numbers zi,Z2,...,xn» we define Mr = (ai^i + 02X2 +
... 4- auZj’Jr jf r is a non-zero real and Mo — Zi'z$a ...z“n, =
niaz{zl, X2,. ■ •, in), M-x = min{xi, x2, ■. ■, xn}. Then for any reals s < t
we have A/_0O < Ms < Mt < Moo.

(17) Root Mean Square Inequality


If ai,02,-..an are nonnegative real numbers, then
/Qi T a‘2 + - + “n > Cll + 0-2 + ... + an

V n ~ n
with equality if and only if ai — «2 - ••• — o.n.

(18) Schur’s Inequality


For any positive real numbers x,y, z and any r > 0, xr(x-y)(x-z) + yr(y-
i)(y - x) 4- zr(z - x)(z - y) > 0. The most common case is r = 1, which has
the following equivalent forms:

1) x3 4- y3 I- z3 -I- 3x1/2; > xy(x 4- y) + yz{y 4- z) 4- zx{z 4- x);


2) xyz > (x 4- y - z)(y 4- z - z)(z 4- x - y);
1 -I- 9xyz
3) ’ if x 4- y 4- z = 1 then xy + yz + zx <
4

(19) Suranyi’s Inequality


For any nonncgalivc real numbers ai,a2> <zn,
n

+nIIftfc - (E a* ■ (E<’
*=1 fc=l >=1
Old and New Inequalities 125

(20) Turkevici’s Inequality


For any positive real numbers x, y, z, t,
x4 + y4 + z4 + L4 + 2xyzl > x2y2 + y2z2 + z2t2 + t2x2 +- x2z2 + y2l2.

(21) Weighted AM-GM Inequality


For any nonnegative real numbers a\, a2,.... an, if Wi, w2, ■■■, are nonneg­
ative real numbers (weights) with sum 1, then
Wjo.1 + W2a2 + ••• I H'nan > '’j'1 "2? ■ • • an" >
with equality if and only if ai ~ a2 ~ ••• - On-

Further reading

1. Andrcescu, T.; Feng, Z., 101 Problems in Algebra from the Training of the USA
IMO Team, Australian Mathematics Trust, 2001.
2. Andreescu, T.; Feng, Z., USA and International Mathematical Olympiads 2000,
2001, 2002, 2003, MAA. -
3. Andreescu, T.; Feng, Z., Mathematical Olympiads: Problems and Solutions from
around the. World, 1998-1999, 1999-2000, MAA.
4. Andreescu, T.; Kcdlaya, K., Mathematical Contests 1995-1996, 1996-1997,
1997-190S: Olympiad Problems from around the World, with Solutions, AMC.
5. Andreescu, T.; Enescu, B., Mathematical Olympiad Treasures, Birkhauscr, 2003.
6. Andreescu, T.; Gelca, R., Mathematical Olympiad Challenges, Birkhauser, 2000.
7. Andreescu, T.; Andrica, D., 360 Problems for Mathematical Contests, GIL
Publishing House, 2002.
8. Becheanu, M., Enescu, B., Inegalitali elementare.. .gi mai pulin elementare, GIL
Publishing House, 2002
9. Bcckenbach, E., Bellman, R. Inequalities, Springer Verlag, Berlin, 1961
10. Drimbe, M.O., Inegalitali idei gi metode, GIL Publishing House, 2003
11. Engel, A., Problem-Solving Strategies, Problem Books in Mathematics, Springer,
1998. • 1
12..G.H. Littlewood, J.E. Polya, G., Inequalities, Cambridge University Press, 1967.
13. Klamkin, M., International Mathematical Olympiads, 1978-1985, New
Mathematical Library, Vol. 31, MAA, 1986;
14. Larson, L. C., Problem-Solving Through Problems, Springer-Verlag, 1983.
15. Lascu, M., Inegalitali, GIL Publishing House, 1994
16. Liu, A., Hungarian Problem Book III, New Mathematical Library, Vol. 42,
MAA, 2001. • ' ■
i 17. Lozansky, E; Rousseau, G„ Winning Solutions, Springer, 1996.
18. Mitrinovic, D.S., Analytic inequalities, Springer Verlag, 1970
I
19. Panaitopol, L., Bandila, V., Lascu, M., Inegalitali, GIL Publishing House, 1995
20. Savchcv, S.; Andreescu, T., Mathematical Miniatures, Anncli Lax New
Mathematical Library, Vol. 43, MAA.
www.mathlinks.ro - Powered by www.gil.ro

I 127
G-IL
www.gil.ro

EDITORA
)

I.S.B.N. 973-9417-35-3 www.vestseller.com.br

You might also like